You are on page 1of 170

90-99 TOEFL

tsuvrq

)/ gter, ,

wwwwjs wwwwjs@sina.com1 April 11, 2001

&

'- + !

%%

@ ? 3SW
PART A . . . . . . . . . . . . . . . . . . . . . . . . . . . . . . . . . . . . . . . . x1.2 PART B . . . . . . . . . . . . . . . . . . . . . . . . . . . . . . . . . . . . . . . . x1.3 PART C . . . . . . . . . . . . . . . . . . . . . . . . . . . . . . . . . . . . . . . .
90
x1.1

90

A 1 UM7K OY

7 7 9

35W

PART A . . . . . . . . . . . . . . . . . . . . . . . . . . . . . . . . . . . . . . . . 13 x2.2 PART B . . . . . . . . . . . . . . . . . . . . . . . . . . . . . . . . . . . . . . . . 13 x2.3 PART C . . . . . . . . . . . . . . . . . . . . . . . . . . . . . . . . . . . . . . . . 15


90

x2.1

A 5 UM7K OY

13

3EW

PART A . . . . . . . . . . . . . . . . . . . . . . . . . . . . . . . . . . . . . . . . 17 x3.2 PART B . . . . . . . . . . . . . . . . . . . . . . . . . . . . . . . . . . . . . . . . 17 x3.3 PART C . . . . . . . . . . . . . . . . . . . . . . . . . . . . . . . . . . . . . . . . 19


91

x3.1

A 10 UM7K OY

17

3IW

PART A . . . . . . . . . . . . . . . . . . . . . . . . . . . . . . . . . . . . . . . . 23 x4.2 PART B . . . . . . . . . . . . . . . . . . . . . . . . . . . . . . . . . . . . . . . . 23 x4.3 PART C . . . . . . . . . . . . . . . . . . . . . . . . . . . . . . . . . . . . . . . . 25


91

x4.1

A 1 UM7K OY

23

3QW

PART A . . . . . . . . . . . . . . . . . . . . . . . . . . . . . . . . . . . . . . . . 27 x5.2 PART B . . . . . . . . . . . . . . . . . . . . . . . . . . . . . . . . . . . . . . . . 27 x5.3 PART C . . . . . . . . . . . . . . . . . . . . . . . . . . . . . . . . . . . . . . . . 29


91

x5.1

A 5 UM7K OY

27

3=W

PART A . . . . . . . . . . . . . . . . . . . . . . . . . . . . . . . . . . . . . . . . 33 x6.2 PART B . . . . . . . . . . . . . . . . . . . . . . . . . . . . . . . . . . . . . . . . 33 x6.3 PART C . . . . . . . . . . . . . . . . . . . . . . . . . . . . . . . . . . . . . . . . 35


91

x6.1

A 8 UM7K OY

33

3CW

PART A . . . . . . . . . . . . . . . . . . . . . . . . . . . . . . . . . . . . . . . . 37 x7.2 PART B . . . . . . . . . . . . . . . . . . . . . . . . . . . . . . . . . . . . . . . . 37 x7.3 PART C . . . . . . . . . . . . . . . . . . . . . . . . . . . . . . . . . . . . . . . . 39 3

x7.1

A 10 UM7K OY

37

c;b
91

31W
x8.1

A 1 UM7K OY

43

PART A . . . . . . . . . . . . . . . . . . . . . . . . . . . . . . . . . . . . . . . . 43 x8.2 PART B . . . . . . . . . . . . . . . . . . . . . . . . . . . . . . . . . . . . . . . . 43 x8.3 PART C . . . . . . . . . . . . . . . . . . . . . . . . . . . . . . . . . . . . . . . . 45

39W
x9.1

92

A 5 UM7K OY

49

PART A . . . . . . . . . . . . . . . . . . . . . . . . . . . . . . . . . . . . . . . . 49 x9.2 PART B . . . . . . . . . . . . . . . . . . . . . . . . . . . . . . . . . . . . . . . . 49 x9.3 PART C . . . . . . . . . . . . . . . . . . . . . . . . . . . . . . . . . . . . . . . . 51

3GW
x10.1

92

A 8 UM7K OY

53

PART A . . . . . . . . . . . . . . . . . . . . . . . . . . . . . . . . . . . . . . . . 53 x10.2 PART B . . . . . . . . . . . . . . . . . . . . . . . . . . . . . . . . . . . . . . . . 53 x10.3 PART C . . . . . . . . . . . . . . . . . . . . . . . . . . . . . . . . . . . . . . . . 55

3GSW
x11.1

92

A 10 UM7K OY

59

PART A . . . . . . . . . . . . . . . . . . . . . . . . . . . . . . . . . . . . . . . . 59 x11.2 PART B . . . . . . . . . . . . . . . . . . . . . . . . . . . . . . . . . . . . . . . . 59

3G5W
x12.1

93

A 1 UM7K OY

61

PART A . . . . . . . . . . . . . . . . . . . . . . . . . . . . . . . . . . . . . . . . 61 x12.2 PART B . . . . . . . . . . . . . . . . . . . . . . . . . . . . . . . . . . . . . . . . 61 x12.3 PART C . . . . . . . . . . . . . . . . . . . . . . . . . . . . . . . . . . . . . . . . 63

3GEW
x13.1

93

A 5 UM7K OY

67

PART A . . . . . . . . . . . . . . . . . . . . . . . . . . . . . . . . . . . . . . . . 67 x13.2 PART B . . . . . . . . . . . . . . . . . . . . . . . . . . . . . . . . . . . . . . . . 67 x13.3 PART C . . . . . . . . . . . . . . . . . . . . . . . . . . . . . . . . . . . . . . . . 69

3GIW
x14.1

93

A 8 UM7K OY

73

PART A . . . . . . . . . . . . . . . . . . . . . . . . . . . . . . . . . . . . . . . . 73 x14.2 PART B . . . . . . . . . . . . . . . . . . . . . . . . . . . . . . . . . . . . . . . . 73 x14.3 PART C . . . . . . . . . . . . . . . . . . . . . . . . . . . . . . . . . . . . . . . . 75

c;b 3GQW
x15.1

5
93

A 10 UM7K OY

79

PART A . . . . . . . . . . . . . . . . . . . . . . . . . . . . . . . . . . . . . . . . 79 x15.2 PART B . . . . . . . . . . . . . . . . . . . . . . . . . . . . . . . . . . . . . . . . 79 x15.3 PART C . . . . . . . . . . . . . . . . . . . . . . . . . . . . . . . . . . . . . . . . 81

3G=W
x16.1

94

A 1 UM7K OY

85

PART A . . . . . . . . . . . . . . . . . . . . . . . . . . . . . . . . . . . . . . . . 85 x16.2 PART B . . . . . . . . . . . . . . . . . . . . . . . . . . . . . . . . . . . . . . . . 85 x16.3 PART C . . . . . . . . . . . . . . . . . . . . . . . . . . . . . . . . . . . . . . . . 87

3GCW
x17.1

94

A 5 UM7K OY

89

PART A . . . . . . . . . . . . . . . . . . . . . . . . . . . . . . . . . . . . . . . . 89 x17.2 PART B . . . . . . . . . . . . . . . . . . . . . . . . . . . . . . . . . . . . . . . . 89 x17.3 PART C . . . . . . . . . . . . . . . . . . . . . . . . . . . . . . . . . . . . . . . . 91

3G1W
x18.1

94

A 8 UM7K OY

95

PART A . . . . . . . . . . . . . . . . . . . . . . . . . . . . . . . . . . . . . . . . 95 x18.2 PART B . . . . . . . . . . . . . . . . . . . . . . . . . . . . . . . . . . . . . . . . 95 x18.3 PART C . . . . . . . . . . . . . . . . . . . . . . . . . . . . . . . . . . . . . . . . 97

3G9W
x19.1

95

A 5 UM7K OY

101

PART A . . . . . . . . . . . . . . . . . . . . . . . . . . . . . . . . . . . . . . . . 101 x19.2 PART B . . . . . . . . . . . . . . . . . . . . . . . . . . . . . . . . . . . . . . . . 101 x19.3 PART C . . . . . . . . . . . . . . . . . . . . . . . . . . . . . . . . . . . . . . . . 103

35GW
x20.1

95

A 8 UM7K OY

107

PART A . . . . . . . . . . . . . . . . . . . . . . . . . . . . . . . . . . . . . . . . 107 x20.2 PART B . . . . . . . . . . . . . . . . . . . . . . . . . . . . . . . . . . . . . . . . 110

35GSW
x21.1

95

A 10 UM7K OY

115

PART A . . . . . . . . . . . . . . . . . . . . . . . . . . . . . . . . . . . . . . . . 115 x21.2 PART B . . . . . . . . . . . . . . . . . . . . . . . . . . . . . . . . . . . . . . . . 118 x21.3 PART C . . . . . . . . . . . . . . . . . . . . . . . . . . . . . . . . . . . . . . . . 119

c;b
96
x22.1

35G5W

PART A . . . . . . . . . . . . . . . . . . . . . . . . . . . . . . . . . . . . . . . . 123 x22.2 PART B . . . . . . . . . . . . . . . . . . . . . . . . . . . . . . . . . . . . . . . . 126 x22.3 PART C . . . . . . . . . . . . . . . . . . . . . . . . . . . . . . . . . . . . . . . . 128


96

A 1 UM7K OY

123

35GEW
x23.1

PART A . . . . . . . . . . . . . . . . . . . . . . . . . . . . . . . . . . . . . . . . 131 x23.2 PART B . . . . . . . . . . . . . . . . . . . . . . . . . . . . . . . . . . . . . . . . 134 x23.3 PART C . . . . . . . . . . . . . . . . . . . . . . . . . . . . . . . . . . . . . . . . 135


96
x24.1

A 5 UM7K OY

131

35GIW

PART A . . . . . . . . . . . . . . . . . . . . . . . . . . . . . . . . . . . . . . . . 139 x24.2 PART B . . . . . . . . . . . . . . . . . . . . . . . . . . . . . . . . . . . . . . . . 142 x24.3 PART C . . . . . . . . . . . . . . . . . . . . . . . . . . . . . . . . . . . . . . . . 143


98

A 10 UM7K OY

139

35GQW
x25.1

PART A . . . . . . . . . . . . . . . . . . . . . . . . . . . . . . . . . . . . . . . . 147 x25.2 PART B . . . . . . . . . . . . . . . . . . . . . . . . . . . . . . . . . . . . . . . . 150 x25.3 PART C . . . . . . . . . . . . . . . . . . . . . . . . . . . . . . . . . . . . . . . . 152


98

A 5 UM7K OY

147

35G=W
x26.1

PART A . . . . . . . . . . . . . . . . . . . . . . . . . . . . . . . . . . . . . . . . 155 x26.2 PART B . . . . . . . . . . . . . . . . . . . . . . . . . . . . . . . . . . . . . . . . 158


99

A 8 UM7K OY

155

35GCW
x27.1

PART A . . . . . . . . . . . . . . . . . . . . . . . . . . . . . . . . . . . . . . . . 163 x27.2 PART B . . . . . . . . . . . . . . . . . . . . . . . . . . . . . . . . . . . . . . . . 166 x27.3 PART C . . . . . . . . . . . . . . . . . . . . . . . . . . . . . . . . . . . . . . . . 168

A 1 UM7K OY

163

4TX 90 B 1 VN8L<PZ
1990 1 * TOEFL

"0
x

1.1 PART A

1. I must have left my keys at my sister's house. 2. We need to arrive early for the graduation ceremony. 3. Sarah spent twice as much time on computer as I do. 4. Let's listen to what Robert has to say. 5. I told you two to cut it out. 6. No one in the world could you expect to nd a harder worker than Lois. 7. Dean Williams was invited by the college president to address the faculty. 8. I'll have this nished in a little while. 9. He's sick of his job. 10. Framed posters make interesting decorations, don't you think? 11. That's precisely the kind of camera I brought with me. 12. If he 'd only give our quizzes back. 13. I didn't mean to cause you so much trouble. 14. It's fun to sit in the students' section at the football game. 15. There will be someone at the airport to meet us, won't there? 16. When will you ever learn! 17. I don't recall locking my suitcase. 18. To pay to see that movie would be foolish, when you can see it on television for nothing. 19. I didn't know that both reports were due today. 20. The classes should have shorter sessions.
x

1.2 PART B

21. M: Do you want the same cut as last time. W: The same on top, but I'd like it a little longer over the ears and in the back. Q: Where does this conversation probably take place. 22. W: We are supposed to meet John here at the train station. 7

\mo 90 d 1 nj^i`kp

M: That's like looking for a needle in a hay stack. Q: What does the man suggest about John? 23. M: Did you say you were driving to town this morning? W: Yes, I had to get a check cashed to pay my bookstore bill. Q: What is the woman going to do in town? 24. W: I'm thinking of take ve courses next semester. M: Wouldn't four be wiser? Q: What does the man imply about the Courses? 25. M: What are you working on so diligently? W: An editorial for the newspaper. If I miss the midnight deadline, it won't be printed until next week. Q: What is the woman trying to do? 26. M: Have you seen the author's latest bestseller? W: I've just nished it, I really recommend it. Q: What does the man and woman discussing? 27. W: You didn't have too much trouble doing that experiment, did you? M: It only took me most of the day. Q: What does the man and woman discussing? 28. M: There are terrible light bulbs. I have to replace the one in this lamp. W: What about checking the wiring in the lamp. Q: What does the woman suggest? 29. W: Where can I nd a map of the university campus? M: Have you tried the information certre? Q: What does the man imply about the information o ce? 30. W: could I hand in my paper a few days late? W: I'm afraid that's out of the question. Q: What does the woman mean? 31. M: We've sure been having a lot of rain lately. W: Haven't we, ever. Q: What do we learn form the woman's response? 32. W: I can't believe your room is such a mess. M: That's Joan's room.

1.3 PART C

Q: What does the woman mean? 33. W: did you get the part you tried out for in the play. M: That role was given to someone else, but I got a better part. Q: How does the man probably feel? 34. W: A policeman saw you go through that red light. M: It was yellow. Anyway he turned left at the last cornor. Q: What will the driver probably do now? 35. W: Thanks for the note you left for me. M: Oh, then you did get it. Q: What had the man assumed?
x

1.3 PART C

Welcome to Physiology 100. I'm Doctor Ann Roberts. I assume everyone here is a sophomore, since this is the Nursing Department second-year physiology course. If you are rst-year nursing students, please see me after class about transferring into introductory course. Okay, I'd like to begin my rst lecture by introducing two important terms, vitalism versus mechanism. How many of you have heard of these terms? Oh, I see some of you have. Well, vitalism and mechanism are two completely opposite approaches to human physiology. According to the vitalist, the laws of physics and chemistry alone can not explain the processes of life. To the vitalist, there is a so-called vital force, and this vital force is totally separate from that of energy. As you may have guessed, vitalism is a kind of philosophical approach. Mechanism, on the other hand, is the view that all life phenomena, no matter how complex, can be explained according to chemical and physical laws. Since we can support mechanism through scienti c experimentation, the modern scientist tends to be a mechanist. But vitalism is not totally dismissed, especially in elds such as brain physiology, where terms such as human consciousness haven't been de ned yet in physic-chemical terms. I recently read an interesting article dealt with this very question. Is the mind separate from brain, or is the mind only the chemical and physiological workings of the brain? I'd like to continue with this next time, so that we can speak in more detail. I have some copies of that article. And I want you to take it with you and read it for next week. 36. For whom is the lecture intended? 37. When is this lecture given? 38. How would a vitalist view the functions of the mind? 39. What does the professor tell her students to do? W: Art Department, the treasurer Brown speaking.

10

\mo 90 d 1 nj^i`kp

M: Hello, Professor Brown, my name is Frak Meloy. I'm thinking of taking your course in acrylic painting, and I'm calling to nd more about it. W: Have you ever worked with acrylic paints? M: I've done very little painting at all. I'm a chemistry major, but I enjoy drawing and the course description says that any student can sign up. I assume that means that experience in painting isn't necessary. W: Yes, that's right. We'll be using acrylic paints instead of oils because they are easier to handle and dry more quickly. Also, beginning students can get a good feel for what they can do with textures by working with acrylic paints. M: My roommate is also a chemistry major but he has painted quite a bit. If he signed up for the course, would he nd it too easy? W: No, he could work on using techniques he already knows and applying them to principles of composition, color and design. There's something for everyone in this course. M: One more question: Do we need to bring our own brushes and paints to class? W: Yes, you can buy them at the bookstore. I'll provide drawing board and any other supplies that's necessary. I hope you decide to join us. M: It sounds interesting, I'll de nitely register today. W: Fine. See you next Monday afternoon, Don't forget your brushes and paints. 40. Who's the man? 41. What does the man want to nd out? 42. According to the woman, what is one main advantage of the acrylic paints? 43. According to the woman, what can beginning painters do with the acrylic paints 44. What night the man's roommate do in the course? 45. What will the man do before the end of the day? Now I 'd like to talk to you about the nal exam. The exam will be held next Thursday, the last day of the exam week. Remember to bring along two or three pens in case you run out of ink. Unlike the midterm, this test will not include multiple-choice questions. It'll consist entirely of essays. You'll have to answer three of the ve essay- questions. The exam will be comprehensive, which means you'll be responsible for all of the subject matter we've covered in class. I would suggest you review your mid-term's as well as the text book and your class notes, The nal will count for 50 46. When will the exam take place? 47. What should the students bring with them to the exam? 48. What will be format of the exam?

1.3 PART C

11

49. Why does the teacher call the exam comprehensive? 50. When is this talk most likely being given?

12

\mo 90 d 1 nj^i`kp

46X 90 B 5 VN8L<PZ
90 5*

"0
x

2.1 PART A

1. Frank rented a car and drove to Dallas. 2. I saw her practicing the violin. 3. He came over to our house just the other day. 4. Sidney has a tooth removed. 5. How on earth can you believe that? 6. The shop's closed for the night. 7. I thought you wouldn't come to class today. 8. Don failed physics and had to take it over. 9. No sooner had the storm started than all the lights went out. 10. He's planning a graduation party for his sister on the 20th of the next month. 11. The bookstore is out of textbooks for French 102. 12. I really need to brush up on my math. 13. There is nobody on campus who doesn't know Irene. 14. He has a job on the side, preparing the laboratory every day for the next experiment. 15. I can't deny that I was awful in the play. 16. Gloria has been knocking herself out on the project. 17. You are going to nish school this semester, aren't you? 18. Prof. Denkins suggested that I use her name as a reference. 19. Shouldn't we be thinking about leaving for work? 20. Lorraine's family was pleased by her thoughtful gift.
x

2.2 PART B

21. M: It's almost midnight, why don't we leave those dishes until tomorrow? W: A good idea, I'm beat. Q: How does the woman feel? 22. W: I don't think you can build your model in less than a week. 13

14

\]o 90 d 5 nj^i`kp

M: Catherine made he's in three days. Q: What does the man say about Catherine? 23. M: These oranges are good. W: Good? They are the best I've ever had. Q: What does the woman mean? 24. M: Are men's suits on the seventh oor? W: No, they are here on the sixth. Seventh is house-ware's. Q: Where does this conversation take place? 25. W: Have you made any plans yet for summer vacation, Brad? M: Not really. What I'd like to be able to do is to nd a decent job.] Q: What does Brad mean? 26. M: I hope I haven't made this chili too hot for you. W: Too hot for me, it couldn't be. I love spicy food. Q: What does the woman mean. 27. M: Have you run up against any problems in getting your visa renewed. W: Not yet. Q: What does the woman say about her visa. 28. W: Kari, did you turn in your locker key? M: No, I got Sam to do it. Q: What does Kari say about the key? 29. M: Can you see the lake today? W: You could if this fog would only clear. Q: What does the woman mean? 30. M: I don't think Jim particularly likes his cousin. M: There have been hard feelings between them for years. Q: What does the woman say about Jim and his cousin? 31. W: All of your classmates seem so enthusiastic about running in the race. M: But in the end only three of them actually took part. Q: What does the man say about his classmates. 32. M: I'm no surprised you didn't care much for horror movies as a rule? W: I did too. I don't care much for horror movies as a rule? Q: What does the woman mean?

2.3 PART C

15

33. W: Did you nd the tie you wanted? M: No, I didn't have any luck at all. Q: What does the man mean? 34. M: Should I buy this new exercise record for Linda for her birthday? W: She already has the cassette. Q: What does the woman mean? 35. W: What's this I hear about your appearing on the six o'clock news. M: Oh that, Some people were lming something on campus and I just happened to pass in front of the camera. Q: What does the man mean?
x

2.3 PART C

Hi, I'm Roris Rily, and I'm the president of the college environmental awareness Club. Before we get started, I'd like to thank everyone for coming down to the lake today to help with the cleaning up project. As you probably know, we're going to be cleaning up all of the garbage here at the lake area. We'll begin by dividing into two ten-person teams, one team will pick up litter in the picnic are and jogging trail, and one will dig out garbage found in the shallow waters of the lake itself. We'll be putting the litter in these plastic trash bags. Toss any aluminum cans you nd into these boxes, and we'll take them to the recycling center later. A local business has donated these "Do Not Litter" signs to the club. So we'll need a couple of people to place them around the lake when we are through. Ok, everyone who wants to clean up around picnic tables and along the jogging path, raise your hands. 36. How will the club members work to clean up the area? 37. Where will the club members put litter? 38. What will the club members do with the aluminum cans? 39. Where will the club members put up the "Do Not Litter" signs? W: David, you play the cello, don't you? M: I did it for about six years. But I haven't practiced much since I came to college. Why do you ask? W: I'm, signing up for a non-credit string ensemble course that meets once a week on Wednesday night. We have several violist, violas and two bass players, but only on cello. M: Who's directing the group? W: Janit Hanson. Maybe you've heard of her. She plays violin in the city orchestra, and she also directs three other small local music groups.

16

\]o 90 d 5 nj^i`kp

M: Mh, I have my cello here, but I think I would need to do a lot of work before my playing would sound any good at all. W: Miss Hanson will give individual instruction as well as teach us as a group. There are only ten of us now. Try to join us. We are all on the intermediate level. And anyway you won't get a grade for the course. M: I'll tell you what. I'll play a little this evening and see how it goes. W: I'm sure you'll be ne, I'll expect to see you at seven o'clock tomorrow in room 14 at the ne arts building. I hope you can come to our regular meeting. 40. Why does the woman want David to sign up for the course? 41. Why is David not sure that he wants to sign up? 42. How often will the group meet? 43. What instrument does the director of the group play in the city orchestra? 44. What is the level of musical pro ciency of the most of the group members? 45. What will David do this evening? Today, I'll be talking about the invention of the camera and photography. The camera if often thought to be a modern invention. But as early as 1727, a German physicist discovered that light darkened silver salt, a chemical compound. Using as a camera, a big box with a small hole to let the light in, he made temporary images on the salt. Silver salt is still the base of lm today. Then a French scientist made the rst permanent picture by using a special piece of metal sensitized with silver salt. A photography he made in 1826 still exists. The painter Dagera improved on the process by placing common. Salt, the kind we eat, on the metal. This was in 1839, the o cial date of the beginning of photography. But the problem was the printing of the photographs. And it wasn't until other scientists developed the kind of paper we now use that good printing was possible and photography became truly modern. In 1860's, Matthew Brady was able to take his famous pictures of the American Civil War, thus making portrait poses very popular. In the 20th century, George Eastman of the United States simpli ed lm developing, and Edwards Land invented the so-called instant camera with self-developing lm. If we say that the photography came into existence in 1839, it follows that it has taken more than one hundred years for the camera to reach its present condition of technical re nement. 46. What discovery was the basis of photography? 47. How was the rst prominent picture made? 48. What does the speaker regard as the o cial date of the beginning of photography? 49. According to the speaker, why was Matthew Brady remembered today? 50. What did Doctor Edwards Land invent?

4FX 90 B 10 VN8L<PZ
90 10 *

"0
x

3.1 PART A

1.Pat dropped the lettr in the mailbox. 2.Fred gave the right answers to the question the instructor asked. 3.The trip was a wonderful experience,don't you agree? 4.He ate two of them. 5.Cheryl rarely goes with he group to the Saturday night dances. 6.She has to buy a bookbag for school 7.I've decided to hang on to my textbooks form last year, 8.Carol works as a nurse,but she would rather teach. 9.If only this rain would stop. 10.Twelve people came,but half left early. 11.Throuth that door,you will nd someone to help you. 12.Max felt sick and stayed home from school. 13.Everyone except Carrie voted for an exam instead of a paper. 14.This tomato plant is still immature. 15.I'm supposed to turn south at the corner,right? 16.Steve wanted a pair of table lamps,but got a follr lamp instead. 17.I don't need to tell you to come alone,do I? 18.He'll turn his back on me if Iask for money. 19.I signed up for a month long course,but Kary signed up for one,that's eight weeks long. 20.He misread the recipe,so the stew he made tasted terrible.
x

3.2 PART B

21.M:ahuh,is this radio still on the warranty? W:It should be,I bought it only six weeks ago. Q:What is the problem? 2.W:I'm worried about those classed I missed when I was sick. 17

18

\fo 90 d 10 nj^i`kp

M:I'll try to bring you up-to date on what we've done. Q:What does the man mean? 23.M:I'd like to get my sister a nice parctical gift for he birthday. W:Well,how does a hairdryer sound? Q:What does the woman mean? 24.M:I'm getting absolutely nowhere with these physics problems. W:How about my going through them with you? Q:What does the woman mean? 25.W:You've already furnished your apartment? M:I've found some used furniture that was dirt cheap Q:What does the man mean? 26:W:Do you ever think of changing your major to art? M:Every once in a while. Q:What does the man mean? 27:W:Can you make me a cabinet four feet high and three feet wide? M:Sure,how many shelves do you need? Q:What most probably is the man's occupation? 28.M:When is Jack going to nish writing that paper? W:I don't know,he's been working on it day in and day out. Q:What does the woman say about Jack? 29.M:The winters here are usually mild. W:Usually.but Ithink this year would be di erent. Q:What does the woman imply about this winter? 30.M:What shall we have for lunch? W:Is there any chicken soup left? Q:What does the woman mean? 31.W:It's hard to believe that half the calss couldn't do all the exercises,isn't it,Charlie? M:I'm afraid that says something about our physical tness. Q:What does charlie mean? 32.W:Would you mind waiting a few more minutes? M:Not at all Q:What does the man mean?

3.3 PART C

19

33.W:Kevin,did you organise all these les? M:No,I got Frank to do it. Q:What happened to the les? 34.W:Do you ever get as much as a foot of snow here? M:Hardly ever. Q:What does the man mean? 35.W:I was hoping we both would be in the discussion calss M:Me too,Emmy,but by the time I got to registration,the course was closed. Q:What does the man mean?
x

3.3 PART C

36-40 M:You must be preety excited about yur trip to Europe.When is it that you are leaving? W:In just three weeks,and I am excited.But there are still a few things Ineed to do before I go. M:Like waht? W:Like renewing my passport,going to the travel agency to buy my plane ticket and guring out what to do with my apartment while I'm gone M:You are not going to give it up,are you? W:No way,I'll never nd another apartment around here But I don't like the idea of paying three months'rent on an empty apartment,either. M:I don't blame you.Parhaps you could sublet it. W:Yes,but who to? M:MMn,let me think.Oh,I know just the person,An old colleague of mine,Jim Tomas,is coming here to do some research this summer,from June to August. W:That's exactly when I'll be away.It sound ideal,as long as the landlord agrees. M:Tell you what,I'll be calling Jim late this week anyway,so I'll mention it tohime then. W:Well,thanks,Bill,let me know what happens.That extra money will really come in handy? 36.What does the woman need to do at the travel agency? 37.Why doesn't the woman want to give up her apartment entirely? 38.How long will the woman be in Europe? 39.How does the man know Jim Tomas? 40.What will the woman most likely do about her apartment?

20

\fo 90 d 10 nj^i`kp

41-45 Your assignment this trem will be to write two major research papers.One of the most important things about writing a research paper is gibing proper credit for your sources of imformatior.Failure to do this is called piagiarism,which is a form of intellectural dishonesty.Plagiansm is a kind of stealing, or at least an unauthorised borrowing of someone else's ideas.Sometimes inexperience students will plagiarize unitentionally,and then be surprised when the teacher won't accept their papers or give them a failing grade.The best way to avoid unintentional plagiarizing is to be very careful in gathering your information.As you take notes on books or magazine articles about the topic you'veselected.First,try to assimilate the information thoroughly Secondly,Write if down in yur own words.This is called paraphrasing.If you do a good job of paraphrasing you'll capture the main idea from your sources without actually using any phrases from it.Most of your notes should probably be paraphrases however,occassionally,you may nd something you wish to quote directly in yourresearch paper.In this case,be sure that you copy the quotation pricisely in your notes and enclose it in quotation marks That way,when you are nalizing your research paper,you'll be able to remember which of your notes are direct quotes,and which are yur own summary of the material.You can learn to incorporate them appropriately and give the origina authorproper credit. 41.What kind of assignment does the speaker discuss with the students? 42.What dows the speaker warn the students against doing? 43.Whom does the speaker mention as likely plagiarizers? 44.According to the speaker,how should most of the notes be taken? 45.How does the speaker say a direct quotation should be used? 46-50 W:Jim,thank goodness,you've arrived,The class presentation started half an hour ago and I was just beginning to panic. M:I am sorry?I'm late,Allen,This morning has been a realmess,I didn't think I was going to make it here at all W:Why are you late?Our whoe presentauon depends onthese graphes you are holding. M:Yes,I know,I'll tell you about it later.First let'ssee how we are doing to time.Two groups are still ahead of us.aren't they?The presentation on the rights of the consumers and the analysis of the stock market.That means I've got about twenty minutes to thsw out. W:You do look cold What happened? M:I've been standing outside in Aretic temperature for over an hour waiting for a bus. W:Over an hour,But I hought your apartment was only a ten-minute bus ride to your campus.

3.3 PART C

21

M:On normal conditions but the bus was delayed because of the wather and when I stepped into a drugstore to call home for a ride,the bus went bY.As luck would have it,there was no one at home,so I had to war another fourty ve minutes for the next bus. W:That's Murphy's law,isn't it.What was it that said"If anything can go wrong,it will"Well,we still get twenty minutes to gather our wits togather. M:We'd better stop talking,People are turning around and looking at us. 46.What is the woman's tone of voice when the rst sees the man? 47.What are the students doing when the man arrives in calss? 48.What class are the man and woman probably taking? 49.How much time do the man and woman have before they address the class? 50.During what season of the wear does the conversation take place?

22

\fo 90 d 10 nj^i`kp

4JX 91 B 1 VN8L<PZ
1991 1*

"0
x

4.1 PART A

1.I couldn't hear her name clearty. 2.I wouldn't mind having a roommate. 3.Little did she realize that the present was for her. 4.I wonder if you could shut the door. 5.We just can't get over the fact that Ted and Margie got married. 6.I ish I had more time to spend with you. 7.I used to watch a lot of TV,but now I can't stand it. 8.On your right is the information booth. 9.He fell down and hurt himself. 10.What a long bus ride this is! 11.Jane wants to buy that painting? 12.Ruth liked Henry's watch so much that he bought himself one exactly like it. 13.You'll come right on time,won'you? 14.Carrol was told that she had gained the top score. 15.Students outnumbered teachers at the rally. 16.No one could keephime iron speaking. 17.The sea's very calm. 18.She stopped feeling the ducks at eht park. 19.I can't risk not telling him. 20.Je and Alan share a common outlook or life.
x

4.2 PART B

21.W:Will you take biology next semester? M:I've had enouth science courses. Q:What does the man mean? 22.W:The chicken dish's really spicy. 23

24

\ho 91 d 1 nj^i`kp

M:So is the saled. Q:What does the man mean? 23.W:Sam won't be able to play in the basktetball game this week M:Yah,he hur his knee pretty badly a while back,I don't know why he didn't quit right away. Q:What does the man mean? 24.W:Did you go to the big sale at Spener's yesterday? M:I had to visit my aunt in the hospital. Q:What does the woman imply about the sale? 25.W:I'm going to the zoo to make some sketches of elephants today. M:May I go with you?I have the same assignment. Q:What do we learn about the man and woman from this conversation? 26.W:Has Srivia been working hard on her term paper? M:Day in and day out Q:What does the woman say abut Srlvia? 27.W:Can you please tell me where I'd nd gym shoes? M:Yes,they'd be in the sportswear department at the back of the sotre. Q:What is the man doing? 28.W:I would really like to know why Donna is always 50 hostile toward me? M:Did you ever think of just coming right out and asking her? Q:What does the man suggest that the woman do? 29.W:I'm going to be up all night suduying for my exam M:Do you really think that's wise? Q:What does the woman imply? 30.W:What will yu do with yur cat when yu leave for cacation? M:I'm having Ken take care of it. Q:What'll happen to the woman's cat? 31.W:Now that they have soundproofed the room,it sure is a lot easier to work in here. M:You said it! Q:What does the man mean? 32.W:Bill thinks you shouldn't use your good knife to x that. M:Why should he care?It's not his knife. Q:What does the man mean?

4.3 PART C

25

33.W:Shouldn't we invite more people to the dance? M:The more,the merrier. Q:What does the man mean? 34.W:Registration ougint to be easier than ever this year. M:Unless the computer breaks down. Q:What does the woman imply about this year's registration? 35.W:Do you have to work overtime on your job? M:Every so often I do Q:What does the man say about working extra hours?
x

4.3 PART C

36-41 Before we get to the student center,I'd like to show you Brrrnely hall,one of the ve co-ed undergraduate residence halls of the university.Just next to it is Barmley dining-hall,Since there are only a few small kitchens in the dorms,most students buy meal contracts,whick entitle them to twenty meals a week at any of the cafeterias.With the exception of Sunday evenings and cacation breaks,dining-halls like Barnley are open every day for breakfast, lunch and dinner.Students simply present their meal tickets at the door and go through the line,helping themselves to as much food as they want.While dietitians work hard to o er students a nutritious and well-balanced diet,many of your young men and women claim that Barnley food,like most dorn food,leaves much to be desired,However,there are certain times,expecially around Thankgiving and Christmas,when the dining-hall sta goes all out preparing seasonal specialties,together with steak and seafood,These dinners are always exceptional and well attended.But regardless of what cooking,Barmley is a good place to get together with friends and to meet new people,as is the student center which is coming up on our right. 36.Why do most students buy meal contracts? 37.Who usually eats in Brmley dining-hall? 38.When are meals not served in Brmley dining-hall? 39.What does a student need in order to enter the dining-hall? 40.What do students who eat in Barmley dining-hall usually complain about? 41.When do students most enjoy eating at Barmley dining-hall? 42-46 W:Wake up,Erik,time to rise and shine.

26

\ho 91 d 1 nj^i`kp

M:Ha,oh,hi,Jane.I must have fallen asleep while I was reading. W:You and everyone else.It looks more like a campground than a library. M:Well,the dorm's too noisy to study in,and I guess this place is too quiet. W:Have you had any luck nding a topic for your paper? M:No,Prof Grant told us to write about anything in cultural anthropology.For once I wish she had not given us so much of a choice. W:Well,why not write about the ancient civillizations of Mexico.You seem to be inteested in that part of the world. M:I am,but there is too much material to cover.I'll be writing forever,and Grant only wants ve to seven pages. W:So then limit it to one region of Mexico,Say the Uka town.You've been there and you said it's got lots of interesting relics. M:That's not a bad idea.I brought many books and things back with the last summer,that would be great resource material,now if I can only remember where I put them. 42.What was the man doing whenthe woman approached him? 43.Why has the woman come to talk to the man? 44.What seems to be the man's problem? 45.What is known about Prof.Grant? 46.Why doesn't the man want to write about the ancient civillizations of Mexico? 47-50 Good water-color brushes cost a fortune,especially the large sizes,which are the most useful.Of all the things you need for painting in water color,brushes are the most important.So you should skimp on everything else if necessary,but buy the best brushes you can a ord,Water-color brushes are made from red sable,the best brush by far,Oxhair,including sabeline camel hair,squirrel hair and other animal hairs as wll as mylon.Some brushes lack rigidity.I would suggest you avoid any brush labelled camel or squrrill,unless you like to paint with a mop.They are oppy,The red sable is the best,but if you can't a ord one,the sabeline or ox-hair brushes are quite well.Nylon brushes appeal to many people because they are inexpensive and they produce acceptalbe e ect on paper, 47.What is the major subject of the talk? 48.According to the speaker,which brush is considered the best? 49.Why do some people prefer nylon brushes? 50.With which aspect of brushes is the speaker primarily concerned?

4RX 91 B 5 VN8L<PZ
1991 5*

"0
x

5.1 PART A

1.Emily and Joe are medicalstudents,aren't they? 2.The conductor of the symphony orchestra was terri c. 3.Ned wasn't invited to the party. 4.I have the store deliver the packages to my house. 5.There is no sport I enjoy more than swimming. 6.You'll nd her number listed in the telephone book. 7.Too bad you didn't make a hotel reservation. 8.The editor should have completed their work earlier. 9.The dormitory laundry service gives out clean sheets each week,doesn't it? 10.If I'd remembered to check the mailbox,I might have found your note. 11.Irene's grades enabled her to earn a scholarship. 12.John doesn't think they've left the o ce yet. 13.No sooner had I started to type my paper than the telephone rang. 14.The re was prevented from spreading because of Sandra's quick action. 15.I went for a walk to wake myself up. 16.Dno't you think it would be wise to review how much we've spent to date? 17.The committee was selected by the college faculty. 18.Standing next to Paul is the dean of admissions. 19.The door banged shut. 20.Peter's job search was unsuccessful.
x

5.2 PART B

21.M:Do you think Martha will get there in time? W:No way. Q:What does the woman mean? 22.W:Are you sure you've corrected all the typing errors in this paper? 27

28

\lo 91 d 5 nj^i`kp

M:Perhaps I'd better read it through again. Q:What is the man going to do? 23.M:Look,I'm sorry to bother you about this,but could you turn that music down? W:Sorry,I didn't realize you could hear it. Q:What will the woman probably do? 24.W:Janet insists she's coming to my graduation. M:But she has to work that week,doesn't she? Q:What does the man imply about Janer? 25.M:Do you think we should urge Bob to study Spanish? W:We'll have to leave that decision up to him. Q:What does the woman mean? 26.W:I ought to call Joan and tell her about the meeting this afternoon. M:Why bother?you'll see her at lunch. Q:What does the mansuggest the woman do? 27.M:What had they decided to call the stadium? W:No one has come up with a good name. Q:What does the woman say about the stadium? 28.W:Gary says he's planing to take an extra course this term. M:He's got to be out of his mind. Q:What does the man think of Gary's plan? 29.M:Have you ever seen the movie"Ute on the Bounty"? M:Once,I think. Q:What does the woman mean? 30.M:I was shocked when I heard you'd nished your research project a whoel month early. W:How I managed to do it is still a mystery to me. Q:What does the woman mean? 31.W:Oh,a new coat! M:My old one was looking a little to worse for wear. Q:What does the man mean? 32.W:Didn't Maria go shopping with you yesterday? M:Even if she hadn't had a lot of studying,she would have preferred staying home to going shopping.

5.3 PART C

29

Q:What does the man imply about Maria? 33.W:Come seimming with me? M:Sorry,but I'm up to meck in work. Q:What's the man's problem? 34.W:This course wasn't supposed to be hard. M:But it sure turned out to be,didn't it? Q:What does the woman think about the course? 35.M:Have we received responses from everybody we invited? W:Ahha,only Tina can't come. Q:What does the woman mean?
x

5.3 PART C

36-40 M:Hi,Sherril W:Hello,Fill,how are you? M:I'm ne,where are you headed? W:Oh,I'm on my way home from work. M:I didn't know you have a job. W:Ya.I work part-time at the supermarket. M:What do you do there? W:I work in the produce section,trimming and wrapping fresh fruit and vegetables.I also stock shelves.Sometimes when it really gets busy,I work at the check-out counter.Have you got a job,Fill? M:Ya,I do vard work for people.You know,cutting grass,raking leaces,falling weeds,things like that. W:I'd like doing that.It must benice to work outdoors. M:Sometimes it is,except when it rains,snows or gets too hot or too cold. W:I guess every job has its drawbacks,There are times when I get pretty tired of carrying things around at my job.But a job is a job.Got to earn money for school. M:Me too,tuition sure is high,isn't it?Well,I'd better get going I have to plant some trees for my neighbors this afternoon. W:Well,don't work too hard,holding down a job,going to class,studying,sometimes can become too much for one person.Take it easy

30

\lo 91 d 5 nj^i`kp

M:You too,ot was great seeing you,Sherrill. 36.Where was Sherrill ging when Fill met her? 37.Where does Sherril do at her supermarket job? 38.Why does Sherrill do at her supermarket job? 39,What does Fill not like about his job? 40.What is Fill going to do next? 41-45 Mark Twain,who wrote the story we are going to read,travelled quite a lot, ofen because circumstances,usually nancial circumctances forced him to.He was born in Floridan Missouri in 1835,and moved to Hannibal,Missouri with his family when he was about four years old.Most people think he was born in Hannibal,but that isn't true.After his father died when he was about twelve,Twain worked in Hannibal for a while and then left so he coule earn mored money.He worked for a while as a typesetter on various newspapers,and then got a job as a river pilot on the Mississsippi.Twain loved this job,and many of his books show it.The river job didn't last however,because of the outbreak of the civil war.Twain was in the federate army for just two weeks,and then he and his whole company went west,to get away from the war and the army.In Nevada in Californis,Twain prosperted for sister and gold without much luck,but did succeed as a writer,Once that happened,Twain travelled around the country giving lectures and earning enough money to go to Europe.Twain didn't travel much the last ten years of his life,and he didn't pyublish much either,Somehow his travel,ever when forced,inspired his writings.Lke many other popyular writers,Twain derived much of the materials for his writing from.the wealth and diversity of his own experisence. 41.The speaker focuses on which aspect of Mrk Twain's life? 42.Where do most people think Twain was born? 43.What job did Teain especially love? 44.Why did Twain go west? 45.What connection does the lecture suggest between Twain's travels and his writings? 45-50 M:Did you watch the TV special about barnacies last night? W:No,I missed it.What did you learn about them? M:That they glue themselves to rocks in the ocean,shortly aftwr birth.They also stay in the same place forever. W:Right,have yu ever tried removing one of these things?It would be easier to chip the rock awaythan to get the barnacles o .

5.3 PART C

31

M:Exactly,and that's why scientists are trying to gure out what the barnnacles glue is made of.It's considered one of the strongest adhesives in nature. W:And it would be have the advantage of being able to work on wet suriaces too. M:Precisely,and because it's natural protein,it probably wouldn't be harmful to people like so many synthetic glue in use today. W:Think of all the ways doctors and dentists could use such a glue,mending broken bones,fastering false teeth. M:There could be countless uses.Scientists hope to learn soon exactly how the glue is made, so that people can make use of it. 46.What prompted the conversation? 47.What is the main topic of the conversation? 48.What did the man learn about barnacles? 49.According to the conversation,waht are scientists trying to discover? 50.According to the conversation,in what eld might the scientists nd this esperially helpful?

32

\lo 91 d 5 nj^i`kp

4>X 91 B 8 VN8L<PZ
91 8*

"0
x

6.1 PART A

1.The bank granted her a loan to couver her school expensies. 2.Bill,thanks for picking up the books for me. 3.She said the demonstration begins at 8 o'clock,didn't she? 4.Linda took night classes for ten years and eventually got her degree. 5.Automobile poliution is a far greater porblem than industrial waste. 6.I.m used to getting up early. 7.Mary said she wanted to live by herself. 8.Red lettering on signs is much more noticable than blue. 9.He stood up for what he thought was right. 10.I'd like to skip the meeting,but I can't just not go. 11.I couldn't get David to tell where he's going on vacation. 12.The radio is too ioud only for Ben. 13.Let's leave the ear at home and walk to town for a change. 14.Kevin raced through the station,afraid he'd miss the train. 15.If we hadn't made the wrong turn,we'd have been on time. 16.Better than half the class was absent. 17.People from the television station have requested the viewers to send in their suggestions. 18.Although John would never believe it,I did try to return his telephone call. 19.Lunch wasn't at all appealling today. 20.Robert tends to keep his opinions to himself.
x

6.2 PART B

21.W:These boxes are too heavy for me to move. M:Here,I'll give yu a hand with them. Q:What does the man say about the boxes? 22.M:Paul says he doesn't like television. 33

34

\ao 91 d 8 nj^i`kp

W:Yes,but he seems to spend a lot of time watching it,doesn't he? Q:What does the women think about Paul? 23.W:You seem well prepared for this Spanish midterm test,I'm impressed. M:I'd better be prepared.I spent the last 48 hours cramming for it. Q:What has the man been doing? 24.W:The seminar originally scheduled for today has been cancelled. M:Too bad,but it'll still be on for next week,isn't it? Q:What does the man assume about the seminar? 25.M:Judy's taking sixteen credits and working in a computer center too. W:How she manages to handle all that I'll never understand. Q:What does the woman mean? 26.W:Where do yu want me to drop you o . M:Right here is ne. Q:What will the man probalby do? 27.W:Is that math course really as hard as everybody says? M:Worse, believe it or not. Q:What does the man say about the course? 28.W:We're supposed to pick up the tickets to sell this afternoon. M:Oh,so they have been printed. Q:What had the man thought about the tickets? 29.M:Do you know anyone who has a spare bookshell? W:Hace you checked with Marsha? Q:What does the woman suggest that the man do? 30.W:Since it's rainning so hard,let's go in and see the new exhibit. M:That's a good idea.Mary Gassette is one of my favorite painters. Q:Where does this conversation probably take place? 31.W:I heard only one person got a perfect grade on the term project.I'm sure it wasn't me. M:But it was. Q:What does the man mean? 32.W:I don't think our classmate should confront ProfSims with these issues. M:I know.I'm going to try to talk them out of it. Q:What will the man try to do?

6.3 PART C

35

33.M:The photographer's approach is very creative. W:Isn't is,though. Q:What does the woman think about the photographer? 34.M:I'm so confused by my notes from professor Johnson's lecutures. W:How about reviewing them now over a cup of co ee? Q:What does the woman suggest they do about the notes? 35.M:Janet,you're redecorating your apartment,aren't you?How's it coming along? W:I'm just taking it one step at a time. Q:What does the woman imply?
x

6.3 PART C

36-41 Good evening ,I'm Margerat Sinclare,Chairwoman of the National Musical Excellence Society,and I'd like to welcome yu to Carnegie Hall,one of the world most famous performance auditoriums.The orchestra will perform in a few minutes,But before it does,I'd like to tell you about the hall's recent trvovation.In 1990,Carmegie Hall was scheduled to be torn down,It was in terrible shape,the roof was collapsing,the brick wallswere crumpling,and there was no money for renovations.Fortunately,famed violinist lsaac Stone heard about the hall' s fate and organized a committee to save CarnegieHall.He and others managed to raise 50 million dollars for repair,Over 250 laborers and artisans have benen working on and o since the monty was raised to restore the hall's grandeur.Today the hall looks better than ever.The orchestra will now play selections rom the works of Leonar Bernstein.After the concer,the reception,will be held in the mian lobby.Thank you for your support.Without further ado,I now present the Cleveland Symphony Orchestra. 36.Where is the speech taking place? 37.What kind of group will be performing? 38.What was Isaac Stone's purpose in organising the committee? 39.Approximately.how many laborers and artisans have worked on the project? 40.Where will the reception be held? 41-45 M:What was professor Calhourn saying about the lecture senes on alternative forms of energy wehn I walked in late?I had trouble with my car again. W:The rst jecture's tonight.All second and third year students are invited to ba et dinner tonight before it starts.Then at 6:30,the speaker will begin her talk on solar energy.

36

\ao 91 d 8 nj^i`kp

M:Can nonengineering students attend all four weeks of the series? W:No,they can only go to this week's lecutre During the next three weeks,the admission will be restricted to engineering students. M:Oh,then I'm going to try to make it tonight.Can I give you a ride over there? W:No,but thanks anyway.wait a minute,didn't you say that you are having car trouble lately.Let me give you a ride. 41.Why was the man late for class? 42.What's the topic of tonight's lecture? 43.For how many weeks is the lecutre series given? 44.How many weeks of the lecture series can non-engineering students attend? 45.What does the woman suggest the man do? 46-50 Do you have trouble sleeping at night?Then maybe this is for you.When you worry aoubt neeting sleep and twisting around,trying to nd a comfortable position,you're probably only making matters worse.What happens is that your heart rate actually increase,making it more di cult to relax.You may also have some bad habits that contribut to the problem.Do you rest frequently during the day?Do you get virtually no exercise,or do you exercise strenuously late in the day?Do you think about sleep a lot or sloop late on weekends?Any of these facts might be leading to your insomnia by disrupting your body's natural rhythm.What you should do then on those sleepless nights?Don't bother with slooping pills,they can actually cause worse insomnia later.The best thing to do is to drink milk or eat cheese or tuna sh.They are all rich in the amano acid that helps produce in the brain a neural transimitter that induces sleep.This neural transmitter will help yuou relax.and you'll be on the qay to get a good night's sleep.Until tomorrow's brodacast,this has been another series"Hint for good health"by Doctor Handison. 46.What s the purpose of the talk? 47.Accourding to the speaker,what happens when you turn and twist to get comfortable? 48.According to the speaker,what sometimes cause people to have trouble sleeping? 49.What does the speaker say about sleeping pills? 50.Where is the talk most probably being given?

4DX 91 B 10 VN8L<PZ
91 10 *

"0
x

7.1 PART A

1.The radio works well only in clear weather. 2.Actually,I don't think Jane's busy now. 3.Gary has to face the facts. 4.If I'd known the bank would be closed today.I would have gone there yesterday. 5.In computer programming,Susan is head and shoulders above the rest of us. 6.She is a rst-year law student. 7.For the next half hour,we will work out in the gym. 8.When will these picutres be ready? 9.Jane was taught to drive by her brother. 10.Philip's never been seen wear a tie. 11.I must have left my o ce keys at home because I can't nd them anywhere. 12.Wasn't it the best meal you ever ate! 13.We won't start a new lesson before next week. 14.You took the bus to get here right? 15.I am afraid that Alan is not up to such a di cult task. 16.My cousin,who is a geography teacher,helped us plan out trip. 17.Dian was too excited to sit quietly and kept pacing the oor. 18.Robert wants to take up drawing. 19.he got bored playing cards last night so he left early. 20.May I have a package of safety pins please?
x

7.2 PART B

21.W:Shouldn't someone pick up the clothes form the cleaner's? M:Don't look at me. Q:What does the man mean? 22.M:George is going to work in New York for the summer. 37

38

\eo 91 d 10 nj^i`kp

W:Can he do that and go to sumer school too? Q:What does the woman imply about George? 23.M:Have you seen Jim around?we are supposed to play tennis. W:Well,his racket is here on the table Q:What does the woman imply? 24.M:I can't remember the due date for our nal paper. W:I think it is the last day of class.but professor Merdoks said not ot wait unitl the last minute to hand it in. Q:What did professor Merdoks suggest to the studenst? 25.W:I hear that your sitster Catherine was a great student. M:She is,She hasn't graduated yet. Q:What does the man say about Catherine? 26.W:Has Sandy ever missed coming to one of our par time? M:Only once or twice at the most. Q:What does the man mean? 27.W:The art museum shop has all sorts of prints that're inexpensive and colorful. M:Especially if you like modern art. Q:What does the man say about the shop? 28.W:Canyou listen for my telephone? M:Sure,I am planning to be here all day. Q:What does the woman want the man to do? 29.M:I don't know whether I should time my speech now or keep on revising my notes,What do you think? W:I'll carry on with what you are doing. Q:What does the woman suggest that the man do? 30.W:Would you like to come mountain climbing with us? M:That's the last thing in the world I'd ever want to do. Q:What does the man mean? 31.M:Kevin's been in such a bad mood lately. W:Hasn't he? Q:What does the woman mean? 32.W:I hear you got lost on your way to the meeting at the hotel

7.3 PART C

39

M:I don't know how I did it.I have been there a million times Q:What does the man mean? 33.W:Anthony,did you x the leaky faucet? M:No,I had a plumber do it. Q:What happened to the faucet 34.M:I think I am going to give up playing table tennis.I lost again today. W:Just because you lost,is that any reason to quit? Q:What does the woman suggest the man do? 35.W:Guess what,I just nominated you for this year's treasurer. M:Oh,no,Sallu,you didn't Q:What does the man imply?
x

7.3 PART C

36-39 Hello,I'm Judy Handison.Before we start our rst lab, I'd like to tell you a little bit about the workbook you'll be using.The rst thing I'd like to point out is that the workbook contains a very large amount of material,far more than you could ever handle in a single semester.What you're supposed to do is to choose the experiments and ctivities you want to do,within a certain framework of course,Part of my job is to help you make your choices.Next,I'd like to mention that in each workbook chapter,there are usual two subsections,The rst if called experiment,and the second is called activity.In the experiment section,the workbook is full of inftructions for all the experiments including alternative procedures.You may use the procedure your wish on the basis of available equipment of personal preference.In the activity section,you will nd suggestions for many experiments ,exercises and projects that you can do on your own time.You'll see that there are usually no detailed instructions for the activities.You're supposed to do them yur own way.Okay,let's turn to chapter one now. 36.What is the instructor describing? 37.What is the main idea of the talk? 38.How are the activities di erent from the experiments? 39.When would this talk be given? 40-45 M:You are my campaign manager,July,how do I convince members of the student body that I am the best candidate for school president?

40

\eo 91 d 10 nj^i`kp

W:It won't be easy M:Thanks a lot. W:I'm just kidding,Bob,Actually I think once we show everyone how well you did as Junior class treasurer,you will win easily. M:How do we do that?Meet with all three thousand people who are members of the student body? W:Come on,what I'm thinking of rst is hanging campaign posters in all the hallways,for most students are sure to see them. M:That sounds good.but everyone puts up posters.What can we do that's di erent? W:The campus radio station is willing to let you have ve minutes tommorrow morning at seven o'clock to outline your plans for the year.Lots of students will be listening then. M:That is a great idea. W:I've also arranged for you to give a speech during dinner tomorrow.At least ve hundrend of the students will be there.You will answer questions after you nish speaking. M:That means I'd better come up with a speech pretty quickly.How about if I write it tonight and show it ot you after chemistry class tomorrow? W:Fine,I'll see you after class. M:Thanks for all your help. 40.What student body o ce does the man want to be elected? 41.Who is thw woman? 42.Where will they put the posters? 43.When is the radio broadcast scheduled? 44.What will the man do tonight? 45.What will they do after chemistry class? 46-50 Up to now,we've fcoused on the positive impact of the automobile on society n the United States.Today we'll look at some of the negative e ects.A number of sociologists blame the automobile for the decline of the downtowen areas of major cities.In the 1950's amd 1960's ,the automobile made it possible to work in the city and yet live in the suburbs many miles away.shopping patterns changed.Instead of patronizing downtown stores,people in the suburbs went to large shopping malls built in the open area outside the cities,where everything is cheaper.Marchants in the city failed:stores closed.Downtown shopping areas became deserted.In recent years,there has been a rebirth for a number of downtown areas,as suburbanites,especially young married couples,have moved into the city to avoid road clogged with evening.Now,let's look at some slides

7.3 PART C

41

that illustrate these trends.By the way,tomorrow I'll explain the e ect of all these automobiles on the environment,for example,What happens to the atmosphere as a result of burning gasoline and depletion of natural resources like fossil fuels? 46.What did the speaker probably talk about previously? 47.How did theautomobile a ect the work force in the 1950's and 1960's? 48.What problem did city merchants face? 49.According to the speaker,Why are some people moving back to the city? 50.What will be included in tomorrow's lecture?

42

\eo 91 d 10 nj^i`kp

42X 91 B 1 VN8L<PZ
91 1*

"0
x

8.1 PART A

1.Jack introduced us to his parents. 2.I nished the assignment a week ago. 3.We didm't think you would mind. 4.Do go ahead with your dessert. 5.He was extremely angry but didn't let it show. 6.Aren't your clever to gure that out! 7.Ann rides her bike to work to save money on bus fare. 8.My marks are higher than I thought they's be. 9.Alex will call before he comes,won't he? 10.The club president announced that each member has a quota of ten tickets to sell for the talent show. 11.We should include these gures in the report,I think. 12.Now that I nally turned in my thesis,I plan to take it easy for a while. 13.I don't think I heard her name mentioned before. 14.No one came to the school play. 15.If only you told me sooner. 16.I've got to get going on my research project. 17.The actors received a standing ovation for their periormance. 18.A friend bought Doris's painting. 19.Everyone thinks Carlo is a very outgoing fellow. 20.I was late coming back from lunch today.
x

8.2 PART B

21.W:Mathew wants to know if he can go with us to the party. M:That's old,this morning he said he wanted to go by himself. Q:What does the man mean? 43

44

\ o 91 d 1 nj^i`kp

22.M:I thought I'd give Pam a call now. W:Isn't it a little late at night? Q:What does the woman imply? 23.M:Would it be okay if I use this machine? W:I don't see any reason why not. Q:What does the woman mean? 24.M:Hi,Ammy,You're looking better,it's nice to have you back. W:Thanks,I just hope I can catch up on all the work I've missed. Q:What does Ammy mean? 25.M:Bruce said he would meet us. W:Well,he's really taking his time getting here. Q:What can be assumed about Bruce? 26.W:Why didn't you make an appointment to see Doctor Handison last week when you rst twisted your foot? M:The injury didn't seem very serious then,I decided to come today because my foot still hurts when I put my weight on it. Q:Why didn't the man,see Doctor Handison eather? 27.M:Do you always look over your own papers for typing mistakes. W:I usually have to do it myself,but I'd rather have someone wlse proofread them. Q:What would the woman prefer to do? 28.M:I stayed up all night studying for the test. W:Again,How you get by with so little rest is a mystery for me. Q:What does the woman mean? 29.W:Let's all go to the baseball game tonight. M:Why waste our money?Teh team's going to lose again. Q:What does the man mean? 30.M:Sarah's been home sick for a week. W:Why don't we drop in on her and nd out how she is doing? Q:What does thw woman mean? 31.W:I need to have a new dress made. M:Doesn't Nancy sew? Q:What does the man imply?

8.3 PART C

45

32.M:How do you like the way I've arranged my furniture in my living room? W:Fine,but I think the walls could use a few paintings. Q:What does the woman suggest that the amn do? 33.M:Would yu like a copy of this article? W:Thanks,if it's not too much trouble. Q:What does the woman mean? 34.W:Let's get this desk in order. M:Yes,it's about time we cleared it o . Q:What can be inferred form the desk? 35.W:We got the tape recorder repaired last week. M:Oh,so it couldbe repaired. Q:What had the man assumed?
x

8.3 PART C

36-40 W:Have you ever lookedreally closely at the snow ake? M:Sure,but they usually melt too fast for me to get a close look.Why do you ask? W:I'm just curious.I was reading an article about the formation of snow akes,and I realized that I had never paid much attention to them before. M:Wll there is a big variety,isn't there? W:Yea,but they all have one of the three basic forms hexagona!columns,this hexagonal plates and the balnching star-shaped form. M:I wonder why are forms di erent,maybe because ice starts to form on dus particles with di erent shape. W:Well I thouht it might have something to do with the water saturation of the air.but we're both wrong,The author of this articale did extensive research and concluded that the shapes of snow crystals largely controlied by the temperature of the air.For example,the featherly starshaped snow ake that everyone thinks is typical occurs only at a speci c temparature. N:Doesn't the relative humdity has anything to do with the shapes? W:Apparently not.The a ect of super saturation is simply to alter the growth rate.The greater the saturation,the faster the snow akes form. M:Wh,next time when it snows,I'll make a point fo taking a close look.

46

\ o 91 d 1 nj^i`kp

36.What are the people discussing? 37.Why has the woman brought up the subject snow ake? 38.How many basic types of snow akes are there? 39.What determined the shape of the snow akes? 40.What does the man say that he is going to do? 41-45 Good evening,welcome to the rst meetiong of ourspring cycling season.It's a pleasure to see so many new faces here.I certainly hope that most of you will soon feel right:a home with our group and bibybling will become a regular hobby for you and a part of your physical tness routine.Some of you may not reallize that the state of New Jersey o ers ample opportunities for bicyclists of all abilities.We have rolling country side,miles of beaches,lovely hills and valeys and thousands of miles of little used roads.Beginners will nd the shore and area south of Princeton suitable for their needs while expert riders will discover that the steep trails of the region around high point o er a challenge to their skill and stamina.In addition,New Jersay is rich in places of historical interest.The state abounds in colonial archeracture,battle elds of the revolution and other historic sites important in the early history of this country,most carefully preserved or have been meticulously restored.We organized tours to a variety of places nearly every weekend. Next Saturday tour will be a relatively easy one,from RectersUniversity to the gate of National Park.Interested cyclists should meet at eight a.m. in the parking lot at the corner of Hamilton Street and Coliege Acenue in New Branderswick.The minimum time of this tour is about six hours.We'll use a buddy system throughour the ride.Each nes rider should team up with a more experienced rider.If the puddles look after each other,we can be sure that no one will be left behind on the back road.You can pick up a map of Sautrday's tour at the information table before you leave this evening. 41.What is the purpose of this talk? 42.According to the speaker,waht makes New Jersey a good place to bicycle? 43.Whay does the speaker mention the historical sites? 44.What will some of the listeners probably do on Saturday? 45.What is the purpose of the buddy system? 46-50 W:Hi,Tom. M:Judy,I haven't seen you in weeks.Where have you been? W:In Florida. M:What,vacationing while the rest of us have been studying on campus in the Feburary cold.

8.3 PART C

47

W:Not exactly,I spent most of my time underwater. M:I don't understand. W:I was on a special eld trip.I went with my marine biology class. M:So you went Souba diving,What were you looking for,Sunken treasure? W:You might say so,the sea is full of treasures,all kinds of strange fascination organisms.Our class concentrated on studying plankton. M:I thoutht planktons are too small to be seen. W:That's a common misconception.The term Plankton couvers a wide variety of freely oating plants and animals from microscopoc one-celled organism to large ones such as the common jelly sh. M:Jelly sh maybe large enough to be seen,but they are transparent,aren't they? W:Yes,most planktons have transparent tissues as a protective camou age,It makes them practically invisible to predators. M:But not invisible to your bioloby class,I hope. W:By concentrating,Iwas able to see the outlines of lots of di erent planktonis plants and animals.In fact,our professor even took photographs of Greeber Quadata,which are small oceanic snails. M:How could the snails show up in the photograghs if they are transparent. W:We squirted GreeberQuadata with a harmless green day.Since particles of the day stuck to their tissues,the snails appeared in green outline in the photograph. M:That sound like a interesting trip.But I think if I'd been in Florida in Feburay,I'd much rather spend my time just swimming and lying in the sun. 46.How did Judy spend most of her time in Folride? 47.What was Tom doing in Feburary? 48.What kind of class went on the eld trip? 49.Acording to Judy,What did her professor photograph? 50.Why is it hard to see most planktons?

48

\ o 91 d 1 nj^i`kp

4:X 92 B 5 VN8L<PZ
92 5*

"0
x

9.1 PART A

1.We tried calling Ken,thinking he'd be home. 2.She doesn't know we nished the experiment. 3.Didn't she handle her presentation well 4.I don't know Maxine although we went to the same university. 5.At the end of the streetis a bookstore where you can buy used books. 6.If he'd only asked me for help I'd have done my best for him. 7.The shoe polish doesn't match the shoes. 8.You'll beed ti get another chair from the living room. 9.I had a piece of Carla's birthday cake although I usually don't like dessert. 10.Here.take this pill,don't chew it. 11.I can't nish what I'm working on until I have your research result. 12.Never have I heard such an unconvincing explanation 13.He used to walk to work when he lived in town. 14.I saw the pitcher of water about to full,but I couldn't reach it in time. 15.It seems as if the bakety has been clsed for months,even though it's only been two weeks. 16.I would have gone to the movie if I'd known what was playing. 17.I can't see the need for a car. 18.Roy asked the dean to waive certain requirements. 19.I turned o the lights,all ready for bed. 20.It's unnecessary to take tests in a course you audit.
x

9.2 PART B

21.W:Congratulations!I understand you got a job,when do you start to work? M:You must be thinking of someone else,I'm still waiting to hear. Q:What does the man mean? 22.M:Prof.Johnson seems so serious. 49

50

\_o 92 d 5 nj^i`kp

W:But his lectures are quite humourous,aren't they? Q:What does the woman thinking about Prof Johnson? 23.W:Gary,why is it that whenever I open my mailbox lately,I pull out letters addressed to you? M:Because until I nd a new apartment,I'm having the post o ce foreard my mail to yur place. Q:What's happening to Gary's letters? 24.M: They said the train won't arrive u until nine. W:Well,what that boils down to is yet another delay in our schedule. Q:What does the woman mean? 25.W:If Prof.Thomas's willing to give us a three-day extension to nish the project,maybe he'll give us a few more days. M:Let's not push our luck,Mary.OK? Q:What does the man mean? 26.W.How do you like the play? M:Oh,I've seen worse. Q:What does the man mean? 27.M:What's happening with the new library building? W:The work crew is just nishing it up. Q:What does the woman say about the library? 28.M:I'm not quite sure how to put this,but about that calculator you let me use,I dropped it ,and now the on-button doesn't light up. W:Oh,that's okay,it hasn't been working right for some time now Q:What was the man's problem? 29.M:The city is going to tear down those old houses and put up a new shopping center. W:Anothere shopping center,that's nothing new. Q:What does the woman mean? 30.W:Do you know anyone who would do some typing on short notice. M:How big is the job? Q:What does the man want to know? 31.M:I thunk the whole class is going on the eld trip next Friday. W:I'm not so sure,not everyone has paid the transportation fee. Q:What does the woman imply? 32.W:I'm taking John to that fancy new restaurant tonight.

9.3 PART C

51

M:You can't go like that,You'd better change. Q:What does the man advise? 33.M:How did you talk with Herry go? W:It never really got o the ground. Q:What does the woman imply? 34.M:Hellen,don't forget to put your umbrella in the car. W:Oh,it's just overcast. Q:What does Hellen mean? 35.M:Have you seen this postcard from Rom.He's in Florida, W:Oh,so he was able to get time o from work. Q:What had the woman assumed about Rom?
x

9.3 PART C

36-40 W:I'm thinking about transfering out of state college into another school in the spring. M:After a year and a half?How come?I thought you liked it here. W:I do.But our commercial art department doesn't give Bachelor's degrees,only associate.I want a bachelor's. M:So where do you want to go? W:I woundn't mind going to Westwater University.It has an excellent reputation for commercial art,but I have a feeling it's very selective. M:But you've gotten grades in the three semesters you've been in the state college,haven't you? W:Yha,mostly As and Bs. M:So what are you worried about,just ask your professor to write letters of recommendation for you,and you'll be set. 36.What is the woman planning to do? 37.What eld is the woman interested in? 38.How long has the woman been studying in state college? 39.What concern does the woman have about Westwater University? 40.What does the man suggest that the woman do? 41-45

52

\_o 92 d 5 nj^i`kp

Several of you have spoken to me about the possibility of make-up labs so that you can complete laboratory work you've missed for one reason or another during the course of the semester.Since this is our last week of class,any such make-up work would have to be done this week.I'm sure you are aware that on-third of your nal grade is based on your lab work,So any missing work will vertainly a ect your nal grade.I've decided to open the lab for three evenings this week.The lab will be open tonight,tomorrow night and Wednesday night from sisx to ten.I'll be there to assist you with materials and equipment.Since your lab notebooks for the semester are due on Thursday,the rst three nights of this week will be your last opportunity to make up incomplete work. 41.Who is th speaker? 42.What is the speaket's main point? 43.At what point in the semester is the talk given? 44.How much of the nal grade is based on lab work? 45.On what day of the week is this talk probably given? 46-50 M:Welcome back.I didn't see you in calculus calss on Friday. W:I wasn't here on Friday.My geology class on went on a eld trip to look at some of the di erent rock formations here in California.Our last stop was Death Walley,In fact,We camped there last night. M:Death Valley?but that's desert.Wasn't it much too hot for camping? W:No,not really.It gets hot during the day,but it cools o very quickly at night.There is no moisture in the air to hold the heat after the sun goes down.If it were humid,it would stay hot all night. M:Well,did you enjogy the trip? W:It was fascinating.The destert is nicer than you might think.Since there is so little vegetation growing,it's very easy to see di erent rock layers,folds and such,and the natural colors of rocks are specacular,There is even one part that's called"Artists"Palette",where impurites in the rocks have made brilliant colors,blues,pinks and green,You really should go see it sometime. 46.What do the two speakers have in common? 47.What was the woman doing on Friday? 48.What had the man assumed abut the death valley? 49.Why does the woman think that the desert is a good place to visit? 50.According to the woman,why is "Artists Palette" particularly interesting?

4HX 92 B 8 VN8L<PZ
92 8*

"0
x

10.1 PART A

1.She learned to use the computer on her own. 2.The art history class is one I never miss. 3.Carolyn's favorite sport used to be golf,but now it's rennis. 4.You returned the book this morning,right? 5.She was about to go to sleep when the phone rang. 6.The game was tied until John scored the winning point. 7.To act as rudely as Mark did is unforgivable. 8.I just sat there and ate,too hungry to say a word. 9.From now on,I refuse to put up with your silliness. 10.They should look for that magazine at the newsstand. 11.The owers in the window really brighten my spirits. 12.If I'd brought a second pen,I wouldn't have to borrow yours. 13.Don't you think Mike will succeed in the organic chemistry course? 14.The school bus drivers are handing our schedules. 15.If he had just paid more attention this morning. 16.We reached home while it was still light outside. 17.To think she's been here six weeks already. 18.You will be able to meet these people by and by. 19.It took Mary twice as long as usual to drive to work. 20.I want to see the view,too.
x

10.2 PART B

21.M:I thought this book was going to be really boring. W:But it's actually quite fasinating,isn't it? Q:What does the woman say about the book? 22.M:This is hopeless.These gures still don't add up right.Let's do the calculations over again. 53

54

\go 92 d 8 nj^i`kp

W:Yes,but why not do them tomorrow.It's very late now. Q:What does the woman mean? 23.W:I think Tom left his umbrella on the bus. M:Why he's so irresponsible is beyond me. Q:What does the woman say about Tom? 24.M:Professor Joans,when can we meet to discuss my grade? W:Would four o'clock be all right? Q:What does the professor mean? 25.M:My chemistry project is in trouble because my partner and I had totally di erent ideas about how to proceed. W:You should try to meet each other half way. Q:What does the woman suggest that the man do? 26.M:I have to look for a room to rent. W:How about trying Caders?Tey usually have a long list of places. Q:Where will the man probably go? 27.W:Did you and Amanda really meet exactly, ve minutes to two in front of the theatre? M:Yes,we were both there on the dot. Q:What does the man mean? 28.W:Should we ask Paul to pick up our clothes at the dry cleaners? M:Why not,it's right on his way home. Q:What does the man mean? 29.W:Well,you cleaned out the refrigerator this time.You must have beenreally hungry. M:All did was x a snack.There weren't many leftovers from yesterday. 30.M:Catherine is taking a microbiology course this semester. W:Should she be taking that this year? Q:What does the woman imply that Catherine should be doing? 31.M:We just saw Joe when we were in New York. W:Really,what's new with him? Q:What does the woman want to know? 32.M:Hey,where did you nd that journal?I need it for my research too. W:Right here,but don't worry,I'll take it out on my card for both of us. Q:Where is this conversation probably taking place?

10.3 PART C

55

33.W:Eggs are supposed to be one of the most healthful foods. M:Next to potatoes. Q:What does the man mean? 34.W:Fill could you give me a hand with this le cabinet?It belongs against the wall,next to the bulletin board M:At your service. Q:What are the speakers doing? 35.M:Harry can sure play baseball. W:Can't he,though Q:What does the woman mean?
x

10.3 PART C

Good morning,students,the school bulletin lists this course as a required writing courrse r second-years students.This does nto mean the rst-year or third-year students may not take it.What it means is that those of you who have never learned how to write a research paper ought to be here,and those of you who already know how shouldleave immediately and sign up for another Enlish course.I hope to cover the entire process of writing a researel paper from selecting a topic to putting together the nal former and presentation.I'll be here to help you every step of the way.But it is your responsibilty to hand in each assignment or time.Your nal grade will be the grade yu receive on your paper. 36.Who was the speaker? 37.For whom is the course intended? 38.What does the speaker ask the students to do who have already learned to write a research paper? Question 39-42 M:Did you know that astronauts have made commercial products in space? W:No,I wasn't aware of that.What kind of products? M:Tiny plastic beads. W:Beads?Do you mean to tell me that astronauts have nothing better to do than make jewelry beads in space?I seems to me they could make more useful things out there. M:Oh,but these beads aren't for jewely.They can be used for many scienti c purposes,from conducting cancer research to calibrating microscopes. W:That sounds better,But why make such objects in space rather than on earth?

56

\go 92 d 8 nj^i`kp

M:Because earth gravitational pull a ects the beads.The beads produced on earth are distorted,not exactly round.The ones made in space are precise spheres. 39.What are the speakers mainly discussing? 40.According to the conversation,what is the likely use for thebeads? 41.According to the man,why did astronauts make the beads in space? 42.What material were the beads made of? Question 43-46 Today,I would like to begin by discussing early European settlement along one of our well-known rivers,the Hudson,which empties into the Atlantic to form New York bay.The Hudson river has a couple of interesting physical features that made it very attactive for settlement by the Europeans.The rst is that the river extends inland from the Atlantic Ocean for more than 150 miles with no waterfalls or rapids,Its surface is virtually at for that entire distance,with to obstacles.Secound,the whole 150-mile stretch is infuenced by tides from the Atlantic Ocean.Roughly every six hours the river reverses direction. woing north when the tide is rising and south toward the ocean when the tide is going down.Obviously there were no obstacles to prevent settles from moving farther upstream on the Hudson river,and this explains why the Dutch penetrated so far inland.They were the rst Europeans to settle in the Hudson valley.Of course,to go upsteeam,the Dutch settlers needed the right kind of boat,and so to navigate the reiver,they design a soon with only one mast but with two salls,one rigged in front of the mast and on behind,The mast was very tall,in many cases over 100 feet tall so that the large salls could catch winds blowing above the shore line hills.Hudson river sloops carried passengers and cargo.The cargo ranging form coal,lumber and hay to fruit,vegetables and livestock.Travelling onlly ten milles an hour in a good wind,the sloop was not too speedy by modern standards,but it was ideally suited to the Dutch settlement,and in fact when the steam boat eventually was introduced,it couldn't keep up with the sloop. 43.What attracted the Europeans to the Hudson river area? 44.What is the characteristic of the rst 150 miles inland on the Huadon river? 45.How do tides form the Atlantic Ocean in uence the Hudson river? 46.Accourding to the speaker,why did Hudson river sloops have tell masts? Question 47-50 M:Hello W:Hello,this is Susan Wilson.May I speak to Greg Robinson please? M:Hi,Susan,it's Greg,What can I do for you? W:Well,I'm calling about the theater group I belong to,the PRinceton Players,we are looking for more people to join us,expecially men and I thought you might be interested.

10.3 PART C

57

M:Oh,gosh,you know how much I love acting,but I'm taking some very hard courses,I might be able to learn my part,but I would hardly have time to come to tine rehearsals. W:Well actuallymwe rehearse only one night a week,thursdays from seven to ten,though we would have to put in extra time before performance. M:Only once a week you say,Well could you give me time to think it over? W:Sure,but look,why don't you come and watch our rehearsal next Thursday?I think you will like the way we work,When you see how much fun it is,you will join right away. M:Okay,I'll come to a rehearsal,but I can't promise more than that, W:Great.I'll give you a call on Wednesday to remind you Talk to you then,bye now. M:Bye Susan. 47.Whay does Suan think Greg might be interested in the theater group? 48.How often does the group rehearse? 49.Whay does Greg ask for time to think about whether be will join the group? 50.What does Suan expect Greg to do?

58

\go 92 d 8 nj^i`kp

4HTX 92 B 10 VN8L<PZ
92 10 *

"0
x

11.1 PART A

1.Switch o the light,will you? 2.We nished painting the furniture last month. 3.I can nd only one of my shoes. 4.Don't you just love this ice cream! 5.We had to wait for the bus in the rain. 6.The game will be in the gym,I will meet you there. 7.That name just doesn't ring a bell with me. 8.Jane looks exhausted,but she'd just slept ten hours. 9.They are expecting us at ve. 10.In no way do I blame you for what happened. 11.Professor Stone agreed to let us turn our papers in on Monday instead of Friday. 12.Bruce and Chuck are twins,can't you tell? 13.The concert seems to be over almost as soon as it started. 14.If I'd only remembered to lock the door. 15.We stopped at the snack bar for a bite to eat. 16.Weren't they good teaching assistants! 17.Lisa boutht some rug cleaner at the supermarket. 18.They missed not only the rst train but the second one as well. 19.Bonnie really has a way with words. 20.It never fails,planning a big picnic and it's sure to rain.
x

11.2 PART B

21.W:Are you looking forward to your move in April? M:You bet I am.The rooms here are too small and there is no storage space. Q:What is the man going to do? 22.M:There are some intersting snells on the beach today. W:What if we collect some after lunch? Q:What does the 59

60

\gmo 92 d 10 nj^i`kp

4H6X 93 B 1 VN8L<PZ
93 1*

"0
x

12.1 PART A

1. Ed, move that chair, will you? 2. Diane looks up to her teacher. 3. I left my grocery list at home. 4. The novel is better than I had expected it to be. 5. He didn't do the work alone. 6. What is there left to do before the end of the day? 7. Formal dress was not required. 8. I rarely eat meat. 9. Rita got her roommate to help her with the packing. 10. The only drawback to your plan is the expense. 11. Last night I tossed and turned in bed, too exhausted to sleep. 12. How can we be out of salt? I bought some only last week? 13. Let's not make them do all the work. 14. She went back to classes right after she was released from the hospital. 15. They disconnected our telephone this morning. 16. Lois nally overcame her fear of public speaking this evening. 17. You had work won't go unrewarded. 18. To be honest, I didn't think much of the play. 19. The professor asked us to look over our papers for mistakes. 20. Few people sat through the entire movie.
x

12.2 PART B

21. W: We should spend this weekend at the beach. M: How about the whole week? Q: What does the man mean? 61

62

\g]o 93 d 1 nj^i`kp

22. M: I missed the bus again today because I turned the alarm clock o in my sleep. I don't know what to do. W: Try putting it far enough away from your bed so you'll have to get up tu turn it o . Q: What does the woman suggest the man do? 23. W: When was the last time you dusted this apartment? M: When was the last time my mother came over? Q: What does the man mean? 24. W: This co ee never seems to taste quite right to me. Maybe we should buy a di erent brand. M: Why not a new co ee pot? Q: What does the man suggest? 25. W: Can I bring you back anything from the store, Sam? M: Get me a roll of Scotch tape and a notebook, will you? Q: What does Sam want his friend to buy? 26. M: Harry seems to be in a bad mood today. W: He'll snap out of it when he nishes his biology project this afternoon. Q: What does the woman imply about Harry? 27. M: Want to go for a long walk or a bike ride? W: You don't want to study now, do you? Q: What does the woman mean? 28. W: I borrowed Peter's favorite pen and now I've lost it. M: When he nds out, he will really raise the roof. Q: How will Peter react when he hears what the woman did? 29. W: That's an amazing sculpture, isn't it? M: You can say that again. Q: What does the woman mean? 30. M: The weather forecast is for rain tomorrow. W: Do you think we'll actually get some this time? Q: What does the woman imply? 31. W: It's no use, Tom, the people next door to us are making so much noise. I can't read this book. M: Why didn't you stay at the library, Ann, it's much quieter there.

12.3 PART C

63

Q: What does Tom imply? 32. M: It must have taken you a long time to x up all these bookshelves. W: It wasn't too bad, I got Doris to do some of them. Q: What does the woman say about the bookshelves? 33. M: Did you know that Bob is leaving for home tonight? W: He can't get away with that. Q: What does the woman mean? 34. M: Where did you put the iron? W: Frank had it last. Q: What does the woman say about Frank and the iron? 35. M: How did you ever nish that long novel so fast? W: It was fun to read. But what was hard was remembering all the characters' name. Q: What does the woman mean?
x

12.3 PART C

Questions 36-39 M: Do you know what happened to me today? I was so embarrassed. W: What? M: Well, Doctor Brown's class nished ten minutes early, so I went to the library between classes. I know I didn't have much time, but I wanted to get those books on industrial revolution. I looked them up in the card catalog and went right to the stacks and found them. So I put them in my book-bag and headed back toward the door. Then it happened. The exit gate in front of the door wouldn't open and a guard immediately warned me that I hadn't checked out my books. He thought I was trying to steal them. W: That must have been embarrassing. Then why didn't the exit gate open? M: I asked him that. It seems that the books in the library are all magnetically coded and when you check them out, the librarian behind the desk demagnetizes them, then the exit gate will open. W: How interesting. I still don't quite understand how they do it though. I'll have to go to the library and see it for myself. 36. Why did the man decide to go to the library? 37. After getting the books, what did the man do?

64

\g]o 93 d 1 nj^i`kp

38. According to the man, what happens to all the books in the library? 39. According to the man, what does the librarian behind the desk do? Questions 40-42 To play their music well, jazz musicians don't really need to know a lot about the rules of harmony and rhythm. Of course they might have a knowledge of both harmony and rhythm, but that kind of knowledge isn't what makes them good jazz musicians. What does make them good is having an intuitive feeling for how the music works. For artists in the medium of jazz, the music comes naturally. It ows almost spontaneously through them. Their music performance is not planned in advance. As they play, they don't monitor themselves in term of a formal theory of performance. As a result, jazz is a natural expression of moods and feelings of the artists, a moment by moment expression of the self. Now you are in for a treat. I brought some recordings of my favorite jazz artists. We'll spend some time listening to some examples of really good jazz. 40. What is the main purpose of the talk? 41. What does the speaker say as a distinctive characteristic of jazz musician? 42. What will the class do next? Questions 43-46 M: It's good to see you back. How was your trip to Arizona? Did you see the Great Canyon? W: Yes, it was fantastic. Now I know why it is listed together with the Nile river as one of the river as one of the great natural wonders of the world. Here, want to see the picture I took? M: Oh, what a view! It is even bigger than I had imagined. I remember reading about the Great Canyon and I think it said that it was formed suddenly when the earth crust split open during an earthquake. W: Well, not that quickly. Look, here is the picture of the Colorado river. See way down at the bottom of the canyon, that river and its tributaries have been wearing the canyon oor away for ten million years. M: Then the canyon is the result of soil erosion. I'll bet geologists have made some interesting discoveries there. W: Not only the geologists, archaeologists have also found the bones of extinct animals in caves in the canyon walls. In a cave one hundred and forty feet about the river, they found drift wood that dated back as far as thirty seven thousand years. M: That means the river must have been a hundred and forty feet higher up when it carried the drift wood into the cave. W: It's very possible, but of course who knows. The Great Canyon is full of mysteries. Wouldn't you like to go to Arizona some day?

12.3 PART C

65

M: You bet I would. 43. Where has the woman been? 44. What type of scienti c discoveries does the woman mainly talk about? 45. Where does the woman say the drift wood was found? 46. What would the man like to do some day? Question 47-50 Good morning listeners, welcome once again to the WCHB series listen and learn, with the approach of summer, you may have noticed trees in your area blossoming and their fruit ripening. But have you ever wondered how fruit ripens? Especially after it has fallen from the tree or vine? The answer is relatively simple. A single chemical called ethylene is produced by the fruit itself and it is this chemical that causes the fruit to ripen. When the fruit produces increased amount of ethylene, the ethylene a ects the fruit physiologically, the fruit begins to breathe oxygen, and the oxygen supply in turn raises the internal temperature of the fruit. This increase in internal temperature allows ripening process to begin, with fruit becoming sweeter, less green in color and softer, in short, delicious. Until our next broadcast, this is Jane Anderson wishing you a fruitful day. 47. What is the audience listening to? 48. What is the main topic of the talk? 49. What is the source of the ethylene described in the talk? 50. According to the speaker, when fruit breathes oxygen, what begins to happen?

66

\g]o 93 d 1 nj^i`kp

4HFX 93 B 5 VN8L<PZ
93 5*

"0
x

13.1 PART A

1. You like researching term papers? 2. I feel as though I've got a fever, but my temperature was normal. 3. I can't believe how good the new furniture looks. 4. Solving the puzzle is easy for everyone except Michael. 5. It's a great day to go for a drive, don't you think? 6. In spite of himself, he couldn't stopping eating. 7. George likes running more when he has a partner. 8. The students can register for the course during the rst class. 9. Won't Lynn be sorry she didn't go to the seminar! 10. He'd take his responsibilities as class president seriously, wouldn't he? 11. Peggy proved herself to be a ne student. 12. I don't believe you understand my question. 13. My aunt is about to publish her third book. 14. The restaurant won't reopen until the renovation's completed. 15. If I'd known he was driving downtown, I'd have asked him for a ride. 16. Is there room for one more at this table? 17. If we leave home at eight, we should get there in plenty of time. 18. You are asking what I think about this proposal? 19. Please drop me a line when you get a chance. 20. The school in rmary doesn't stock prescription medicines at this time.
x

13.2 PART B

21. Grace and his friend went down to that old house on Nasle street. What for? What does the woman want to know about Grace and his friend? 22. Do you know where I can get a copy of that report? 67

68

\gfo 93 d 5 nj^i`kp

How about the dean's o ce? What does the man mean? 23. This is certainly a great restaurant. Isn't it ever? How does the man feel about the restaurant? 24. We should get started on the project. The sooner the better as far as I'm concerned. What does the woman mean? 25. Frank is certainly in a good mood. The bargain he got on his new stereo made him very happy. What does the woman say about Frank? 26. I'd like to apply for the part-time job. Fine, just ll out his form, and someone will be with you in a moment. What does the woman mean? 27. They are building a new shopping center near my house. Not another one. What does the man mean? 28. Nancy, you look trouble. I am. I am having di culty deciding on a topic for my history paper. What does Nancy mean? 29. Here I am. I'm lucky I made it to the exam on time. I can see you were in a hurry. You are wearing your sweater inside out. What does the woman mean? 30. Does John know the class is having a surprise party for him when he turns twenty-one this week? No, he thinks we are giving a retiring party for the dean. We've hidden the presents for him. What are the man and woman planning? 31. Is it supposed to rain tomorrow? If you believe the weather forecast, we'd better bring our umbrellas. What does the man mean? 32. I must have seen this play ve times. Don't you think it's great? Not that great.

13.3 PART C

69

What does the man imply? 33. Your son certainly shows a lot of enthusiasm on the tennis court. I only wish he'd show as much for his studies. What does the woman imply about her son? 34. Look outside and see how beautiful the snow is? So we did get snow after all. What had the man assumed? 35. Has the technician called about the repairs yet? When he does, I'll have you talk to him. What does the man mean?
x

13.3 PART C

Questions 36-39, Good afternoon, I'm Torris Don, housing director here at the university. I'm visiting all the dormitories this week to inform students about check-out procedures, I know you have a lot on your mind with nals coming up, but there are a few things you need to be aware of, as you prepare to leave for vacation. This dormitory will be closed during the summer months and will reopen on September rst. You must vacate your room by June third. Even if you're registered for classes during the summer, you must leave this dorm by June third. If this does pose a problem for you, you should contact my o ce as soon as possible. You should remember to turn in your room key before you leave. Failure to return your key can result in a 25-dollar ne. You must also make sure that all of your personal property has been cleared out of your room. I'll be passing out a form for you to ll out concerning the condition of your room, you should report on the form any damages to your room which has occurred over the last year, such as holes in the room's walls, doors or windows. That way, our summer maintenance crew will know where to make repairs before the next school year starts. If you have any questions during the next few days, please ask your residence advisors or call my o ce. Now please take one of the forms as tey are passed around. 36. What does the speaker mainly discuss? 37. When does this talk take place? 38. Before they leave their dorm rooms, what must students do? 39. What must students indicate on the forms? Questions 40-42, listen to a conversation between two members of a university choir.

70

\gfo 93 d 5 nj^i`kp

M: It sounds like a lot, but by the time you add up the air fare, meals and lodging, it's pretty reasonable. W: But I doubt we'll all be able to go. Remember there are 32 people in the choir this year. M: We can make it. For our last trip, we got the alumni to make donations and the professor applied for a grant from the Association of Choral Directors. W: Did he put anyone is charge of fund raising? M: Russ. She took care of it last time. She'll probable be calling the choir members to work with her. W: I really hates soliciting donations, but it'll be worth it, this tour is a once-in-a-life-time experience. 40. What problem are the speakers discussing? 41. What will Russ probably have to do? 42. Why will Russ probably be calling the Choir members? Questions 43-46 Word comes from California of a new weapon in the war on household pests. Two scientists working for a rm in Annahine, California developed a new method to eliminate insects without using dangerous chemicals. The new poison: hot air. The basic idea is that insect pests can not adjust to temperatures much above normal. In laboratory experiments, cockroaches and termites can't survive much more than a quarter of an hour at 100 degrees. Ferenhite or about fty degrees Certigrade. The new method involves covering a house with a huge tent, and lling it with air heated to around 65 degrees Certigrade. Hot air is forced in with fans and the tent keeps the heat inside the house. Since termites must be continued for four to six hours. But when it's all over and the insects are dead, there is no toxic residues to endanger humans or pets, and no funny smells. Scientists claim that there's no danger of re either. Since very few household materials will burn at 65 degrees Certigrade. In fact, wood is prepared for construction use by drying in the ovens at 80 degrees Certigrade, which is substancially hotter. 43. What is the program about? 44. What makes the new system better than other treatments? 45. Why are the houses covered with tents? 46. Why does the announcer mention that construction wood is dried by heat? Question 47-50 M: Math Department, Doctor Webster speaking. W: Hello, Prof. Webster, this is Janet Hill calling, I'm living two doors down from your teaching assistant, Don Williams. Don asked me to call you because he has lost his voice and can't talk to you himself.

13.3 PART C

71

M: Lost his vice. Oh, what a shame! Is there anything I can do for him? W: Well, he has a class this afternoon from two-thirty to four and he won't be able to teach it, but he doesn't want to cancel it either. M: Want me to try to nd somebody else to teach the class? W: No, not exactly. What he wants to do is to get someone go in for him, just to pass back the mid-term exams. He's already marked tem and they are on the desk in his o ce. The whole thing wouldn't take more than ten minutes. M: His classes are two-thirty, eh? Well I'm afraid at that time I was going to be on campus anyway so I could do it for him. What room is his class in? W: Cader Hall, room two-fourteen. Will you need his o ce key to get the exams? He's given it to me and I could bring it to you. M: Actually, that won't be necessary. We have a master key in the maths department. So I can get into his o ce. W: Thank you very much, Prof. Webster. Don doesn't have another class to teach until Thursday, and hopefully, he will be able to talk by then. He'll call you as soon as he can. Oh, yes, I almost forgot. Could you put the next assignment on the board, too. It's all the problems on page forty- ve, and they are due at the next class. 47. What is Don's problem? 48. What favor does Don want someone to do for him? 49. What does Janet o er to do? 50. What does Janet almost forget to ask professor Webster?

72

\gfo 93 d 5 nj^i`kp

4HJX 93 B 8 VN8L<PZ
93 8*

"0
x

14.1 PART A

1. I'm about to go swimming. 2. He doesn't like to drive at night unless he has to. 3. I can't get used to this hot weather. 4. I preferred to sit in the aisle seat. 5. Didn't the summer just y by! 6. No sooner had I turned on TV than the phone rang. 7. The bookstore's out of lined notebook paper. 8. No one understands cameras better than Mary. 9. My father makes me promise to study hard. 10. They can't go to Europe this summer because of college expenses. 11. You haven't gone to the dean yet? 12. Cathy is running out of time on her project. 13. The meeting shouldn't take more than ten minutes or so. 14. She was afraid she'd miss her class. 15. What a week this has been! 16. Nancy got Ann to answer the phone for her. 17. I can't deny that what you say may have some basis in fact. 18. Masmi's looking into the possibility of getting a loan. 19. Carl was broke after his vacation. 20. You need to spell out clearer instructions for me.
x

14.2 PART B

21. Do you mind if I use your phone? Of course not. What does the woman mean? 22. It's a nice pen, where did you get it? 73

74

\gho 93 d 8 nj^i`kp

It came in the mail yesterday from my mom and dad. What does the man say about the pen? 23. I can't stop thinking about my research. Neither can I. I am working on my these day and night. What does the man mean? 24. Do you think the price of our meals will go up next term? There is no question about it. What does the woman expect? 25. I wish I could nd that new grocery store. It must be around here some where. Maybe you'd ask the newsstand, they are often helpful. What does the woman suggest that the man do? 26. Thanks for bringing the books back. I thought you need them to study over the weekend. Thanks for letting me use them. What does the man done? 27. Will you come to my poetry reading next week? I'll be out of town then. What does the man mean? 28. Why were you late for the meeting this afternoon? I just lost track of time. What does the man mean? 29. We need to let everyone know about the bene t concert, but we don't have much money for advertising. How about using the school radio station? They broadcast free public service announcement. What does the man suggest that they do? 30. Janison said he wanted to get involved in the student government this year. But he hasn't gone to a single meeting, has he? What does the man imply about Janison? 31. It's almost ve o'clock and Fred'd not here yet. If he doesn't hurry, he won't make his ight. Why is the woman worried? 32. I heard there are a few seats left for tonight show.

14.3 PART C

75

Really? I was under the impression that they were sold out a long time ago. What does the woman mean? 33. The students have been protesting against the increasing tuition. Yes, I heard about the protest, but I don't know how much good it will do. What does the man mean? 34. Would you pass me the sport section, please? Sure. If you give me the classi ed ads and editorial section. What are the people doing? 35. Professor Cux said you can pick up your term paper at her o ce. So, she has graded it. What had the man assumed?
x

14.3 PART C

Questions 36-39, listen to a conversation. M: I'm thinking about transferring out of state college into another school in the fall. W: After a year and a half? How come? I thought you liked it here. M: I do. But here our ight program doesn't give degrees, only licenses, and I need both a license and a degree. W: So where do you want to go? M: I wouldn't wind going to Makmillen university. It has an excellent reputation for its school of aviation, but I have a feeling it's very selective. W: But you've gotten good grades in the three semesters you've been in the state college, haven't you? M: Yah, mostly A's and B's. W: So what were you worried about then, just ask your professors to write letters of recommendation for you, and you'll be set. 36. Who is the man planing to do? 37. What kind of career is the man interested in? 38. What concern does the man have about Makmillen University? 39. What does the woman tell the man to do? Questions 40-42, listen to a talk:

76

\gho 93 d 8 nj^i`kp

Good afternoon! As you know, this is a course in beginning political theory. We'll be meeting each Monday, Wednesday and Thursday at 8:00 clock for the next twelve weeks. Each Monday, I'll give a lecture on the di erent concept of political thought and I will talk about justice for example. On Wednesday, you'll have to hand in one-page paper on the topic, and as a group. We'll discuss the issues involved. On Thursday, I'll turn your paper with suggestions for a revision. Also on Thursday, we'll explore through lecture and discussion what prominent political thinkers had to say about the topic. Over the weekend, you'll revise your paper and hand it in on the following Monday. I'll grade only the revised. Keep your papers brief, because I'll accept on papers that exceed a single page. I'll be available on Tuesday and Friday afternoons to discuss your papers with you. Are there any questions? 40. What is the speaker's purpose in this talk? 41. When will the speaker be available to discuss papers? 42. What must a student do with the paper that is returned on Thursday? Questions 43-46, listen to a conversation between two people as they are walking down the street: M: It looks like we are going to have a shower at any minute now. W: I thank so too. Is it strange how you can sense it? M: I know what you mean. W: Look at the clouds in the sky. Whenever they seem dark and low like this, you know it's going to rain, or look at the object that's far away, if it seems clear to you, it's probably going to pour. M: Yes, and everything seems to have a stronger smell to me then. W: I know, and people often say it's going to rain when they have aches and pains in certain parts of their body. My mother always complains that her knee hurts just before a storm. M: And have you noticed how the animals react before a storm? W: Sure, horses and cows will seem to go a protected area and huddle together. M: Yah, but with all these sensible signs, how come we aren't carrying our umbrella with us now? 43. What is the main topic of the conversation? 44. What does the woman say how distant object looks to her before a storm? 45. What does the man say seem to happen to his sense of smell? 46. What do the speakers have this time? Question 47-50, listen to a talk given in a biology class:

14.3 PART C

77

Today, I'd like to discuss about the rst known instance of a strange evolutionary strategy. This phenomenon occurs in the lowering plant called the scarlet jelly. The scarlet jelly depends on both hummingbirds and months for pollination, and the blossoms seem to change their color for each of these pollinators on rm mountain near Steven, Arizona, where these owers grow, hummingbirds arrive at mid-July to drink nectar from the scarlet jelly. In July, the owers are bright red, which is attractive to the hummingbirds. By late August, however, the hummingbirds leave the area for warmer climate. As the weather cools, the color of the plants blossoms change to white, a color that attracts a particular moth species, the night- ying hawk moths. Apparently, this kind of moths can see these white blossoms easily at night. These moths continue to pollinate the owers until well into September. The scarlet jelly demonstrates an unusual way in which a plant lures pollinators to continue its life cycle. 47. What is the main topic of the talk? 48. Why is the scarlet jelly, the plant, considered to be unusual? 49. Why did the hummingbirds leave the mountains? 50. According to the speaker, in that way a white blossom is attractive to moths?

78

\gho 93 d 8 nj^i`kp

4HRX 93 B 10 VN8L<PZ
93 10 *

"0
x

15.1 PART A

1. She dropped o to sleep in the chair instead of getting up and going to bed. 2. Tuition has nearly doubled in the last ve years. 3. Won't you please get in that line? 4. If I only saved more money. 5. The meeting has been postponed until further notice. 6. They used to eat in the cafeteria, but now that they have an apartment, they cook at home. 7. Just between you and me, I think Je 's sister swims better than he does. 8. None of the people who applied for the job has the required credentials. 9. They won't have gone to the gym without us. 10. I wish I had tried these shoes and see if they t. 11. Ann rides her bike to work to save money on bus fare. 12. Not one came to the school play. 13. It's been a year since I last saw my old roommate. 14. There aren't as many trees as there used to be. 15. Never have I heard such a ridiculous excuse. 16. I pull down the shad to darken the room. 17. I was late coming back iron lunch today. 18. The actors received a standing ovation for their performance. 19. The candidate was speechless overwhelmed by the size of the crowd. 20. I lent him my car for the day.
x

15.2 PART B

21. Do I have to type this application? You'd better if you want to get the job. What does the woman mean? 22. Let's go to the zoo when we are in San Diego. 79

80

\glo 93 d 10 nj^i`kp

How can we do that? We'll be there just a few hours. What does the woman imply? 23. I have trouble concentrating when my roommate plays loud music. Why don't you just ask him to turn down the volume, Harry? What does the woman suggest to Harry? 24. I wish the plane would get here. It's already eight-thirty. There's no need to get nervous. The announcement said it would be fteen minutes late. What does the man mean? 25. Weren't you a little cold at the parade yesterday? A little cold, I froze. What does the man imply? 26. If I eat another bite, I'll burst. Me, too. What does the man mean? 27. Matthew wants to know if he can go with us to the party. That's odd, this morning he said he wanted to go by himself. What does the man mean? 28. Have you heard who won the election? I missed the news and haven't had time to read the paper. What does the woman mean? 29. Isn't it wonderful that Sally won the scholarship? You said it. What does the man mean? 30. I thought I'd give Pam a call now. Isn't it a little late at night? What does the woman imply? 31. I think we should move on to the next item. Ok, but I'd like to take this matter up again at the end of the meeting. What does the man mean? 32. If you are interested in gymnastics, the university has an outstanding team. I can't even climb the stairs without having to stop to catch my breath. What does the man mean?

15.3 PART C

81

33. Do you always look over your papers for typing mistakes. I usually have to do it myself, but I'd rather have someone else proofread them. What would the woman prefer to do? 34. I thought Paul might be able to help me gure out this computer program. Paul is about the last person I 'd ask if I were you. What does the man imply about Paul. 35. Let's get this desk in order. Yes, it's about time we cleared it o . What can be inferred about the desk?
x

15.3 PART C

Questions 36-39, listen to a conversation. W: Good afternoon, Dr, Anderson's o ce. M: Hello, my name is John Cremer, and I was hoping I could come in today to see the doctor. W: Are you a patient of Doctor Anderson? M: Well, no. I'm in town at a convention and the manager of the hotel where I'm staying suggested that I call you. W: What seems to be the problem? M: Well, I got this ringing in my ears. W: Are you in any pain or is there any discharge? M: No discharge, just a slight earache. W: The doctor will be busy at the hospital until this evening. The earliest you could see him would be tomorrow morning at ten. M: I'll come in then if that's okay. W: That will be ne, in the meantime you should try to take it easy. 36. Who is the man speaking to? 37. What is one of the man's symptoms? 38. Where is Doctor Anderson now? 39. What advice does the woman give the man? Questions 40-42, listen to a lecture about the history of higher education in the United States: Three developments in the United States higher education that you are bene ting from today started more than a century ago following the civil war. The rst of these was the rapid growth

82

\glo 93 d 10 nj^i`kp

of the technological, engineering, architecture, law and medicine. Second was the provision of graduate study, such as what had long existed in France and Germany. Harved and John Hopkins Universities quickly took the lead in this eld, but the state universities did not lag far behind. Third was the increased provision for the education of woman. This included the establishment of new woman's colleges, such as Vasa, Welisly and Smith, and the adaption of co-education, the provision for graduate study, and the increased opportunities for woman began over a century ago, continue to this day, well over then decades since the end of the civil war. 40. What is the main topic of the talk? 41. How many major educational changes does the speaker discuss? 42. From the talk, what assumption can be made a about education for woman in the United States prior to the civil war? Questions 43-46, listen to some students' talk about a job on campus. W: Hi, Larry, are you waiting to see Professor Jamason, too? M: Yah, since I got one of the ve highest grades in her managerial economics class, she asked me if I'd be interested in working as her assistant next semester. I'm here now for my interview. W: Oh, yes. I know all about that job. I did it two years ago. M: Really? Did you like it? W: I think it was the best I've had at school. It paid eight dollars an hour, which was three dollars an hour more than I got working at the school post o ce the year before. M: That is a good salary. What did you do? W: I was in charge of grading all the problem sets that were assigned as homework. I never had trouble doing it, and of course, Prof. Jamason was always available to help me if I had any questions. M: I think I'd enjoy doing that sort of work. It would be very good experience for anyone thinking about becoming a teacher. W: Absolutely. You also learn how to use the computer data base, because the records are kept on it. And building up your computer skills is a good preparation for lots of jobs. M: The job sounds great, but I'm a little worried about how much time it might take? W: It's pretty reasonable. It never took me more than ve hours a week to do all the grading and then another thirty to fourty minutes to record the grades on the computer. M: That sounds manageable. I guess you can do the work when it ts into your own schedule, too, can't you? W: Oh, yah, you can do the grading in your room or in the library. You just need to get each set back for the next class, but that means you always have at least two days and sometimes four.

15.3 PART C

83

M: It sounds great. W: Good luck with your interview. 43. Why was the woman able to give Larry so much information about the job? 44. What do the students say about the salary for the job? 45. What does Prof. Jamason's assistant do? 46. What is Larry's main concern about the job? Question 47-50, listen to a talk given at a meeting of bicycling club. Good evening, welcome to the rst meeting of our spring cycling season. It's pleasure to see to many new faces here. I certainly hope that most of you will soon feel right at home with our group and bicycling will become a regular hobby for you and a part of your physical tness routine. Some of you may not realize that the state of New Jersey o ers ample opportunities for bicyclists of all abilities. We have rolling countryside, miles of beaches, lovely hills and valleys and thousands of mile of little used roads. Beginners will nd the shore and area south of Princeton suitable for their needs while expert riders will discover that the steep trails of the region around High Point o er a challenge to their skill and stamina. In addition, New Jersey is rich in places of historical interest. The state abounds in colonial architecture, battle elds of the revolution and other historic sites important in the early history of this country, most carefully preserved or have been meticulously restored. We organized tours to a variety of places nearly every weekend. Next Saturday tour will be a relatively easy one, from Recters University to the gateway National Park. Interested cyclists should meet at eight a.m. in the parking lot at the corner of Hamiltom Street and College Avenue in New Branderswork. The minimum time of this tour is about six hours. We'll use a buddy system throughout the ride. Each new rider should team up with a more experienced rider. If the buddies look after each other, we can be sure that no one will be left behind on the back roads. You can pick up a map of Saturday's tour at the information table before you leave this evening. 47. What's the purpose of this talk? 48. According to the speaker, what makes New Jersey a good place to bicycle? 49. Why does the speaker mention the historical sites? 50. What is the purpose of the buddy system?

84

\glo 93 d 10 nj^i`kp

4H>X 94 B 1 VN8L<PZ
94 1*

"0
x

16.1 PART A

1. Today is the last day to register for the class, isn't that right? 2. The bill arrived from the bookstore a week ago. 3. Did David show up at the game? 4. Under the window is the new heating unit. 5. Mary used to enter all the races, but now she's given up running. 6. This sauce is sweeter than I prefer. 7. Let's work at home this week. 8. You will come to the art exhibit, won't you? 9. My sister lent me some change. 10. As soon as Larry gets here, we'll leave for class. 11. I can't help thinking it was all my fault. 12. Terry and Chris take notes for him. 13. The salesperson misread the price tag. 14. It pays to take a look at the laboratory manual before a class. 15. We couldn't be more pleased by the pictures. 16. I really felt the movie was a waste of money, didn't you? 17. If only he'd drive more slowly. 18. The dean herself phoned the registrar's o ce. 19. Jane's essay was criticized by a famous author. 20. Don't tell me you're going out again.
x

16.2 PART B

21. You are looking a little overwhelmed. No wonder, I got a million things to do and all of them have to be nished in the next twenty minutes. What does the man mean? 85

86

\gao 94 d 1 nj^i`kp

22. Our basketball team's won every game so far. Isn't that because of the new coach? What does the man imply? 23. Were there a lot of people at the concert last night? It was really pretty crowded, some people were standing in the aisles. What does the man mean? 24. Do you have any idea what this notice is all about? I'm as in the dark as you are. What does the woman mean? 25. I thought you are going to take that psychology course at graduate school. I was, but I spoke with Dean Johnson, she talked me out of it. What did Dean Johnson do? 26. Has the school closed because of the weather? Only once, I think. What does the man mean? 27. Lois's going to play volleyball this afternoon. Should he be doing that today so soon after her injury? What does the woman imply? 28. Aren't you watching the movie on television? It is not until eight. What does the woman mean? 29. This is the longest assignment we've had all semester. You're telling me, we'll be lucky if we can do half of it. What does the woman say about the assignment? 30. What time should I be at the festival this evening? On, so you can come, about 7:30. What had the woman assumed? 31. Bob wants to know how much this car costs? Why, he can't buy a new car until he sells his old one. What does the man mean? 32. Would you like some dessert? No thanks, I've been sitting here too long already.

16.3 PART C

87

What does the woman mean? 33. Larry certainly made a mess of that plain job. Don't be too hard on him, he was only trying to help. What does the woman say about Larry? 34. You really should get more exercise, Hellen. Well, look who's talking. What does the woman imply about the man? 35. Some of the younger students seemed quite aggressive in the meeting. Yes, they really took over when it came to the issues of voting rights. What does the man say about the younger students?
x

16.3 PART C

Questions 36-39, listen to a conversation between two friends. W: I think we ought to turn around and head back down the trail. M: Tired, after just a few mile? W: Not at all, but those are cumulonimbus clouds building up there. M: So, I don't really care if it gets cloudy. The sun's been pretty strong. W: Right, but those clouds mean a thunderstorm, lots of rain and sometimes even hail with those kinds of clouds. M: That doesn't sound so great, but I thought that those cumulus what it's were fair weather clouds. W: You are thinking of cumulus clouds I bet. Those are de nitely cumulonimbus. M: How on earth did you ever hear all that, from a meteorological course? W: I'm an English major, but growing up on a farm, you learn a lot about the weather. M: You convince me, let's turn back, and maybe on the way down, you can give a lesson on clouds. W: Sure thing, but let's get going. 36. Where does this conversation take place? 37. What is the woman trying to accomplish in this conversation? 38. What does the woman do at present? 39. What will the man and woman probably do next? Questions 40-45, listen to a talk.

88

\gao 94 d 1 nj^i`kp

The silkworm gave people the idea of making rayon, a synthetic material that feels much like silk. Container De Shatiney was the scientist who nally succeeded in initaing silk commercially. Shatiney noticed that the silkworms digested mulberry leaves into liquid cellulose, and force the liquid out through tiny holes to form slender threads. He was unable to match the exact digestive chemistry of silkworms, but he tried desolving mulberry leaves in nitric acid to extract the cellulose, then imitated silkworms by forcing the mixture through tiny holes, Shatiney tried using trees as another source of cellulose, and nd the results were just as good. Since then rayon has replaced silk in many commercial enterprises. 40. What is the main topic of the talk? 41. What is Shatiney known for? 42. What especially interested Shatiney about silkworm? 43. What do silkworms eat? 44. How did Shatiney imitate the silkworm? 45. What did Shatiney use trees for? Questions 46-50 W: Can I help you? M: Yes, I'd like to read some articles that are on reserve in the library for anthropology 311. W: Professor Gilers's class? M: That's right. How could you know? W: Let's just say you are not the rst person coming in asking for those articles. M: Oh, well, seeing as how I haven't read any of them yet, it doesn't really matter which one you give me rst. W: I'm afraid I can't give you any of them at the moment. They've all been checked out. M: You're kidding, all of them? W: Every last month. I've asked professor Gilers twice already to bring in additional copies of the articles, but no sooner do I place them on the shelves than they are gone. See that girl in the black sweater? She's been waiting for half an hour for these same articles to be returned. M: And here I went out of my way to free out the whole afternoon to read. W: I'm sorry, but there is not a whole at all I can do about it. All I can suggest is that you come in rst thing tomorrow morning and try again, we open at eight. 46. What can be inferred about the woman who's speaking? 47. What is the man surprised to learn? 48. What can be inferred about the students in anthropology 311? 49. What does the woman say about Professor Grilers? 50. What does the woman suggest that the man do?

4HDX 94 B 5 VN8L<PZ
94 5*

"0
x

17.1 PART A

1. Frank likes his co ee black, but Sarah takes hers with cream and sugar. 2. The start of the football game has been delayed an hour. 3. Another lab report, that's what I have to do over the weekend. 4. I've discarded the old paper. 5. It'll take me only three minutes to duplicate a key like that. 6. The speaker showed up late for his statistics lecture. 7. I almost thought you were someone else with that short haircut. 8. No matter how hard he tries, he doesn't seem to get anywhere. 9. My sister detests rock music. 10. This computer is driving me up the wall. 11. This is supposed to be ction but I think the author has drawn on her own experience. 12. Despite chilly weather, we royal fans enjoyed every second of the outdoor concert. 13. I got the time mixed up and went to the wrong class. 14. Only breakfast will be served at the cafeteria during the holidays. 15. He climbed those steps? 16. The place in the book was marked by a sheet of paper. 17. The new school is nowhere near nished yet. 18. No sooner had we begin having dinner than the doorbell rang. 19. I expect to have the papers ready to sign by this afternoon. 20. Her discovery was critical in the development of the vaccine.
x

17.2 PART B

21. Were you able to understand that French novel without any help from the teacher? I did pretty well, but I had to fall back on my dictionary occasionally. What does the man mean? 22. It's much too beautiful a day to waste indoors. 89

90

\geo 94 d 5 nj^i`kp

You're right. Let's make the most of it. What are the women probably going to do? 23. Do you think we can wrap up this report by ve? Have you looked at the clock recently? What does the man imply? 24. Hi, Janis, do you want to see a show tonight? I've got two passes to the theater. Thanks, but the Olympics are on the television. What can be inferred about Janis? 25. It never fails, it's raining hard outside and I'm stuck without an umbrella. I'd like to let you have mine, but I have to go out soon. What does the man imply? 26. Both Sam and George are very strong runners. I've got a feeling San's going to nish rst. What does the man mean? 27. Sally really is amazing, isn't she? She has that part-time job and she studies harder than most of us. I know, yet she still nds time to volunteer at the hospital. What do the speakers imply about Sally? 28. I've got to x the hole in my coat. Why not just buy a new coat? What does the man imply? 29. I never seem to have any money. Why not put part of your salary in a special account each week? What does the woman suggest the man do? 30. Barbara, I'm glad you could assist us in our demonstration, but aren't you supposed to go to Dr. Clown's lecture today? I sent Kathy to take notes for me. Why is Barbara free to help with the demonstration? 31. Any luck at the box o ce? I got us two tickets for the Mtinee, but one is in the 4th row the other's in the 8th. What does the woman mean? 32. What did you use to make that soup?

17.3 PART C

91

I put everything in the refrigerator into it. What does the woman say about the soup? 33. The dean just announced that Dr, Je ery is going to take over as Chairperson of the English Department. I knew it all along. He is the obvious choice. What does the man mean? 34. Caroline's postcard says she'll be back today. What our-of-the-way place did she send it from? What does the man want to know? 35. Excuse me, can you tell me where the Students' Association is meeting? It was going to meet in Room 110 but was changed to the auditorium for more space. What does the man imply?
x

17.3 PART C

Questions 36-38, listen to a conversation between a police o cer and the young man: W: Can I see your license and registration please? M: What is the matter, o cer? Did I do something wrong? W: You were speeding. M: I was speeding? W: You certainly were. Do you have any idea how fast you were going? M: I'm not sure, but I was going about 35. W: This street is in a 25-mile-per-hour speed zone, you know. M: It is? W: In this state, the speed limit in residential area is 25 unless otherwise posted. Besides, there are signs all along the streets. M: I'm sorry, o cer, I guess I didn't notice. I was thinking about my job interview. I'm on my way to it right now and I haven't planned to come this way. W: What do you mean? M: Well, I was coming down on Elm Street but the tra c was really backed up. There was some kind of construction going on. I turned on to a side street and ended up here. Give me a break. I'm nervous enough as it is. W: Well, since you have an interview. I'll give you a warning this time. But keep your eye on the speedometer from now on, whether you're on a street you are familiar with or not.

92

\geo 94 d 5 nj^i`kp

36. Why does the o cer think the man should notice the speed limit? 37. What can be inferred about the man? 38. What does the police o ce nally do? Questions 39-42, listen to a talk given to students about recycling trash. Thank you all for coming today to hear about the new recycling program on campus. Right now all recycling is voluntary, but in the near future it will be most likely mandatory. The recycling we've planned is really a 3-part program: sorting, collecting and distributing. The rst part, sorting, is the hardest to get people to do. Most people don't like to take the time to separate their trash. And this is usually where most recycling programs fail. However, we are determined to make ours work. Instead of seeing the usual brown trash cans all over the campus, from now on you'll see four di erent colors of trash cans. Pink thrash cans are for paper, green cans are for glass, maroon ones are for metal and white ones are for other wastes. Just remember, pink: paper green: glass maroon: metal white: waste. After the trash is sorted, they need to be collected and them distributed to the correct recycling facilities. That's where all of you come in. Since this program's still a volunteer program, we're asking our students to give a little of their time to the project, probably no more than 6 hours per person each week. And again, your job is to collect the sorted trash and take it to di erent centers for each type of trash. If you are willing to help us with the program by giving 6 hours a week, please write your name and phone number on this note pad. 39. What is the main purpose of the talk? 40. According to the speaker, why do most recycling programs fail? 41. What will be the color of the trash can for paper? 42. What will be the primary responsibilities of volunteers? Questions 43-46, listen to a conversation between 2 university students: W: Oh, Larry, I have been meaning to talk to you. M: Hi, Jenis. What's up? W: I have this great job lined up to manage the clothing store at the mall. M: So what's the problem? W: Well, one of the professions in my department just told me about a summer internship program that's available. She thinks I might be able to intern in the o ce of the Wayfare Hotel here in town. M: That sounds like a great opportunity too. Why not take advantage of it? W: I'd love to, especially since I'm studying hotel management. It would be a great way to get some practical experience in my eld.

17.3 PART C

93

M: And you never know, it might lead to something with them after graduation. They are one of the biggest hotel chains in the area. W: You're right. But the drawback is I wouldn't be making nearly as much money as I would be working in the clothing store, not to mention the discount I could get on clothing there. M: How much is the internship paid? W: They pay their internship a small stipend and give them free room and board for the summer. M: Well, if I were you, I would take the internship anyway. You could always get a job during the school year next year to make a few extra bucks. 43. Why does Jenise want to talk to Larry? 44. Why might the summer internship be a good opportunity for Jenise? 45. What is the main disadvantage of the summer internship for Jenise? 46. What does Larry suggest Jenise do? Questions 47-50, listen to a talk about cola avored soft drink. How many of you drink cola? Nearly everybody. Did you know that cola started out not as a soft drink but as a cure for headache back in the late 1800's? John S. Pamberton, a druggist from Atlanta, had experimented for many months trying to nd a cure for the common headache. The worked in his backyard, mixing and heating di erent combinations of oils and avors until he found one that seems promising. Pamberton bottled the mixture and began selling it in drugstores as a concentrated syrup that the customer had to mix with water before drinking. Cola's transformation from a concentrated syrup to a carbonated soft drink came about quite by accident. One day, a customer came into a drugstore complaining of a headache and asked for a bottle of cola syrup. He wanted to take it right away. So he asked the clerk to mix the medicine while he waited. The clerk, instead of walking to the other end of the counter to get plain water, suggested mixing the syrup with soda water. The customer agreed, and after drinking it, remarked how good it tasted. The clerk continued o ering the mixture and carbonated cola grew in popularity. Today carbonated cola are sold in most countries around the world. And although they no longer contain the ingredients to kill headaches, they are still very refreshing. 47. What does the speaker mainly discuss? 48. How was cola originally sold? 49. Why does the speaker mention John Pamberton? 50. How was cola syrup made into a soft drink?

94

\geo 94 d 5 nj^i`kp

4H2X 94 B 8 VN8L<PZ
94 8*

"0
x

18.1 PART A

1. Don't forget to go to the bank on Thursday because Friday is a holiday. 2. Can we get this tape recorder repaired? 3. I thought Steve was very disagreeable during dinner last night. 4. She doesn't know what to do with her time today. 5. It used to take Betty months to nish reading a book, but now she gets through one a week. 6. The visitors asked us to recommend a restaurant that isn't too far away. 7. Mark likes to spend time with his friends. 8. It will be ten minutes till the basketball game starts. 9. The paintings in Mrs. Peterson's collection are irreplaceable. 10. He was approached by three companies with job o ers, but he turned them all down. 11. He was too restless to sit still. 12. He ran out in such a hurry that he forgot his coat. 13. A store downtown has the same pair of shoes on sale. 14. Donna should save a breath because she can't convince her friends. 15. All we did was to listen to him go on and on about his trip to Washington. 16. It looks as though we've run out of computer paper. 17. I'm not sure how much longer I'll be living a t home. 18. I'm not sure how much longer I'll be living at home. 19. It's only a matter of time before my car breaks down again. 20. The professor used to personalize his talks with stories about his college days.
x

18.2 PART B

21. It's a bit warm out today. Warm? You can fry an egg on the sidewalk. What does the man mean? 22. This must be my tenth cup of co ee today. 95

96

\g o 94 d 8 nj^i`kp

How you're going to be able to fall asleep if you keep on like this? What does the woman imply that the man should do? 23. I thought you were going to read this book by today. Oh, but I already have. What does the woman say about the book? 24. Sarah's the new class president, and she's also one of the best students in the school. You might say she's done all right. What does the man mean? 25. Why don't we drive downtown now? Wouldn't it be better to wait after rush hour? What does the man mean? 26. How many chairs should I bring in for the meeting, six can nd. We have to have at least that many. Bring in all you can nd. What does the woman mean? 27. Oh, I must be sitting in the same position too long. My legs have fallen to sleep. Shake it a little before you get up. What does the woman suggest the man do? 28. Before we play again, I'm going to buy a good tennis racket. Your shoes aren't in very good shape either. What does the woman imply? 29. Let's call carol one more time. Maybe she is home by now. Can't we wait till morning? What does the woman mean? 30. Did you hear about Jim? I wouldn't give that rumor any credibility. What does the woman mean? 31. Why are you squinting at the picture? Well, I normally wear glasses, but I broke them. What does the man imply? 32. Have the parts we need for the copier arrived yet? I ordered them last week, but something's holding them up. What does the man mean?

18.3 PART C

97

33. We have this or you might be interested in these note-cards. I think I'd really rather have some pale yellow paper for my correspondence. Where does this conversation probably take place? 34. I thought you said you couldn't come to the meeting. Oh, no, I can. What does the man mean? 35. This pizza tastes bland there is not enough sauce. I've had that, too. What does the woman mean?
x

18.3 PART C

Questions 36-38, listen to a conversation between two friends: M: Hi, Kate. Mind if I join you? W: No, not at all, have a seat, Jack. M: Have any plans for the weekend? W: Yah, I'm really excited. I'm going up to New York city for a couple of days. M: That's a coincidence,. My roommate's going to New York too. Are you driving. Maybe you two could ride together. W: No, I'm going to take train to the Plain Station. I've already got my ticket. M: So what are you going to do in New York? W: I'm visiting a friend I met last summer at the last music camp. Remember I went to a music camp in Vermount? I shared a camp with Laura and she's just moved to Mahanton. So I'm going up to see her new place. M: You two probably have a lot to talk about? Didn't you tell me you were from New York? W: No, I'm from California. I've never been to New York before, I'll give you three guesses what we are going to do Saturday night. M: Something to do with music, I bet. W: Right, we are going to hear the New York Philharmonic at Lincoln Center. M: Wow, I wish I were going with you. 36. How will Kate travel to her destination? 37. Where did Laura and Kate meet? 38. What do Laura and Kate have in common?

98

\g o 94 d 8 nj^i`kp

Questions 39-43 Scientists believe that when the oceans were young, they contained only a trace of salt, and that the level of salinity has been going up gradually. Over the years, rains have been wearing away the rock mantle of the continent. Minerals released by these erosion have been carried to the sea by the ow of river water. This water adds several billions tons of salt to the ocean annually. There are other sources of oceanic salinity too, one source is underwater volcanic action. These eruptions occurred along the mid-ocean ridge and release great amount of salt into the sea each year. However, these sources don't increase the actual mineral composition of the water by much because the enormous chemical requirements of marine life. Oysters, clams, snails and other mollusk use salt to build shells and skeletons. Coral reefs also consume large amount of salt. Also, some salt leaves the ocean through the process of evaporation. As salts escape into the atmosphere, they go into the creation of rain drops, this is why the coast receives more salt exists in the atmosphere near the ocean, thereby attracting the increased moisture and producing rain. 39. What is the topic of this talk? 40. What does the speaker say about the ocean's level of saltiness? 41. What is one of the sources of the oceanic salinity mentioned? 42. Why does the speaker mention oysters and clams? 43. According to the speaker, why do coastal areas receive more rain than inland region? Questions 44-46, listen to a interview broadcasted through the radio. M: Good morning. This is Allen broadcasting from university radio. In the local news this morning, the mountain-lake cycle tour is about to begin, and in the studio, we have with us Marilen O'conner, president of the Ottawa bicycle club, the tour's sponsor. Marilen, can you tell us what the tour's all about? W: Well, Alien, the mountain-lake cycle tour is a two-day bicycle four from Ottawa to Kingston and back. It is not a race, but more like an endurance test for recreational cyclists. People do it for tness and fun and try to better their time every year. M: How long will it take to cycle one-way, Marilen? W: About seven to nine hours for the average cyclists. Racers can do it in four hours. But I said, the tour is not intended to be a race. M: How many people will be taking part this year? W: This is our biggest year ever. We have twelve hundred cyclists registered. M: I bet they'll block the road when the cyclists start out? W: Don't worry, we've introduced the staggered starts to avoid these problems. Our rst group will leave Ottawa at seven o'clock, and other groups will follow every twenty minutes.

18.3 PART C

99

M: Thank you, Marilen. Now down to the starting line of the university. 44. Where does this interview take place? 45. Who is Marilen O'conner? 46. Why do cyclists participate in the mountain-lake cycle tour? 47. In what order will the cyclists leave the starting line? Questions 48-50, listen to a talk given in an anthropology class. Good morning, class. Last time, we started to talk about some crops and traditions of early agriculture-based society. We had said that corn is one food that dominated the food-growing activities of those early American Indian tribes that relied on farming for food. The early farmers, or the American Indians, planted lots of varieties of corns, including the kind of corns that are eaten today as popcorn. The corn was prepared in di erent ways. It could be eaten fresh from the eld, sometimes it was boiled, sometimes the corn was roasted or patched, but most of the corn was dried on the card, and later it was ground into our. The advantage of this method is that the dried corn can be stored for long periods of time without spoiling. The American Indians harvested the corns twice, the rst harvest was the green corn or sweet corn harvest. This harvest was accompanied by a ceremony of thanksgiving. The farmers from disaster, and the ninal harvesting of the right corn and the harvest ceremony took place about six weeks later. During this harvesting, the farmers pulled back the husks of corn and braided them together. Big bundles were prepared this way and were dried for use later on throughout the dried husks to make all sorts of di erent things like mats, moccasins, dolls and ceremonial masks. 48. What is the main subject of this talk? 49. According to the speaker, how was most corn prepared? 50. After the farmers ate the corn from the second harvest, what did they do with the husks?

100

\g o 94 d 8 nj^i`kp

4H:X 95 B 5 VN8L<PZ
95 5*

"0
x

19.1 PART A

1. Laura used to drive a red car. 2. If only I paid attention in class. 3. Beth nished her letter, right? 4. My dog loves to go out in the rain, but my cat hates it. 5. The boy ran down the hall, too shy to stop and talk. 6. How come Michael's always growling sth.? 7. I can't really see myself in that job. 8. Diane is going to practice more and do better at the next swimming meet. 9. Because he brought the wrong documents, my cousin couldn't apply for his driver's license. 10. Larry had an electrician repair his stereo. 11. You know what is the cost that school is? 12. Whatever persist you to do that? 13. Why didn't I think to borrow the workbook, too? 14. These calculus problems are not only tough, they take forever to do. 15. I can't leave a project un nished. 16. I should take care of that pile of mail on my desk. 17. The cashier tagged a surcharge onto the bill. 18. Just look at that new dormitory. The modern architecture seems so out of place among all the older buildings. 19. Severe weather conditions have been forecasted follow these mild days. 20. To think I was once interested in becoming a veterinarian.
x

19.2 PART B

21. M: Do you realize it's almost mid-night. W: I can't believe it. Where is the time gone? Q: What does the woman imply? 101

102

\g_o 95 d 5 nj^i`kp

22. M: Leaving the party so soon? We're just getting ready to out the cake. W: Sorry, I can't stick around. Q: What does the woman mean? 23. M: Remember Carl's wedding is June twenty-second. W: Thanks for reminding me. I thought is wasn't until sometime in July. Q: What does the man mean? 24. W: I lived with my sister this summer and didn't have to pay rent. So I was able to save most of my salary. M: Not me. The more I earned, the more I spent. Q: What does the man mean? 25. M: Is Jame coming back right after the holiday? W: No. I don't expect to see him until next week. Q: What does the woman say about Jame? 26. W: Is it all right if I ask you a personal question? M: You can ask. I don't promise to answer. Q: What does the man mean? 27. M: I hope I don't oversleep, I've simply got to catch the rst ight out. W: If I were you, I'd request the wake-up call from the front desk. Q: What does the woman suggest the man do? 28. W: I've noticed that you get letters from Canada from time to time. Would you mind saving the stamps for me. My sister collects them. M: My roommates already asked for them. Q: What does the man imply? 29. M: Has tomorrow's meeting been con rmed? W: Ahh, I was told it's been called o . Q: What does the woman imply? 30. W: Are you join the chess club this year? M: Chess eats up too much of my time. Q: What can be inferred about the man? 31. M: I'm never going to trust the restaurant critic from that magazine again! That didn't taste anything like the meal we had in chinatown. W: It de nitely wasn't worth the wait.

19.3 PART C

103

Q: What does the woman imply? 32. W: Should I help to clean up by throwing away these newspapers. M: I want to clip a couple of articles rst. Q: What does the man mean? 33. W: Nancy'll pleased about Bill's promotion. M: OH, I heard she wanted that job herself. Q: What does the man imply? 34. M: I've nally found a research topic for our sociology class. W: Really? I'm going to be up all right trying to come up with something by tomorrow. Q: What does the woman mean? 35. W: I wish I knew what kind of tree that was, growing in front of the apartment building. M: I know, it's tall like an oak, but the leaves are in di erent shape. Q: What can be inferred about the tree?
x

19.3 PART C

36-39 listen to a conversation between two friends. W: How are you new neighbors, Mancy? M: They seem nice enough, but they have a son who's driving me crazy. W: What do you mean? M: He comes home every night around 10 with his car windows rolled down and radio blaring. It stops as soon as he turns the car o . But by then Brian and Lisa are wide awake. W: OH, no. M: Oh, Yes. Sometimes it takes us ll mid-night just to get them settle down again. W: Have you tried talking to them? M: We haven't even really met them yet except to say a quick hello. I hate to get o on our own foot. W: You are not going to like them when you do meet them if you keep on simmering. M: I know, but I feel stupid complaining. It's not as though he blasting his stereo all night. W: You said yourself it is driving you crazy. M: Well, you know how early I have to get up to be here at the o ce. I'm just not getting enough sleep and neither are the kids. They're so irritable when I get home in the afternoon.

104

\g_o 95 d 5 nj^i`kp

W: Maybe you could go over sometime with a little gift: a plant for the yard or something. Then you could ask about their son whether they have any other children and they'll be sure to ask about yours. M: Yeah, and then what? W: Then you could mention that the hardest thing at this stage is getting your kids to get sleep at night? M: And keeping them in the sleep. W: That's the idea. And you should do it soon. The longer you wait, the harder it'll be to do politely. 36. What bothers Nancy about her neighbor's son? 37. When does the noise stop? 38. Why is Nancy reluctant to speak to her neighbors about the problem? 39. What suggestion does Nancy's friend make? 40-42 listen to a lecture being given to a group of dieters. In addition to exercising regularly, eating a good breakfast is considered by many health experts to be a signi cant part of a successful way of reduction plan. After a night of fasting the body needs to be replenished with nutrient in order to maintain a high level of energy throughout the day. Several nutritionists suggest that dieters should eat up to one third of their daily calories at breakfast time. These calories should be in the form of balanced meal of nutritious food. A balanced meal consists of ve key elements: Proteins, Carbonhydrats, Fats Vitamins and Minerals, people who're dieting need a variety of foods to assure a constant supply of nutrients their bodies need. If a well-balanced diet is not followed, the dieter may become sleepy, nervous, or feel out of sorts. If a poor diet is maintained for a long period of time, the dieters may become vitamin de ciet vitamin de ciency can cause serious health problem. For these reasons, many doctors recommend that everyone, especially those on diet, starts the day with up to one-third of the daily requirement of nutrients in the form of a well-balanced breakfast. 40. What is the main subject of the talk? 41. According to the speaker, how much of a day's total calories should be consumed at breakfast? 42. According to the speaker, what is needed to make certain that the body has a constant supply of nutrients? 43-46 listen to a conversation about weather for recasting. W: We'd better be looking for sales on down jackets and thermos underwear. It's going to be really cold this winter.

19.3 PART C

105

M: Well, you didn't expect it to be warm, did you? W: Of cause not, but I just heard the national weather services prediction for the next 90 days. They said it's supposed to be much colder weather than usual. M: Yeah, maybe. Personally, I think those long-range forecasts are useless. W: Not as useless as you think. M: Oh, come on, when you're talking about what's going to happen three months later, you might as well just pick a forecast out of our hat. W: Well, you are half way right. They aren't very good for precipitation. But they are a lot better for temperatures especially for this time of the year. M: Really, so I should take them seriously about the cold but not count too much on a lot of extra snow? W: Ahha. M: But what did you say about this time of the year. Do some seasons really give them more problems in making forecast. W: Autumn is the worse, apparently weather patterns change so much then, just think how variable our weather has been the last three month. M: Come to think of it. That's true. It probably would have been hard to predict all those changes back in the summer. You know, you're beginning to convince me there is more to forecasting than I thought. How come you know so much about it? W: I get my information from an expert. My sister's a meteorologist. 43. On what aspect of weather forecasting does the conversation focus? 44. According to the woman, what do 90-day forecast predict fairly accurately? 45. Why is it di cult to predict the weather far in advance for Autumn? 46. What is the man's attitude toward forecasting at the end of the conversation? 47-50 listen to a talk by a campus guide. Good morning, my name is Marsha and I'm a senior in the Math Department here at Madison. Today Greg and I'll be taking you around to a few places you'll be visiting a lot,. Before we start, though. Does everyone have a copy of a map of the campus? You should really keep this with you for the rst couple of weeks, at least until you'll become familiar with the buildings and landmarks. You sure wouldn't want to miss any classes this early in the term. Today we will visit some of the most important places on the campus. First, we'll go to the register's o ce where you'll sign up for your classes. So if you haven't seen your advisor yet, you can wait in the commons outside while the others get their class schedules. You have to come back here on your own later. Next, we'll go to the library. I think you'll be pretty impressed with the

106

\g_o 95 d 5 nj^i`kp

real books collection, and nally. We'll go to the cafeteria where you'll probably spend a lot of time socializing at least until your rst exams are scheduled. The number of the room for your classes will be on the schedule you'll get from the register, and it will be up to you to nd those rooms yourselves. So, if there are no questions we'll start the tour now. 47. What is the speaker's job? 48. What does the speaker suggest the listeners keep with them? 49. Where does the speaker say the listener will spend a lot of time? 50. According to the speaker, what must the listeners do on their own?

46HX 95 B 8 VN8L<PZ
95 8*

"0
x

20.1 PART A

1. Do you want to go on a trip with us to Florida this spring? It will cost about $ 300 a person. What can be inferred about the man? 2. My watch stopped again. And I just got a new battery. Why don't you take it to Smith's Jewelry. They can check it for you. And they're pretty reasonable. What does the man mean? 3. We're going to change our meeting from Monday to Tuesday. It's all the same to me. What does the man mean? 4. We plan to go to the beach after class. Want to come? I'd love to. But Prof. Jones want to speak with me. What will the woman probably do? 5. Janet sounded worried about her grades. But she's getting As & Bs, isn't she? What does the man imply about Janet? 6. You look great since you've been taking those exercises classes. Thanks. I've never felt better in my life. What does the man imply? 7. I had a hard time getting through this novel. I know how you feel. Who can remember the names of 35 di erent characters. What does the woman imply? 8. That's a long line. Do you think there'll be any tickets left? I doubt it. Guess we'll wind up going to the second show. What does the woman mean? 9. This course in much too hard for me. 107

108

\]go 95 d 8 nj^i`kp

Sorry you decided to take it, huh? What does the man ask the woman? 10. Are you going home for winter vacation? I'd agreed to stay on here as a research assistant. What can be inferred about the woman? 11. Hello! Hello! This is Dr, Grey's o ce. We're calling to remind you of your 4:15 appointment for your annual checkup tomorrow. Oh, thanks. It's a good thing that you called. I thought it was 4:15 today. What does the man mean? 12. How wonderful you won the scholarship. Can you believe it? No. It's almost too good to be true. What does the man mean? 13. Excuse me. Prof. Davidson. But I was hoping to talk to you about my class project for economics. I have a class in a few minutes. Why don't you come to see me during o ce hours tomorrow? When will the woman discuss her project with Prof. Davidson? 14. How are you feeling? The stu the nurse gave me seemed to have helped. But it's making me awfully drowsy. What does the woman mean? 15. Bill Smith has volunteered to write a summary of the proposals we've agreed on. Will I have a chance to review it? What does the woman want to know? 16. Why don't you wear that yellow shirt that your sister gave you for your birthday. I love that shirt. But it's missing two buttons. What does the man mean? 17. How many classes do you have today? Just one. From 3 till 6. What does the man mean? 18. Our football team didn't play very well. That's true. But at least we won the game. What does the man mean?

20.1 PART A

109

19. This has been an unusually cool summer. Uh huh! I actually had to get out my wool sweaters in August. What does the woman imply? 20. I got some bad news today. The store where I work in laying o sta . Are they going to let you go? What does the woman want to know? 21. I'd like to pick this lm up by 4 tomorrow afternoon. I can have it for you at 2 if you like. What does the woman say about the lm? 22. I talked to Philip today and he said he'd be coming to the party. Oh, so he can come after all. What can be inferred about Philip? 23. Gary insists on buying the food for the picnic. That's pretty generous. But shouldn't we at least o er to share the expense? What does the woman suggest they do? 24. How's the new job going? Well. I'm getting used to lots of new things. But I wish the supervisor would give me some feedback. What does the woman mean? 25. Did Linda ever nish that introductory chapter? I'm not sure. She's spent hours on end rewriting it. What does the man imply about Linda? 26. The supermarket down the street is selling everything half price because they are going out of business. Sounds like an ideal time to stock up on co ee. What does the man mean? 27. Have you heard anything about the new professor? Just that she's no pushover. What does the man say about the professor? 28. I need to get a copy of my birth certi cate. Sorry. But we can only accept requests by mail now. What does the woman mean?

110

\]go 95 d 8 nj^i`kp

29. When is the earliest ight from Washington to New York? There's a shuttle at six. And if that's full, there's another at 7. What does the man mean? 30. How do you like to help me plan the refreshments for the astronomy club meeting tomorrow night? Sure. Let's be careful not to overdo it though. Last time we had enough for 3 clubs put together. What does the woman mean?
x

20.2 PART B

Question 31-33 David, can I give you a hand with one of those grocery bags? Sure, Nanny. Could you take this one please? I didn't realize how heavy these bags would be. Why did you buy so much stu when you have to walk back home from the store? Well, I didn't intend to buy a lot. But I'm having some people over and I guess I needed more than I expected. What's the occasion? Now the people I live with, the Kremers, have been on vacation for a month and I thought I'd surprise them. I'm inviting some of their friends and families for a welcome home dinner. Oh, that's really thoughtful of you. I gure it's the least I can do for them. They've been letting me stay with them rent free while I'm in school. Really? That's pretty generous of them. Well, they understand how di cult it is to make ends meet when you're a student. They've been such a big help to me. I thought that this might be a small way to thank them for the generosity. 31. What is David trying to do? 32. Why did David think he wouldn't have a problem? 33. Why is David appreciative of the Kremers? Question 34-37 Wonderful I spent most of my time at the art museum. I especially liked the new wing. I was amazed to hear the guide explain the problems they had building it. Right. I just read an article that went on & on about the cost. 90 million total I think.

20.2 PART B

111

Yeah. The guide mentioned that. You could see they spared no expense. Hm. It looked really unusual, at least from what I saw in the picture. It is. The basic design is two triangles. In fact there are triangles all over. The paving stones in the courtyard, the skylights and even a lot of the sculptures. One sculpture is a mobile. It's in the courtyard and it's made of pieces of aluminum that moves slowly in the air. It's really impressive. That was in the article too. It said that the original was steel and it weighed so much that it wasn't safe to hand. Right. They did it over in aluminum so it wouldn't come crashing down on someone's head. You know the article went into that in detail. There was even an interview with the sculptor. I'd like to read that. Would you mind if I borrow the magazine sometime? No. I wouldn't mind if I haven't thrown it out yet. 34. What did the woman think of the new wing of the museum? 35. How had the man learned about the museum? 36. According to the woman, what do the paving stones, skylights and mobile have in common? 37. What was the problem with the original mobile? Questions 38 to 41 In the few minutes that remain of today's class. I'd like to discuss next week's schedule with you because I'm presenting a paper at a conference in Detroit on Thursday, I won't be here for either Wednesday's or Friday's class. I will however be here for Monday's. Next Friday, a week from today, is the midterm exam, marking the half way point in the semester. Prof. Andrews has agreed to administer the exam. In place of the usual Wednesday's class, I've arranged an optional review session. Since it is optional, attendance will not be taken. However attending the class would be a good idea for those worried about the midterm. So remember: Optional class next Wednesday Midterm, Friday. 38. What is the purpose of the talk? 39. At what point during the semester does the talk take place? 40. What did Prof. Andrews agree to do? 41. What will occur at next Wednesday's class time? Question 42 to 46 Today's lecture we'll center on the prehistoric people of Nevada Desert. Now most of these prehistoric desert people moved across the countryside throughout the year. You might think that they're wandering aimlessly. Far from it, they actually followed the series of carefully planned moves.

112

\]go 95 d 8 nj^i`kp

Where they moved depended on where food was available. Places where plants were ripening or sh were spawning. Now often when these people moved, they carried all their possessions on their backs. But if the journey was long, extra food and tools were sometimes stored in caves or beneath rocks. One of these caves is now an exciting archaeological site. Beyond its small opening is a huge underground grotto. Even though the cave is very large, it was certainly two dark and dusty for the crawlers to live in. But it was a great place to hide things. And tremendous amounts of food supplies and artifacts have been found there. The food includes dried sh seeds and nuts. The artifacts include stone spear points and knives. The spar points are actually rather small. Here is a picture of some that were found. You can see their size in relation to the hands holding them. 42. What is the main subject of this talk? 43. What point does the speaker make about the prehistoric people of the Nevada Desert? 44. Why didn't the people live in the cave described by the speaker? 45. What have archaeologists found in the cave? 46. Why does the speaker show a photo to the class? Question 47 to 50 To us, the environment in which sh dwell often seems cold, dark and mysterious. But there are advantages to living in water. And they've played an important role in making sh what they are. One is that water isn't subject to sudden temperature changes. Therefore it makes an excellent habitat for a cold blooded animal. Another advantage is the water's ability to easily support body weight. Protoplasm has approximately the same density as water. So a sh in water is almost weightless. This weightlessness in turn means two things: 1) A sh can get along with a light weight and a simple bone structure. And 2) Limitations to a sh's size are practically removed. Yet there is one basic di culty to living in water the fact that it is incompressible. For a sh to move through water, it must actually shove it aside. Most can do this by wiggling back and forth in snakelike motion. The sh pushes water aside by the forward motion of its head and with a curve of its body and its exible tall. Next the water ows back along the sh's narrowing size, closing in at the tall and helping the sh propel itself forward. The fact that water is incompressible has literally shaped the development of sh. A at and angular shape can be moved through water only with di culty. And for this reason, sh have a basic shape that is beautifully adapted to deal with this peculiarity. 47. What is the talk mainly about? 48. What does the speaker mention as a problem that water presents to sh?

20.2 PART B

113

49. The speaker compares a sh's movement with that of what creature? 50. What aspect of a sh does the speaker discuss in the most detail?

114

\]go 95 d 8 nj^i`kp

46HTX 95 B 10 VN8L<PZ
95 10 *

"0
x

21.1 PART A

1. How is your paper coming along? My typewriter is broken. What does the man mean? 2. Have you tried Susan's apple-pie? I got the last piece. And it was out of this world. What does the woman mean? 3. If you're staying late, will you be sure to look up the o ce when you leave? Oh. I hope I won't be more than an hour. What does the man mean? 4. Nancy really wants to ski on Thursday. Yes, but she can't. Can she? What does the man say about Nancy? 5. Let me help you with those packages. Thanks. But it is only 3 quarters of a block. What does the woman tell the man? 6. Did you know that Arthur has three brothers living on three di erent continents? He must get in incredible phone bill every month. What does the woman imply about Arthur? 7. This calculator isn't working right. I think you've got the battery in upside down. What does the woman mean? 8. We should buy a good guide book and study it before our trip to Montreal. We could. But they're so overpriced. What about the library? What does the man imply? 9. I always want a little something to eat about this time of day. So do i. Let's have a snack now and then have a light lunch later. 115

116

\]gmo 95 d 10 nj^i`kp

What are the speakers discussing? 10. I've just been over to my friend Tom's new apartment. It's much bigger than my place. But more expensive I bet. What does the woman mean? 11. Could you mail these letters for me please? More letters? Your friends are going to be very happy to hear from you. What does the man imply about the woman? 12. Does Prof Ford always come to class? Is ice cold? What does the man imply about Prof Ford? 13. Would you have some time this week to go over these questions with me? How does tomorrow sound? What does the man mean? 14. Hey? John! John! Save your breath. He's out of earshot. What does the woman mean? 15. You only have water to serve your guests? This isn't just water. This is imported mineral water. What does the woman imply? 16. I see a new bookstore has just opened on Main street/ It may be a new store. But the books are far from new. What does the man mean? 17. Alice has been spending a lot of time at the library lately. Well. She's got a paper due and two nal exams next week. What has Alice probably been doing? 18. It's going to cost a fortune to get my car xed. Why don't you just trade it in for a new one. What does the man suggest the woman do? 19. Winter is over at last. Time to pack up my gloves and boots. I've been waiting for this for months. What does the woman mean?

21.1 PART A

117

20. How did the game go the other night? Did your team with? Are you kidding? That would be a rst. What does the man imply? 21. Danis told us he likes to play cards. But we've invited him three times and he hasn't come once. What can be inferred about Danis? 22. I invited my class home for co ee. In this tiny place? What does the man imply? 23. Excuse me, I don't understand why ght 213 has been delayed. The weather seems ne now. I'm afraid New York got three times as much snow as we did here. It should be clear by morning though. What does the woman imply? 24. Take a look at this gift catalog. Maybe we can nd something to get Janet for her new house. OK. But remember we can't a ord a lot. What does the woman mean? 25. Are you still waiting for Bob? I don't know why I bother. The store will be closed by the time we got there. What does the woman mean? 26. Would you like to come with us for co ee a little later? I'm o ca eine medical restriction. What does the woman mean? 27. Mary seems surprised that she got a research grant. Well, she should have been, everybody knows that she's brilliant. What does the man mean? 28. Mind if I leave my umbrella here in the hallway? Not at all. But rst shake it o outside, would you? What does the woman tell the man? 29. According to this article, the former boxing champ is going to try again. He's coming out of the retirement for the third time.

118

\]gmo 95 d 10 nj^i`kp

Is a comeback at this age at all likely? What does the man imply about the former box champion? 30. Don't you think Prof. Morrison's test was too di cult? Well, I must admit I had been expecting more than just passing grade in biology. What does the man mean?
x

21.2 PART B

31-34 have you made any plans to go away during semester break? I've been thinking of skiing. I really haven't had time to think about my vacation. I've been concentrating on getting ready for my exams, especially Philosophy. But I'll probably go to the beach. Why the beach? Well. It would be mice to get away from this cold weather and just lie ine the sun and relax after working so hard. It's true that skiing does require work. And you have to get up early and wait in long lines for the chair lifts. Thanks. I think you help me make up my mind. Sure. Now maybe you can use your mind to think about something else, like your studies. 31. With whom is the man speaking? 32. What has the woman mainly been thinking about? 33. Where will the woman probably spend her vacation time? 34. What does the woman think the man should do? 35-38 Hello, Jim. I haven't seen you in a while. What seem to be the problem? Actually I'm a little embarrassed about coming here. I feel ne right now. but you know how much stu is going around. Andy way every year around the holidays like clockwork I come down with something. So you're interested in prevention. What symptoms do you usually get? You know, cough, fever, runny nose, my head and bones ache, chills even. I'm usually miserable for a week and it ends up ruining my holidays. Sounds like a typical u to me. As you said, lots of people have it in uence often strikes when people are over tried stressed out and not eating nutritious food. And also you increase your exposure to a virus when you're in big crowds where lots of people are coughing and sneezing.

21.3 PART C

119

I certainly spend a lot time in department stores around the holidays buying gifts for people. Yes. And so you increase your exposure to airborne viruses just when your body's resistance is already low from all the running around you do. So what can I do to ward o the u? Actually it's fairly simple. Get a lot of rest, eat well. That way your immune system will be boosted. And you'll be more able to ght o illness. All these things make sense. But one more question. Aren't I bound to get sick anyway if there's an outbreak in the dorm? Oh, you didn't mention you lived in the dormitory. In that case I'd also suggest you get immunized. The vaccine available prevents' the three main types of in uenza. Why don't you go to the university health center. The shots are free there. I'll do it right away. It will be nice to feel well during. The holidays for once. 35. Why did the man go to see his doctor? 36. How does the man describe his health problem? 37. What might be a reason the man gets ill? 38. Why does the doctor suggest the man go to the university health center?
x

21.3 PART C

39 to 42 A lecture in a architectural design course. Dozens of valuable paintings and frescoes were badly damaged today when rain water poured through a roof at the state museum. As freak rainstorms lashed in the central and southern sections of the state, part of the museum's roof which was scheduled to be repaired collapsed and water cascaded into two storerooms. The museum director said that several well known pieces, including oils, watercolors and frescoes, had been damaged, although only one objects, a 19 century fresco was damaged beyond repair. She added that inspection for damages has not been completed and therefore she could not estimate costs of the repair and replacement. 39. What part of the museum ooded? 40. What was damaged? 41. What can not be restored? 42. What did the director say about the cost of damages? 43 to 46 welcome to the New Bedford institute's, series of lectures on the mysteries of the sea. You may have noticed the drawing on the cover of the program for today's presentation. It depicts a

120

\]gmo 95 d 10 nj^i`kp

sea creature that scientists have been interested in for many years. It is said that in the 19th century a group of shermen were surprised by seeing a huge squid that they said was as big as a house and had enormous tentacles. The shermen were frightened out of their wits according to reports from that time. The creature sank back into the ocean and was never seen again. Marine biologists believed that this species of giant squid called archituth is still exits. And is comes to represent how little we know about sea creatures. Compared with what we know about the animals on the land. We do know that there are many more di erent kinds of marine species than there are land species. But we just don't have the technology yet to do sustained research. Even short unmanned trips are so hard to accomplish. In fact researching in the deep oceans has been compared to ying an airplane overland throwing down a net and seeing what you get. In other words it's very hard to picture the whole situation when that's the way you have to get your samples. Most of what is known about deep sea creatures, which as I said is actually very little, comes from ocean beds that dried up long ago. We're fortunate to have a few samples of these fossil remains on display on the second oor. We'll take a look at them after a while. 43. Why were the shermen frightened of the squid? 44. What does the giant squid symbolize to biologists? 45. What does the airplane example illustrate about the deep sea research? 46. Where has most of the information about rare sea creatures been found? 47-50 With the introduction of radio, newspaper publishers wondered how broadcasting would a ect them. Many feared that radio as a quick and easy means of keeping people informed would displace the newspaper industry altogether. Others hoped that the brief newscast heard on the air would stimulate listeners interested in the story so they'd buy the paper to get more information. This second idea turned out to be closer to the truth. Radio & print were not substitutes for each other but actually supported each other. You see the relation ship between di erent media is not always one of displacement but can be one of reinforcement. However this is not always the case. Take television & motion pictures for example, with the popularization of TV, the motion picture industry su ered greatly. Movie attendance dropped when audience members chose to stay at home and be entertained. Likewise. When a football game was shown on the air, the stands were often empty because fans chose to watch the game at home. 47. What is the main topic of this talk? 48. According to the speaker, what is the relationship between radio & the newspaper industry? 49. According to the speaker, how did the introduction of television a ect motion pictures?

21.3 PART C

121

50. Why does the speaker mention a football game?

122

\]gmo 95 d 10 nj^i`kp

46H6X 96 B 1 VN8L<PZ
96 1*

"0
x

22.1 PART A

1. It's such a nice day. Why don't we have lunch outside? OK. Bur let's nd someplace that's not too noisy. What does the man mean? 2. Look at the time. I'm going to miss my bus. Don't worry. I'll drive you to the stop. And if the us has already left I can get you to your apartment. What does the woman mean? 3. Can you recommend a hotel in New York? Well, I can think of several. What's your budget like? What does the man imply? 4. Any messages for me? Someone did call. But there was so much static I couldn't make out what he was saying. What does the woman mean? 5. Didn't George give a great speech? Are you serious? What can be inferred about the woman? 6. Can I open a checking account here? I'm sorry. You'll have to step over to the manager's desk. What does the woman imply? 7. Now that I've nished my exams. I'm going to relax and go to a movie tonight. Lucky you! I've still got two nals to take. What can be inferred about the woman? 8. It's very nice of you to give me your tickets for the play. Please. Don't mention it. I'm going to be out of town this weekend anyway. What can be inferred about the man? 9. I almost forgot. I still owe you ten dollars from the other night. 123

124

\]g]o 96 d 1 nj^i`kp

Do you have change for a twenty? Oh{. Not at the moment. Let's just straighten it out some other time. What does the woman mean? 10. Hey! What happened to all that food we bought? The refrigerator is empty. My little brother with a big appetite was here. He really combed these out. What does the man mean? 11. I'm taking Prof. Bam's course next semester. Anything I need to know about it? If I were you. I'll take careful notes. Her exams are based on her lectures. What advice does the man give the woman? 12. I'm glad you nally decided to go skiing with us next week. I still have to get my supervisor to agree to it. What does the woman imply? 13. Did you see the college newspaper? They did a story on our voter registration campaign. I did. Maybe it'll spark some interest on campus. Without more volunteers we'll never meet our goal. What does the man mean? 14. Have you net the new sales manager yet? We've been introduced about three times, he seems a little forgetful. What can be inferred about the sales manager? 15. Excuse me, but do you happen to have some change for the paring meter? No. But if you go into the restaurant you'll probably be able to change a dollar bill. What does the woman want to do? 16. I am going to tell that neighbor of mine to turn down that music once and for all. I see why you are angry. But I've always found that the polite route is the most e ective. What does the man mean? 17. I love your new sofa. But why don't you put it over there under the window? Oh, but the plants are doing well on the table there. What does the man imply? 18. At the rate of its being used, the copier is not going to make it through the rest of the year. The year? It's supposed to be good for ve. What does the woman say about the copier? 19. I've been invited to a dinner party at Janet's. Do you think should bring something?

22.1 PART A

125

You could pick up a cake. Chocolate is her favorite. What does the woman suggest the man do? 20. I'm sorry I missed your soccer game. But I had the u. Don't worry about it. We couldn't have played worse. What does the woman mean? 21. Do you feel like watching the evening news? You ought to stay away from me. I have a bad could and wouldn't want you to catch it. What does the woman mean?. 22. Hew! It's a real scorcher today. And the forecasters are saying there is no end in sight. What does the man imply? 23. Uh-uh. Look I'm going to be a little late for class. I hope Prof. Clark does start on time today. Are you kidding? You can set your watch by the start of his class. What can be inferred about Prof. Clark? 24. Are you keeping count on the news from home since you've been here? I've been getting weekly updates. What does the man mean? 25. Well, what did you think of the theater director? You mean Emily Thompson? She was away attending a conference. What does the man mean? 26. That movie was awful. And yet it got such great reviews. It was hardly worth the price of admission. What does the man mean? 27. I'm just looking for bike that will me get to the library and back. With the roll of the way they are you'll need a sturdy one. What does the woman imply? 28. I'll take this suit. It ts me really well. And while I amend it I like the shirt and the tie too. We have some nice socks that match. What does the woman mean? 29. Sorry I didn't see you on the news. Yeah. And it's not likely they are going to rebroadcast it anytime soon.

126

\]g]o 96 d 1 nj^i`kp

What does the man imply? 30. The science exhibit is opening today. You are coming with me this afternoon, aren't you? I have too much work to do for tomorrow. What does the man mean?
x

22.2 PART B

31-34 Conversation between a student and guidance counselor *Hello, John. You must be pleased. After all how many students are lucky enough to have been accepted at their rst and their second choices? *Not many I know. But I'm not sure yet which one to choose. *Well, you seem to have doubts about the state university. But its biology department has a ne reputation. What more could a biology major want? *Yeah. And they also have internships for seniors. But a friend told me that for the rst two years some lectures have a hundred and fty students. You probably wouldn't get to know any of your teachers. *Well, you might actually. Because those classes also have small discussion sections. Twice a week, and have no more than twenty students. *I know. But I've heard that they are usually taught by graduate students. At White Stone College all classes are taught by professors. *What about Sating? Do you prefer a small town like White Stone? Or a bigger place like the state capital? *That doesn't matter to me. What I do care about is getting individual attention from the faculty than making friends. *Look, I've known you for four years now and you seen to be a pretty outgoing person. I don't think you have any trouble making friends at the state university. It sounds to me that you are learning toward White Stone though? *I am. The only problem is that the White Stone's tuition is really high and I'm not sure I can a ord it. *You could still apply for a student loan or sign up for a work-study program. *Yeah. I think I'll look into that. 31. Why did the man go to see the woman? 32. What concern does the man have about the state university? 33. What does the man consider to be an advantage of the White Stone College?

22.2 PART B

127

34. What is the man likely to do in the near future? 35-39 Conversation between two students. *Hi, Claire. How does it feel to be back on campus? *Keach, hi. Well, to tell you the truth, I have mixed feelings. *Oh, why? *I have this great summer job that I really hated to leave. I worked at the wild life research center in Maryland. *That makes sense for a genetic major. What did you do? Clean the cages? *This is a wild life center, not a zoo. This place breeds endangered species and tries to prepare them for life in the wild. *You mean the endangered species like the tiger and the panda? *Well, endangered species, yes. But not tigers or pandas. I was working with whooping cranes and sandtail cranes. It was really neat. I taught the baby crane how to eat and drink, and I help the vet to give medical check-ups. *I can see why it was hard to leave that job. But how did you teach a bird how to eat and drink? *We covered ourselves up with clothes and used puppets made out of stu ed cranes to show the baby chicks what to do. Then the chicks copied what the puppets did. *Clothes? Puppets? Sounds like fun. *It was. The clothes and puppets are the key tool of this. We all covered ourselves up, the scientists, the vets, the genial stock, everybody. You see, baby cranes will become attached to their care-takers. *So if the care-taker is a person, the crane will stay in places where people are. *Yeah. And their chances for survival aren't very good. But by covering ourselves and by using birds puppets the chicks are more likely to seek out other birds rather than people. And their transition to the wild has a better chance of being successful. *A chance of being successful? Hasn't this been done before? *It's been done with sandtail cranes and everyone is optimistic about its work with whooping cranes too. *Yeah. It's exciting, isn't that? 35. Why does the woman say she has a mixed feelings? 36. What was the woman's job? 37. What is the goal of the wild life center? 38. Why does the man mention the tigers and pandas? 39. Why do the sta members cover themselves with clothes as they work?

128
x

\]g]o 96 d 1 nj^i`kp

22.3 PART C

40 to 42 A lecture in a architectural design course. In 1871 the rst passenger elevators were used in o ce buildings and allowed architects to build higher than people could comfortably walk. Another innovation was in building technique. In 1885 the steel skeleton was introduced and allowed for the construction of tall building that could withstand high winds. We take for granted some of the other inventions that enabled people to live and work in skyscrapers. For example, few people realized that the telephone was necessary for vertical communication and that ush toilet and vacuum incinerator made waste disposal possible. Now as we entered the age of super skyscraper, some with more than 200 oors, we see the need for even more technological innovations. In the area of heating and cooling systems for example. For all their bene ts, these super tall building do cause problems though. For one thing they place enormous train on parking and tra c row in urban areas. But let's leave behind these technical concerns and move on to consider some of the design elements that have come to characterize the age of the skyscraper. 40. What does the professor mainly discuss? 41. What does the professor say about the invention of the elevator? 42. What will the professor probably discuss next? 43 to 46 Talk in an introductory biology class. Good morning. Let me welcome all the new members of the ski patrol. My name is Brenda Peters and I've been a ski-patroller for nearly three years. After working in o ces and restaurants I nd this job very satisfying. After all, if you love the outdoors, it's a great job. You won't get rich but you' get in great shape and you will be able to help people everyday. And for me the best part of the job is giving ski lessons I've made a lot of friends that way. Of course your main responsibilities will involve ensuring the safety of everyone who skis here. All trails have to be checked daily to make sure they are groomed probably. You also will have to be watching out constantly for people who aren't skiing safely, or who may think they are better skiers than they really are. And to help those who are injured, you'll have to know basic rst aid. Tomorrow you will begin training in a rst aid system that skiers speci cally to the outdoors. We pride ourselves in our ability to get people o the mountain quickly and safely. OK, so you are free to explore the slopes for the test of the day. All in all, I think that being a ski patroller is great job I hope you will all feel the same way. 43. What does the speaker mainly discuss? 44. According to the speaker, what is one advantage of working on a ski patrol? 45. What does the speaker say she likes most about giving ski lessons?

22.3 PART C

129

46. According to the speaker, what is one of the man responsibilities of ski patroller? 47-50 A lecture given in a geology class. If you ew over certain parts of Nebraska and Texas by plane, you might notice some large areas appearing as bright green circles many hundreds of feet across. This green is unusual in the high plains area where the climate is very dry. These green patches are the result of a new technique for mining the underground water. In this technique, miners bore deep holes in the ground until they reach a special geological formation called ocheropher. The water which has collected in this ocherophers for hundreds of years in called fossil water or ground water. It pumped up through the bored hole and spray over the land to irrigate the crops. Raising crops such as cotton and wheat, water in this way creates the fertile green areas that contrasts vividly with the natural brown of the plains. Crop yields have increase dramatically. However they've created a serious environmental problem. The problem is that the water is being removed from many ocherophers faster than it can be replenished naturally. Ground water levels have dropped rapidly and it's becoming more di cult and expensive to get this water. In some parts of Texas, water levels in some of the ocherophers have declined cess of using water faster than it can be replaced is wide-spread and serious. 47. What does the speaker mainly discuss? 48. What bene ts have resulted from the technique? 49. What is happening to ground water? 50. What does the speaker imply about ground water?

130

\]g]o 96 d 1 nj^i`kp

46HFX 96 B 5 VN8L<PZ
96 5*

"0
x

23.1 PART A

1. Would you like some milk in your co ee? Please. What does the woman mean? 2. Could you help bother me plan a surprise party for Meg? Sure. What can I do? What does the woman mean? 3. Let's jog for another mile. I'll try. But I'm running out of steam. What does the man mean? 4. Should we call Marsha and tell her about the meeting? I'm not sure. It's up to you. What does the man imply? 5. I can't seem to solve this problem. Neither can I. What does the woman say about the problem? 6. Hello. I'm calling to see of the summer position you advertised in the paper. Is it still available? Uh, yes. Certainly. When could you come to the o ce for an interview? What will the man probably do? 7. It took me ve days to drive down to Florida. Five days? I could've walked there in less time. What does the woman mean? 8. Excuse me, did anybody nd a black umbrella after the last show? I left it under my chair. As a matter of fact, we did. Check it at the ticket counter. That's where we turn it the lost-and-found items. What does the man suggest the woman do? 131

132

\]gfo 96 d 5 nj^i`kp

9. I spent the whole weekend totally absorbed in this biography. And you still haven't nished reading it? What can be inferred about the biography? 10. You haven't phoned Harry yet, have you? As a matter of fact, I have. What does the man mean? 11. I can't seem to nd my photo album. I think Mary is looking at it in the living room. What does the man mean? 12. Did you hear that Mitchell turned down that job? Yeah. The hours were convenient, but she wouldn't have been able to make ends meet. What does the man say about Mitchell? 13. Is there a bus I can get to the station? There is. But you can't rely on it. I can give you a ride if you can wait while I put these things away. What is the woman going to do? 14. I hate memorizing vocabulary. Well, it's part of learning a second language so there not whole at all you can do about it. What does the woman tell the man? 15. Have you had a chance to wear your new shirt yet? That reminds me. I've been meaning to exchange it to a larger size. What does the man imply about the shirt? 16. Did you tell Carl that the concert starts at eight? I've tried several times, but the line's been busy. What does the man mean? 17. I had the brown paper and the string. Could you hand me the tape and the scissors please? Sure. Here they are. But remember all this has to be weighted before it goes to. What is the man probably doing? 18. Andrew likes his new place. But he is not too happy about all the noise. What did he expect? He is right next to the airport. What does the woman mean? 19. I don't remember exactly what the lab hours are. But they are posted on the door.

23.1 PART A

133

I just checked that schedule and it says that the lab opens at ten. But it's all locked up. What can be inferred about the lab? 20. The subway is running behind schedule, and tra c is backed up for blocks. I don't know if we'll make the 7:15 show. It's a beautiful night. Let's try to get there on foot. And if we don't make it. Let's just have dinner near the theater. What does the woman suggest they do? 21. We've been working on this proposal for so long that my eyes are starting to blur. Why don't we get out of here? We can wrap it up later. What does the woman mean?. 22. That was really an interesting piece of music. How did you nd out about it? They played it in the classical station last week. What does the man say about the music? 23. I spent the whole weekend working on that chemistry assignment. Don't tell me. I have to do the whole thing tonight. What does the woman imply? 24. Come to the movies with us. Everyone needs to take a break once in a while. I guess I might as well. I've been studying so long I can hardly concentrate. What does the man mean? 25. Pete's really out of it these days. Yeah. I know. Ever since he met Ann, he's been in another world. What does the woman imply about Pete? 26. Look at all those cars lined up for the ferry. There must be forty ahead of us. Yeah. I think it's going to be a while. What does the woman imply? 27. I thought Pam said the math test wasn't until Monday. Ellen, you should know better than to take Pam's words for anything. What does the man imply about Pam? 28. So far the clubs are about three hundred dollars in the red, and we still have four months to to before membership renewal. Well, we may have to raise our dues. What does the man suggest they do?

134

\]gfo 96 d 5 nj^i`kp

29. I hear they hire two more students to work in the mall room. They are just a little short of a full sta man. What does the woman mean? 30. Fred is o to the golf course again. You would think he was practicing for the championship. What does the woman imply about Fred?
x

23.2 PART B

31-34 Two friends discussing weekend plans. *Susan, I could really use your help this weekend. *What is it, John? Another term paper? *No, no. This is easy compared to that. My cousin is coming on Thursday. She has an interview at the college and I promised my aunt I look after her. We are going to the game on Friday, but Saturday I'm on duty at the library all day and can't get out of it. Uh, I was wondering if you could show her around during the day and maybe we can all meet for dinner later. *Sure. I don't have any plans. What kind of things does she like to do? *Actually I haven't seen her for three years. She lives so far away. But this will be her rst time on a college campus, she is still in high school. So she probably enjoy anything on campus. *Well, there is a music festival in the auditorium. That's a possibility. Only I hope it doesn't show. They are predicting 68 inches for the weekend. Everything will be closed down then. *Well, how about for the time being. I'll plan on dropping her o at your place on the way to work, around eleven. But if there is a blizzard, I'll give you a call and see if we can gure something else out. *Sounds good. Meantime I'll keep Saturday open. We can touch base Friday night when we have a better idea of the forecast. *I hope this works out. I feel kind of responsible. She won't know a way around. And I want her to have a good time. Anyway I really appreciate your help. I owe you one. *No problem I'll talk to you tomorrow. 31. What does John ask Susan to do? 32. What will John do on Saturday? 33. What does John say about his cousin's interests? 34. What can be inferred about John's cousin? 35-37 A conversation in a university cafeteria.

23.3 PART C

135

*I see you are having the sh for lunch. *That's right. Fish sticks and tomato soup, my favorites. *I bet they were frozen. *What? *The sh sticks. *Of course they were frozen. We are hundreds of miles from the ocean. The cafeteria can't a ord to y in fresh sh. *I just mentioned it because an anecdote Professor Chambers told in class this morning. *Which class? *My American social history. It's a lot of fun. *He talked about sh sticks? *Not exactly. Bur he did talk about frozen sh. OK, this is back in 1920, right? It's 20 degrees below zero. And this guy Clarence Birdseye's out ice- shing. *Where is this, Antarctica? *No. Massachusetts I think. Anyway he catches a sh and drops it beside him on the ice and it freezes solid. *So? *So, later at home he thaws out the sh in a bucket of water and it's alive. Of course Birdseye is amazed. But he eventually gures it out that the sh froze so fast that no large ice crystals fomed. *What do you mean? *Look, usually when a plant or animal cell freezes, large ice crystals form inside and eventually tear the cell walls. That's what kills the frozen plant or animal. And also what changes the taste of something that's been frozen. But if you freeze something quickly, only very small ice crystals form. So Birdseye invented the process of quick freezing food. *And that was the beginning of the frozen foods? *Right. 35. Why does the woman tell the man the story? 36. Why is it important to quick freeze fresh food? 37. What was Clarence Birdseye surprised to learn?
x

23.3 PART C

38 to 41 A lecture about the railroad industry.

136

\]gfo 96 d 5 nj^i`kp

At the beginning of the century the railroads were used to haul everything. Powerful railroad barons made fortunes without laving to be accountable to the public or considerable to the customers. But cars and trucks changed all of that. And by 1970, the rail industry was beset with problems. Trucks were taking all the new business. And even so the rail industry remained indi erent to customers. Also many regulations kept the rail industry from adjusting to shifting market. But in 1980, the rail industry entered the modern era when a deregulation bill was passed that allowed railroad companies to make quick adjustments to fees and practices. Companies reduced their lines by 1/3 and used fewer employees. They also took steps to minimize damage to product. And to increase their shipping capacity by stacking freight containers on railroad cars. To accommodate these taller loads, underpasses and tunnels were enlarged. The image of the rail industry has changed dramatically. Today companies are very responsive to customers and are gaining increasing market shares in the shipping industry. The railroad safety record is also strong. Freight trains have an accident rate that is only 1/3 that of the truck industry. Trains also come out ahead of the trucks on environmental grounds because they give o only 1/10 to 1/3 the pollution that is emitted by trucks. And railroading does not were out highways as trucks do. 38. What does the speaker mainly discuss? 39. What development caused a decline in the use of railroads? 40. What is one reason why the railroad industry is gaining public support? 41. According to the speaker, what expense does trucking public support? 42 to 45 Talk in an introductory biology class. This co ee can contains soil from my garden. And I prepared slide samples to show you that it is alive. This ordinary backyard dirt is crawling with microbes. Microbes is not a very speci c term. There are hundreds of thousands of di erent species called microbes. What they have to common is that we can't see them with a naked eye. They are microspopic. Look at this slide. You should see some round cells. Those are yeasts. Yeasts are fermenters. And they are necessary for making bread, beer, yogurt and so forth. Now look for an irregular shape with hairs coming out of it. That's a mold. Molds are decomposers, and they are responsible for the decomposition of my compile teeth. You should also see some protozoa. Some protozoa, like the one causes malaria are harmful to people. A microbic causes disease is called a pathogen. Finally you should see a lot of squiggly lines. They are bacteria. The oldest form of life on earth. Like the other microbes, bacteria are single-cells do not have a nuclear. So I've got about a teaspoon of soil in my hand here. In that teaspoon are about ten thousand protozoa, 200 thousand mold cells, a million yeast, probably a billion bacteria. 42. What is the main purpose of the talk? 43. Why did the speaker take a soil sample from his garden?

23.3 PART C

137

44. What does the speaker imply about yeasts? 45. What does the speaker ask the class to look at during the talk?

138

\]gfo 96 d 5 nj^i`kp

46HJX 96 B 10 VN8L<PZ
96 10 *

"0
x

24.1 PART A

1. I want to play tennis tomorrow but I didn't bring my racket with me this evening. Do you have one I could borrow? What does the man imply? 2. I thought this shirt was a great deal but I washed it once and it shrunk so much that I can't wear it. Some bargain. You should ask for a refund. What does the woman mean? 3. I broke my ankle last Tuesday. And now I have to be on crutches for six weeks. I'm sorry to hear that. Is there anything I can do for you? What does the woman mean? 4. Why didn't you call me last night like you were supposed to? I did your time was busy. What does the man mean? 5. Sue, would you like a sandwich or something? Oh, please don't bother. I can get something later. What does the woman mean? 6. This report is due tomorrow. Would you be able to work on it with me tonight? Unfortunately I have another commitment. What does the woman mean? 7. I wonder where the books I ordered are. I expected to receive the package several days ago. Maybe you'd better check it with the company. They could be temporarily out of stock. What does the man suggest the woman do? 8. I just have to type this last page and then I'm through. By then I'll be done too. What does the woman mean? 9. Did you read the editorial in the paper about the mayor's speech? 139

140

\]gho 96 d 10 nj^i`kp

I sure did. But I think they twisted the meaning of what he said. What does the man say of the editorial? 10. Why do we go to see a movie tonight? A good comedy might cheer you up. I would. But I the reason that I've been so down is all this work I have to do. What can be inferred about the man? 11. I'd like to try to sell some of my textbooks from the last semester. You and a few hundred other people. What does the woman imply? 12. Could I talk to you for a minute about the discrepancy I found in this graph? I'm kind of in the middle of things right now. What does the woman mean? 13. Do you know if George is coming to the meeting? Oh, no. I was supposed to tell that he is sick and can't come. What does the man mean? 14. Dick, please don't tie up the phone. I need to make a call. I'll be o in a minute. What will the man do? 15. Saliy says we should meet her in the park at noon. I thought we were meeting at the library. What do the speakers disagree about? 16. I'm sort of upset with my brother. He hasn't answer either of my letters. Well, just remember how hectic your freshman year was. Give him a chance to get settled. What does the man imply? 17. I wonder what this new avor of ice cream tastes like? I tried it last week. If I were you, I would stick with an old favorite. What can be inferred from the conversation? 18. Pete had hoped to have his apartment painted by this time. But he hasn't started yet, has he? What does the woman imply about Pete? 19. You don't believe in diet, do you? There is nothing wrong with them but they have to combine with exercise to do any good.

24.1 PART A

141

What does the man mean? 20. I'm amazed that you still haven't gotten to know your neighbors. They tend to keep to themselves. What does the man mean? 21. Joe, could you please help me straighten the rug? I'll move it while you lift up this side of the desk. Sure. Oh, just a minute. The lamp is still on it. What will the man probably do next? 22. I had to stand in line at the bank for about half an hour. There were only two tellers open. That's where you and I di er. I would've gone back another time. What does the woman imply? 23. I was touched that our neighbor brought over a dish when we moved in. Yeah. Mrs. Smith really goes out of her way for others. What can be inferred about Mrs. Smith? 24. Where are you up to now? To meet a friend to come in on the subway. What does the woman mean? 25. My company is ying me out to Hawaii on business next week. That's great. Where are they putting you up? What does the man want to know? 26. It's good to see you, Mary. How have you been? Actually I have been feeling under the weather recently. What does the woman mean? 27. It's a really nice apartment. But the owners want two-month rent in advance and I just don't have it. Do you think it would help if they knew what a good tenant you are? You could get your landlord to write them a letter. What does the woman suggest the man do? 28. Good morning, East Coast Data Process. May I help you? Caroline? Oh, dear, I'm sorry, I thought I dialed Jack Easton, your number must be just above his in my address book. What can be inferred from the conversation?

142

\]gho 96 d 10 nj^i`kp

29. There's a nonstop train for Washington leaving here at 2:15. That will be faster than taking the one that leaves at 2:00 and it will give us time to get a bite to eat. What does the woman imply? 30. Sorry about your report. I didn't realize it was in that stack of papers. Don't worry about it. Luckily I saved a copy on my computer. What can be inferred from the conversation?
x

24.2 PART B

31-34 Conversation between two students. * Hi, Bob. How is your oral presentation coming along? * What oral presentation? I don't have to give mine until the end of the next week. * You mean you haven't even started working on it yet? * No. I need the pressure of a deadline to get inspired. * Gee. I'm just the opposite. I can't concentrate unless I know I have plenty of time. Besides I have a big physics test next week so I want to get my presentation out of the way. * What's your presentation on? * William Carlos Williams. * Wasn't he a poet? I thought we were supposed to focus on a short story. * He wrote short stories, do you know? * Really? I never knew that I guess I'll learn more on Tuesday. Is that when you are supposed to talk? * Uh-huh. But I was going to o er you sneak preview. I'm looking for someone to tell me whether I'm talking to fast and whether you can follow what I'm saying. * I guess I could do that. When were you thinking of? * Will tonight be all right? I have a last to go to now but I'll be around after dinner. * That sounds good. I'll stop by your room. It's probably quieter there than in my hall. 31. What do the students mainly discuss? 32. What does the woman want the man to tell her? 33. What will the man probably do after dinner? 34. Where do the students arrange the meet? 35-37 Conversation between two students.

24.3 PART C

143

* Have you ever read anything about pseudo sciences? * You mean fake sciences? Yes. In fact I was just reading some articles about the brain. I have been looking through some of my roommate's science magazines and I came across an article on phrenology. * Phrenology, wasn't that the pseudo science founded by the scientist Franz Gall? * Yes. Gall maintained that people's characters could be determined by the size and the shape of their skulls. For example he though that a pump in a certain place on the head means that the person had the ability of a musician. * Really? I wonder what phrenologists would say about the bumps on my head. Would they say I have the abilities to be a doctor, or a plumber, or a thief. * Well, I'm not sure exactly what the connection is between a person's abilities and the physical characteristics of the head. But although there's no scienti c basis for phrenology, it is true that the bead is the center control for the rest of the body. * I guess you are right. Scientists now know that di erent parts of the brain control di erent parts of the body. * Yes. And I wouldn't be surprised that the scientists one day discover that certain aspect of phrenology has scienti c application. 35. What is the source of the woman's information? 36. According to phrenology, what determines a person's character? 37. What does the woman say about the sections of the brain?
x

24.3 PART C

38 to 42 Talk given in a library science class. In the early 1800s, the paper industry was still using rags as its basic source of ber as it had for many centuries. However the rag supply couldn't keep up with the growing demand for paper. The United States alone was using 250 thousand tons of rags each year. And a quarter of that had to be imported. It was clear that a new source of ber was needed to keep up with the demand for paper. The answer to this problem turned out to be paper made from wood pulp, something that was abundantly available in north America. IN Canada, the rst wood pulp mill was set up in 1866 and it was immediately successful. But while wood pulp solved the problem of quantity it created a problem of quality. Wood contains a substance called lignin. The simplest way to make large quantities of cheap paper involved leaving the lignin in the wood pulp. But lignin is acidic and its presence in paper has shorten the life expectancy of paper from several centuries for rag pager to less than a century for paper made from wood

144

\]gho 96 d 10 nj^i`kp

pulp. This means that books printed less than a hundred years ago are already turning yellow and beginning to disintegrate, even though books printed much earlier maybe in the conditions. This is bad enough for the older books on your bookshelf but it opposes a huge problem for libraries and the collections of government documents. 38. What does the speaker mainly discuss? 39. What did the paper industry need a new source of ber in the early 1800s? 40. What can inferred about the rst wood pulp mill in Canada? 41. According to the speaker, what is the problem with lignin? 42. According to the speaker, what problem do libraries face? 43 to 47 Radio announcer talking about a current topic. A recent report has shown that here in the United States, we've experienced an evolution concerning our attitudes towards the workweek and the weekends. Although some calendars still mark the beginning of a week as Sunday, more and more of us are coming to regard Monday as the rst day of the week with Saturday and Sunday comprising the two-day period thought as the weekend. In fact the word "weekend" didn't even exist in English until about the middle of last century. In England at that time, Saturday afternoons had just been added to Sundays and holidays as a time for workers to have o from their jobs. This innovation became common in the United States in the 1920s, but as the workweek were shortened during the Great Depression of the 1930s, the weekend expanded to two full days - Saturday and Sunday. Some people thought that this trend would continue due to increasing automation and the workweek might decrease to four days or even fewer. But so far this hasn't happened. The workweek seems to have stabilized as forty hours made up of ve eight-hour days. After this commercial I'll be back to talk about the idea of adding Monday to the weekend. 43. What is the speaker mainly discussing? 44. According to the speaker, what is changing in the way people think about the week? 45. According to the speaker, how has the amount of time people work changed from the early part of the century? 46. What does the speaker imply about the workweek in England in early 1800s? 47. According to the speaker, what a ect did some people think the increasing use of automation would have? 48 to 50 A professor talking to her music students. I don't think I have told you about my trip to Tanglewood's music festival. When I was in college, I won a music competition and the prize was a week at Tanglewood. Anyway it is one of the world's most famous music festivals and the summer pome the Boston Symphony

24.3 PART C

145

Orchestra. It is located in the beautiful Berkshire Hills in New England. The summer musical season consists given over about mine weeks: from just 1st through the rst week in September. The biggest stars on the music scene appear here. The year I went I was Lucky enough to see Leonard Bemstein conducting. I understand it is sometimes hard to get tickets but of course mine were a part of the prize. If you want to sit inside the tickets are expensive. It's much cheaper to sit outside on the lawn. But it might rain, or sometimes it is really cool even in the summer. Either way the sound system is excellent. So it doesn't really matter where you sit. I seem to recall that the festival got started in the 1930s. Some Berkshire residents invited a symphony orchestra to perform a few outdoor concerts. The concerts were so successful that after a couple of years that things really took o . And the festival has got bigger and better every year. Attending was such a wonderful experience. I'd love to be able to go again. And I hope that all of you would be able to go too. 48. Why did the professor originally go to Tanglewood? 49. According to the professor, what is the disadvantage of sitting on the lawn? 50. What does the professor imply about the festival? 51. What is the killdeer's nest in parking lot an example of? (A) 52. According to the speaker, what is a possible reason that birds began to build nests in trees? (A)

146

\]gho 96 d 10 nj^i`kp

46HRX 98 B 5 VN8L<PZ
98 5*

"0
x

25.1 PART A

1. The view is spectacular. Could you take a picture of me with the mountains in the background? I'm afraid I just ran out of lm What does the woman mean? (C) 2. Excuse me, were you ready to order now? I'll be with you in just minute. What does the man mean? (B) 3. I think I forgot my umbrella. Did you notice that it was raining outside? Yeah. It is. And I just realize that I left my car window open. What will the man probably do next? (C) 4. How does your daughter like her new school? Fine. She seems to have made new friends in no time. What can be inferred about the man's daughter? (C) 5. There is an article here in this magazine you might interesting. It's about buying running shoes. If it's not chemistry and it's not on the nal exam, I can't read it now. What is the man probably doing? (B) 6. You are washing your car even on vacation. It makes me feel guilty. You shouldn't. It's just that I have nothing better to do at the moment. What does the woman imply? (C) 7. My doctor told me I needed to go for some expensive treatment for my injured knee. Are you sure? Maybe you need a second opinion. What does the woman suggest the man do? (C) 8. Hello, could me t me in for an appointment today? I need a stylist who's good at cutting curly hair. Judy is good at that. She is the owner. How about the noon? 147

148

\]glo 98 d 5 nj^i`kp

What will the woman probably do? (B) 9. There is nothing I like more than a good mystery novel when I've got some spare time. I like to read too. But I prefer non- ction: history, social commentary and stu like that. What does the man imply? (C) 10. I'm no expert. But that noise in your refrigerator doesn't sound good. Maybe you should call and have it checked out. You are right. And I suppose I've put it o long enough. What will the woman probably do? (A) 11. What's up with Donald? I've never seen him so happy. His supervisor gave performance evaluations this morning. What can be inferred about Donald? (A) 12. It's not going to snow again tomorrow, is it? It was supposed to be warm all week. Well, if you go by the forecast that I heard, you shouldn't put your coat and hat away quite yet. What does the woman imply? (B) 13. I don't know how it happened. But I got two di erent appointments at lunch tomorrow. I'm supposed to meet both David and Jim. Why don't you make one of them a breakfast meeting? What does the woman suggest the man do? (B) 14. It's all right to wear jeans for a class presentation, isn't it? That's what I'm wearing. But if we wear jackets too, maybe we won't look so casual. What does the man imply? (C) 15. This spring weather is perfect for playing tennis. Unfortunately the only time I get to enjoy it is when I'm walking to class or to the library. What does the man imply? (D) 16. Boy, how quickly technology changes. So many people have a computer in their home nowadays. I know. I feel so behind the time. What can be inferred about the woman? (D) 17. Joan and her friend went to new restaurant last night and said that it served the best food they ever had. That's quite a recommendation. Maybe we should see for ourselves. What will the speakers probably do? (D)

25.1 PART A

149

18. I hear you have a brother who went to school here too. Have I ever seen him? Well, he graduated last year. But you would never have guessed that we were brothers. What does the man imply? (D) 19. I'm thinking of heading to the gym before going to dinner. Care to join me? If you don't mind waiting while I go get my gym bag.. What does the man mean? (B) 20. I'm in a terrible mood. My boss didn't like the report I wrote. Well, don't take it out on me. What does the man mean? (A) 21. So I guess it's been a year now since your last checkup. Have you had any health problems? None to speak of. What does the woman mean? (B) 22. Have you ordered your graduation announcements? No, I had Don do it for me. What does the woman say about the graduation announcements? (D) 23. I hear you got a big parking ticket. Yeah. I never realized Lot 3 was only for faculty. What does the man imply? (C) 24. I went through a whole box of paper and a printer ribbon just trying to get my resume right. It'll be worth it. You know just to make a good impression. What can be inferred from the conversation? (B) 25. So you weren't happy with the way the newspaper covered the rally protesting the rising tuition fees? No. The article underestimated the number of students who were there and I don't think it explained our point of view very well. What can be inferred about the man? (A) 26. When are you ever going to nish this report? You've been working on it for three months. Only two and half. But it does seem longer. What does the woman say about the report? (D) 27. I've been meaning to get my eyes checked. I just haven't gotten around to it yet. Why don't you call for an appointment right away? Once on your calendar you will get it done. What does the man suggest the woman do? (B)

150

\]glo 98 d 5 nj^i`kp

28. Hey, Lisa. Has Professor Smith returned my call yet? I just got in a little while ago myself. What does Lisa imply? (C) 29. I got an invitation to a nancial planning seminar. And I don't want to go alone. Count me in. I need all the help I can get managing my money. What will the woman probably do? (B) 30. I hope you are not too put out with me for stopping by Fred's on the way over here. I had to pick up an assignment. Well, that's not a big deal. But you might at least phone if you know you are going to keep someone waiting. What does the woman mean? (C)
x

25.2 PART B

31-34 Conversation at a registrar's o ce at a university. * I want to register for this mathematics course. * I'm sorry registration has closed. * Closed? The clerk told me I could come back and register any time during the rst week of classes. * Well, that's not possible. The computer's o cial student account has already been sent to the state. And that's what our budget is based on. Who told you that anyway? * Some woman here when I tried to register three weeks ago. She said I just had to pay a late fee. * She must have been a temporary worker. They don't have much training. Why didn't you register then? * She said I couldn't until I had my birth certi cate. Here it is. * Your birth certi cate? * Well, I'm a new part-time student. So she ask for identi cation. I don't drive so I don't have a driver's license. * Huh. That's no reason to demand a birth certi cate. We only need to establish residency: a phone bill with your name and address on it would've been ne. * Really? Only prove of my address? * Yes. I'm afraid she gave you the wrong information. Still you'll have to wait and take your math's class next semester.

25.2 PART B

151

* But that's on it fair. * Well, I sympathize with your problem, but frankly, I don't think there is anything anyone can do for you. You were trapped in the system. If you want to you can talk to the director. She will hilp you if she can. * Great. * Don't get your hopes up. 31. What problem does the woman have? (B) 32. Why does the woman have to go to the o ce two times? (D) 33. According to the man, what does the woman need to show the evidence of? (B) 34. Why does the man imply when he tells the woman "not to get her hopes up"? (A) 35-38 Conversation between two students doing chemistry experiment. * Hi, Mary. Do you want to start writing a lab report after we nish this experiment? * I can't. In fact I need to nish early because I'm going over to the psychology department to talk to Professor Smith about a job opening. * You mean a job on campus? * Yeah. And it sounds pretty interesting. It involves helping with your study on learning style. You know, about how some people learn best by sight, while others learn best by hearing or touch. * Yeah. I know that's an area of expertise. * Right. Anyway for her study she's taking some high school students who aren't doing very well in their classes and testing them to nd out what their learning styles are. Then tutors, people like me, will work with them presenting material to them in their particular learning style. * Hey. That is interesting. Now will you mostly do the testing or the tutoring? * Both I hope. I want to be involved from start to nish. * Are you getting paid for this? * I'm sure we'll get something though, probably not much. Anyway it doesn't matter to me, I just want to have some hands-on experience. * Yeah. And it'll be nice to help those high school students too. * That's what I thought when I saw the ad. You know you could do it too. You don't have to be in her classes to work on the study. * Really? Do you have any idea what the schedule is like? * Late afternoon then evening for tutoring I think. After all the kids are in regular classes until three thirty.

152

\]glo 98 d 5 nj^i`kp

* Actually that's perfect for me. * Then come along. We will save the lab report for later. But we'd better make sure we do a good job on our experiment rst. * Yeah. First thing's rst. 35. What are the speakers mainly discussing? (D) 36. Why is the woman interested in working with Professor Smith? (B) 37. What will the college students do for the high school students? (C) 38. What will the speakers probably do next? (D)
x

25.3 PART C

39 to 42 A museum guide talking about native American pottery. Hello, I'll be your tour guide today here at the art museum so I'd like to welcome you to this month's exhibit of native American pottery. We'll begin our tour in a few minutes. But rst I'm going to tell you something about the way this pottery was created. Pottery was made all over ancient North America by many di erent groups of people. One of the earliest of these ancient American cultures was the Hohokam people. They lived in what is now Arizona from about 300 BC to AD 1500. And it's their pottery that you will be looking at today. All of the pottery was made from clay. Some objects were mug, bowls and ladies for drinking and eating. You will also see nger rings and animal-shaped incense burners which we believe were probably used in special ritual. The Hohokam formed their pottery vessels from coils of clay. Then shaped them with special tools to create very thin sides on the vessels. Afterwards they painted the pottery with red design. Actually many of the pieces here have designs right on them that show how the pottery was used. Now I hope you'll enjoy the beauty and the uniqueness of the Hohokam pottery and that will give you some interesting insights about the people who created it. Please feel free to ask me any questions and thank you for joining us today. 39. What does the speaker mainly discuss? (C) 40. What is the purpose of the talk? (B) 41. What did the Hohokam do with their pottery? (B) 42. What does the speaker say about the way the hohokam pottery was made? (A) 43 to 46 A talk in an American history class. I'm going to introduce two current points of view about the motivation for writing the United States Constitution back in 1787. The rst one is called the idealist view. The idealists basically believe that the writers of the Constitution were motivated by ideas. Which ideas? The ideas of the revolutionary war, such as liberty and democracy. The idealists remind that the young

25.3 PART C

153

country had a lot of problems: an economic depression, a large war debts, lawlessness and trade barriers between the states. They argue that the representatives needed to control these problems in order for the United States to survive. The other point of view is the economic view. The economic view is that the writers of the Constitution were concerned about their own nancial interests. According to them most people were living wealth for the wealthiest people were afraid of losing their money. The writers wanted a strong central government that would promote trade protect private property and perhaps most of all collect taxes to pay o the United States' large war debts. Because a number of those who wrote the constitution had loaned money to the government during the revolution. Which view is correct? Well historians who wrote during the calm and prosperous 1950s found reasons to believe the idealist view. Those who wrote during the trouble of 1960s found support for the economic view point. I'd say that neither view is complete, both the idealist and the economic perspective contribute a part to the whole picture. 43. What is the talk mainly about? (D) 44. According to the economic view, who bene ted the most from the new Constitution? (B) 45. What can be inferred about the views of the historians ? (C) 46 to 50 Part of a speech on birds by a biologist. Many egg-laying animals merely lay their eggs and leave. Turtles for instance, the eggs hatch on their own. The current theory about birds is that the earliest birds did just that when they were cold-blooded creatures living in warm places. However when they became warm-blooded creatures living in cold places they had to remain on the eggs to keep them warm. The process we call incubation. For this they needed a place a nest. Very likely the rst nests were just primitive depressions scrape into the ground. Even now many species still lay eggs in this sort of crude nests. In fact every spring a mother killdeer lays her in some pebbles along the edge of the parking lot just outside this building. Primitive nests on the ground were ne for some birds but others began to elevate their nests in branches perhaps to avoid predators. These early elevated nests were probably loose platforms of sticks and twigs the types still built by ospreys and mostarians today. The latest evolvement in nest the most recent version, so to speak, is the cup-shaped nest. This is the one we regard today as the typical bird's nest, you know, like a robin's nest. 46. What is the main topic of the talk? (C) 47. What evolutionary change in birds led to nest building? (B) 48. According to the speaker, where were the rst bird's nests located? A) 49. What is the killdeer's nest in parking lot an example of? (A) 50. According to the speaker, what is a possible reason that birds began to build nests in trees?

154

\]glo 98 d 5 nj^i`kp

46H>X 98 B 8 VN8L<PZ
98 8*

"0
x

26.1 PART A

1. M: I don't think you have time to send out the invitations to all the students. W: Oh, yes. I will. Q: What do we learn about the woman from this conversation? (A) 2. W: Nobody told me that Bill was in the hospital. M: Sorry. I meant to give you a call when I found out but it slipped my mind. Q: What does the man mean? (D) 3. M: I don't know if I will be able to turn in my economics paper on time. W: Have you heard that the professor gave us a week of extension on it. Q: What does the woman mean? (B) 4. M: I'd like to make an appointment with the doctor for tomorrow. W: Unfortunately he is completely booked. Q: What does the woman mean? (C) 5. M: Joe just went down to the engineering meeting. W: Where is it? Q: What does the woman want to know? (A) 6. M: I have a collect call from Mike Peterson. W: I will accept the charges. Q: What does the woman mean? (B) 7. W: I'd like really to go to the concert tonight, but I don't know if I could spare the time. M: Music always relaxing me. It might be worth it in a long run. Q: What does the man suggest the woman do? (C) 8. M: Those airplanes are certainly loud. W: Aren't they though? Q: What does the woman think of the airplanes? (A) 9. M: Helen and I are thinking of renting a house at the beach in June. Are you interested? W: You? I guess it is cheaper then, but do you really think it will be warm enough? 155

156

\]gao 98 d 8 nj^i`kp

Q: What does the woman mean? (D) 10. M: I'm getting hungry. I think we should go to the dinner soon. W: Me too. All I have for lunch was a chocolate bar. Q: What does the man mean? (C) 11. M: Your apartment always looks so good. So spotless and mine is such a mess. W: I've been at the lab the all week. It is my roommate doing. Q: What does the woman imply? (A) 12. M: I am running out of coins during my laundry. W: That's too bad. Q: What does the woman mean? (C) 13. W: It's a shame that you didn't win your tennis match. M: I might have won if I listened to my coach. Q: What does the man imply? (C) 14. M: The Variety Theatre nally went out the business. W: Well, that's not a surprise. It was the worst one in town. Q: What does the woman mean? (B) 15. W: Shall we run around the park or go for a bike ride? M: It makes no di erence to me. They are both good activities. Q: What does the man say about the activities. (D) 16. M: I can't to the life to get that washing machine downstairs to work. Do you have any suggestions? W: Try washing just half of the normal load. Q: What does the woman suggest the man do? (A) 17. M: I'm think of dropping my swimming class. I am just not catching on. W: Stick with it. I did and I learned how to swim eventually. Q: What can be inferred about the woman? (C) 18. M: Doctor, this cough medicine doesn't seem to helping. Can you give me a di erent prescription? W: Let's give another day or two to see how you are doing then. Q: What does the doctor imply? (D) 19. W: Would you like to see those pants in another color? They are also coming in brown and in Navy.

26.1 PART A

157

M: Actually the gray is ne but I prefer something in wool. Q: What will the woman probably do next? (D) 20. W: Professor Burns seems to think that there is only one way to write paper and that's her way. M: No kidding, she sure wasn't like that the last semester. Q: What can be inferred about professor Burns? (B) 21. W: This Barbecue sure beats the last one we went to Ha? M: Oh that's right. Everyone had to spend the whole time inside. The good thing is the weather decided to cooperate this time around. Q: What can be inferred from this conversation? (D) 22. M: That new position requires a letter of reference. I guess the one the professor wrote for me last year should be ne. Don't you think? W: It is a little dated though. You might need to submit a current one. Q: What does the woman suggest the man do? (A) 23. W: I don't think I want to be on the curriculum committee anymore but I'm not sure how to get out of it . M: Well, you know there are plenty of people who will be interested. Me, for example. Q: What does the man imply? (C) 24. W: Excuse me could you do for me the Customer Service? I need to have this gift wrapped. M: We can take care of that right here man, and no charge. You can choose either silver or gold with the matching bowl. Q: What will the woman probably do the next? (B) 25. M: These plants next to the window always look brown. You wouldn't know by looking at them that I have the watered them every week. W: Maybe they don't like the direct sunlight. I have the same problem of some of my plants and little shade could help them immensely. Q: What does the woman suggest the man do? (A) 26. W: Oh no I just picked up the pictures I took at Dan and Linda's wedding and looked at them and none of them came out. M: They are dark, aren't they? What a shame. Well I'm sure the professional photographer got everything. Q: What does What does the man mean? (A) 27. M: I get a feeling that Sally never really listens to me.

158

\]gao 98 d 8 nj^i`kp

W: You said it. As it is that she is always using the time to rehearse what she will say next. Q: What does the woman imply? (D) 28. W: Will you make sure all the members of student advisory committee know what to expect at the tomorrow's meeting? M: They will have a brie ng this afternoon. Q: What does the man mean? (D) 29. W: How do you like my new poster. It was only twenty dollars. M: Really? The frame alone is worth the money. Q: What does the man mean? (D) 30. M: I hear Mary isn't getting much support in her running against Steve in the election. W: It is not over yet. I think she will make a come back. Q: What does the woman mean? (A)
x

26.2 PART B

Question 31-34. Listen to a conversation between two friends. M: I have been studying too much and need a change. So I just making plans to go away during January break. W: Really? Where are you going? M: I'm planning to visit New Mexico. W: My sister and I had the vacation there last year and we had a great time. M: Did you get into Albuquerque? W: Sure. Whenever we were skating. M: Is it far from the mountains? W: Not at all. There are even though Albuquerque on the high plateaus. There are even higher mountains near it. Just half an hour away from the city there is a snow-covered slope. M: Well. As the mountains are just thirty minutes away, I guess I should take my ice skate and my ski's. W: De nitely. M: I heard that the weather there is great. W: It is. No humidity, moderate temperatures, but you do need to be careful about high altitude. M: What should I do about that?

26.2 PART B

159

W: Oh, just take it easy for a few days. Don't go hiking up to the mountains or exercise too vigorously. Just do everything gradually. M: I'm sure I will be ne. And I will let know all about my trip when I come back. 31. What's the main purpose of the man's trip? (B) 32. Why does the woman know so much about Albuquerque? (D) 33. What can be inferred about the man? (C) 34. According to the woman, what may cause the man the most problems in the Albuquerque? (C) Question 35-38. Listen to two students talking about eating in the school cafeteria. M: Hey Linda, do you get that letter about the new options for food service next year? W: Not yet. Are there a lot of changes? M: There sure are. Instead of paying one fee to cover all meals for the whole school year, we are now be able to choose by seven, ten, fourteen or twenty-one meals per week. They give you a card with the number of meals you get for a week marked on it. W: That's a big change Tom. And a complicated system. M: Yeah. But it will be much better for people who don't eat three meals a day, seven days a week in the cafeteria because they don't have to pay meals they don't eat. W: So what's the deal for those who do eat at school all the time? M: It's better for them too. Because the meal you contract, the cheaper each one is. W: I see. It is still sound rather complicated. M: True. It took me several hours to gure it out. I decided to go with the ten Meals. W: Why is that? M: Well, I never eat breakfast and I often go away on weekends. So the ten meal plan gives me lunch and dinner each weekday at a fairly low price. And I won't be paying for meals I don't usually eat. W: And what about the weekend when you are on campus? M: Well, there are often guests on campus at weekends. So they allow you to buy single meals on a walk-in basis on Saturdays and Sundays. The price per meal is much higher in that way. But I an away so much that it will still be less money for me to pay single prices on the weekends rather than sign up for the fourteen meal a week plan. W: Oh, I guess I'll have to sit down and gure out my eating pattern so I can get the best deal. 35. What's the main feature of the new method of paying for meals? (B) 36. When do the students pay for the meals they contract for? (C)

160

\]gao 98 d 8 nj^i`kp

37. How does the new plan bene t the students who eat all their meals at the school cafeteria? (D) 38. How can weekend guest eat at the cafeteria? (A) Questions 39 to 42. Listen to this talk being given to college campus. I was really glad when your club invited me to share my coin collection. It's been my passion since I collected my rst Lincoln dime in 1971. That is the current coin with Abraham's image. Just a little history before I started my own collection. Lincoln pennies are made of copper and they were the rst the United States coins to bear the lightness of the president. It was minted in 1909 when the country was celebration the centennial of Lincoln's birth than 1809 that the decision was made to redesign the one-cent piece in his honor. Before that, the penny has an American Indian head on it. The new penny was designed by artist Victor David Braner. It is interesting because he put his initials DVB on the reverse of the coin ad the original design. There was a general abort when the initial was discovered. And only a limited numbers of coins were strutted with the initials on them. Today a penny with the initials from a San Francisco mint called the 1909s' DVB was worth 500 dollars. Now when I started my coin collection, I began with penny for several reasons. There were a lot of them, several hundred billion were minted and there were a lot of people collecting them. So I have plenty of people to trade with and talk to about my collection. Also it was the coin I could a ord to collect as a young teenager. In the twenty- ve years since then, I have managed to acquire over three hundred coins some of them are very rare. I will be sharing with you today some of my rare specimen including the 1909s' DVB. 39. Why does the woman collect coins? (C) 40. Why were letters DVB on pennies? (D) 41. What was one of reasons the collector collected coins as a teenager? (B) 42. What will the speaker do next? (B) Question 43 to 46. Listen to talk from a biology class. Today I want to talk to you about the wasps and their nests. You recall the biologist divided species of wasps into two groups solitary and social. Solitary wasps as the name implied do not live together with other wasps. In most species the male and female get together only to mate and then the female does all the work of building the nest and providing the food for the o spring by herself. Solitary wasps usually make nest on the ground and they separate the chambers for the individual o spring with the grass, stone or mud, whatever is handy. What about social wasps? They form a community and work together to build and maintain the nest. A nest begins in the spring when the fertile female called the queen build the rst new compartment in the nest and lay eggs. The rst o spring are females but cannot lay eggs. These females called

26.2 PART B

161

workers. They build a lot of new compartments and the queen lays more eggs. They also care for the new o spring and defend the nest with their stingers. By the way only the female have stingers. Most social wasps make nest of paper. The female produces the paper by chewing out bers or old wood. They spread the papers in thin layers to make cells, which the queen lays her eggs. Most of you I'm sure have seen these nests suspended from the trees. They may also be built on the ground in abundant road bowls. 43. Who builds the nest of solitary wasps? (C) 44. Why the female wasps are more dangers to people than the male wasps are? (D) 45. What is the main function of the queen ? (B) 46. What are the nests of social wasps made of? (C) Question 47 to 50. Listen to a talk in class about United States history. What was the most popular mix about the United States in the 19th Century was that of the free and simple life of the farmer. It was said that the farmers worked hard on their own land to produce whatever their families' needed. They might sometimes trade with their neighbors, but in general they could get along just ne by relying on themselves, not on commercial ties with others. This is how Thomas Je erson idealized the farmers at the beginning of the 19th century. And at that time, this may have been close to the truth especially on the frontier. But by the mid century sweeping changes in agriculture were well under way as farmers began to specialized in the raising of crops such as cotton or corn or wheat. By late in the century revolutionary invents in farm machinery has vastly increased the production of specialized crops and extensive network of railroads had linked farmers throughout the country to market in the east and even overseas. By raising and selling specialized crops, farmers could a ord more and ner goods and achieved much higher standard of living but at a price. Now farmers were no longer dependent just on the weather and their own e orts, their lives were increasing controlled by the banks, which had powder to grant or deny loans for new machinery, and by the railroads which set the rates for shipping their crops to the market. As businessmen, farmers now had to worry about national economic depression and the implement of world supply and demand on for example, the of price of wheat in hands. And so by the end of the 19th century, the era of Je erson's independent farmer had come to a close. 47. What is the main topic of the talk? (D) 48. According to the professor, what was the major change in the agriculture during the 19th century? (A) 49. According to the professor, what was one result of the increased use of machinery on farms of the United States? (B) 50. According to the professor, why was world market important for the United States agriculture? (C)

162

\]gao 98 d 8 nj^i`kp

46HDX 99 B 1 VN8L<PZ
99 1*

"0
x

27.1 PART A

1. Why are you leaving so early? The movie doesn't start till seven. I don't want to be at the tra c there. It's a nightmare on the express way during rush hour. What does the man mean? (A) 2. Excuse me, but could you tell me how to get to the Excelsior Hotel? I thought it was on this corner but I seem to have made a mistake. Hmm. I'm sorry. Maybe you should try calling them. There is a phone over there by the candy store. What does the woman suggest the man do? (C) 3. Can I borrow your calculus textbook? I left mine in the classroom. And it was gone when I went back. That happened to me once. I'd almost given up on nding it until I checked it at the lost-andfound at the information desk downstairs in the lobby. What does the woman imply about the man should do? (D) 4. Did you see the diamond ring Bill gave to Linda? I sure did. It must have cost him an arm and a leg. What does the woman imply about the ring (C) 5. I'm always late for my morning classes. It's because of all the tra c out near where I live. Well, you wouldn't have that problem if you move into our campus. What does the man suggest the woman do? (D) 6. Jennifer is going to the shore again this weekend. Well, she's always been a beach person. What can be inferred about Jennifer. (A) 7. Hey, Larry. Wanna meet a few of us for co ee in a little while? Hmm. I would if I weren't so far behind in this reading I'm doing for history. What will the man probably do? (A) 8. Oh, something in this room is making my eyes edge. I must be allergic to something. Hmm. I wonder what it is. 163

164

\]geo 99 d 1 nj^i`kp

What does the woman want to know? (C) 9. My brother is coming this weekend and I thought three of us could go out to dinner Saturday night. Any suggestions? It's up to you. I don't know the restaurants around here that well. So you know a better place to go than me. What does the man mean? (B) 10. Can I get a ride into the o ce with you tomorrow? Another day would be ne. But I got to be downtown for a meeting rst thing in the morning. What does the woman imply? (D) 11. After two weeks of tennis lessons I think I nally managed to improve my backhand. Like my mom always says: "Practice makes perfect." What does the woman mean? (B) 12. I just heard about your acceptance into law school. Do you think you will be able to join your brother's rm when you graduate? Not likely. He is a tax lawyer and I'm going to major in criminal law. What does the woman imply? (D) 13. Could you give me a ride to the dean's o ce? My interview for the scholarship is in an hour. Don't you have anything else to wear other than that sweater? What does the woman imply? (D) 14. Well, if you are seriously considering buying a car, I'm trying to get rid of mine. All it needs is some new paint. Thanks. But most used cars end up being more trouble than they are worth. What will the man probably do? (C) 15. Did you watch that comedy special on TV last night? I don't think I ever laughed so hard. Don't even talk to me about it. The only night I really wanted to watch something and we had a power fail in my building. What does the woman mean? (A) 16. Do you think it'll be able to get this ink stain out of my pants? It won't be a problem but I'll need to send them over to the main cleaning facility. That's an extra day's time. What does the woman mean? (A)

27.1 PART A

165

17. You'll have to be a lot more persuasive if you want to convince the committee to accept your proposal. I'm not sure what else I could say. Besides I don't think they will reject it. What does the woman mean? (C) 18. I'm having trouble slicing the bread with this knife. Oh. Sorry about that. I haven't gotten around to sharpening it yet. What can be inferred about the knife? (B) 19. Do you know if Sarah has reserved the room for the committee meeting yet? No. But if she hasn't we should have her try to get it at the auditorium. We'll need the space. What does the woman want Sarah to do? (C) 20. I lost the piece of paper Laura gave me. You know, the one with her address on it. You might be able to nd it listed in the phone book/ What does the woman mean? (D) 21. I heard on the radio that not only is it going to be super hot tomorrow but also the humidity's going way up. Sounds like I'm going to have to nd an air-conditioned place to be in. What does the man mean? (B) 22. I can't concentrate on this nal report any longer. Maybe I should take a nap before we continue. You know they say the physical activity makes you more alert. What does the woman imply? (B) 23. I hope you like the novel I lent you. I wasn't sure whether it was the kind of book you would be interested in. You know, I had the same doubt at rst. But once I started I simply couldn't put it down. What does the man mean? (C) 24. The museum exhibit that our professor recommended just closed. Last day yesterday. Oh. I was really looking forward to seeing it. What does the man mean? (C) 25. If George misses one more meeting we are going to have to nd one new committee secretary. We'd better give him a n ultimatum. What does the man suggest they do? (B)

166

\]geo 99 d 1 nj^i`kp

26. Are you sure you don't mind getting the concert tickets? I wouldn't be able to pay you back until Friday when I get paid. No problem. I'm glad I can help and we'll be able to go together. What does the man mean? (C) 27. Have heard the news? The manager posted this month's work schedule. She did? Where? What does the woman ask? (C) 28. Will you be living in the dormitory this year? Not if I can help it. I've been thinking of renting an apartment o campus with some friends of mine. What does the man imply? (D) 29. The plot of that movie is hard to nd. It makes more sense the second time. What can be inferred about the woman? (B) 30. I'm so soaked from the rain. I'd go back to my room to change my clothes if there were ore time before the performance. I could use drying o too. But I hate the idea of missing even a few minutes of this concert. What can be inferred about the speakers? (D)
x

27.2 PART B

31-34 PART OF A TELEPHONE INTERVIEW * Dr. Thomas? This is Keet Bradley from the daily news. I'd like to ask you some questions about the new o cial standard weight that you purchased. * I'd be happy to help you. What would you like to know? * First of all, how was the standard weight used? * Well, the people in our department use it to check the scales all over the country. The department of weights and measures, we are a government agency. It's our responsibility to see that all the scales measure a kilogram accurately so this is the way we use to adjust the scales. * How did you check the scales before? * We have an old standard weight that we used to use. It had to be replaced because it was imprecise. You see it was made of poor quality metal that was too porous. It absorbed too much moisture.

27.2 PART B

167

* Oh. So when the weather was humid it weighed more and when it was dry it weighed less. * Exactly. And that variation can a ect the standards of the whole country. So our department had the new weight made out of higher quality metal. * How much did it cost.? * About 45 thousand dollars. * 45,000 dollars? For a one kilogram weight? That's more expensive than gold. Is it really worth that much? * I'm sure it is. Industries depend on our government agency to monitor the accuracy of scales so that when they buy and sell their products there is one standard. Think of the drug industry, for example, those companies rely on high accuracy scales to manufacture and package medicine. 31. What is the conversation mainly about? (B) 32. How was the weight used? (A) 33. Why was it necessary to replace the old standard weight? (C) 34. What does Dr. Thomas probably think about the cost of the new weight? (C) 35-38 CONVERSATION BETWEEN A STUDENT AND A PROFESSOR * Elizabeth Martin speaking. * Dr. Martin, my name is Mark Johnson. My roommate, Benjamin Jones, is in your art history class. Uh-m, Art History 502? * Yes. * Well, he is sick and won't be in your class today. He asked me to bring his term paper to your o ce. * OK. The paper is due by 3 o'clock. * I have a class from 1 to 2. I'll bring it to your o ce after my class. * Well, I have a meeting this afternoon. So you can drop it o with the secretary of the art history department. She'll see that I get it. * Ok. Oh I almost forgot. I'm a biology major. But my advisor told me that I need one more humanities course to graduate. I've noticed that you are teaching a course on landscape painters next semester. Could tell me a little bit about it? * Sure. Well, it's a course for non-art majors. We'll be looking at several di erent painters and examining their works. We'll be looking at several di erent painters and examining their works. We'll also look the history and politics of the era in which they lived. * That sounds interesting. What else is required? * There is no nal exam. And there is only one required book. But each student has to give a major presentation about the individual painter at the end of the course.

168

\]geo 99 d 1 nj^i`kp

* Hmm. It sounds good. Will you be in your o ce later today? I'd to talk to you some more. * Well, my meeting's scheduled to last all afternoon. Why don't you stop by tomorrow? Any time in the afternoon. My o ce is in the ne arts building right next to the library. * Thanks. I'll do that. 35. Why does the man call the woman? (B) 36. What does the man almost forget to do? (A) 37. What will the students be required to do in the course the woman describes? (B) 38. What does the woman suggest the man do? (C)
x

27.3 PART C

39 to 42 TALK IN AN ARCHITECTURE CLASS As you probably know, log structures are gaining popularity. They are no longer just the simple country homes that we think of as the traditional log cabin. Some upscale homes now incorporate natural round logs in sealing beams and walls. People seem to think that the rounded logs give their homes a cozy warm atmosphere. And even people who want to build a traditional log cabin on their own can buy a kits with precut logs that t together like pieces of lig-saw puzzle. Before showing you some slides of modern log houses, I'd like to give you a little historical background on the subject. Log cabins were rst built in the late 1600s along the Delaware river valley. The European immigrants who settled there brought centuries' old traditions of working with logs. And in this heavily wooded area logs were the material in hand. Log cabins were the most popular in the early 1800s with the settlers who were moving west. They provided the answer to the pioneer's need for a sale and sliding boards for windows. But the log buildings that have probably had most in uence on modern architects are those of the mountain retreats of wealthy New Yorkers. These country houses which were popular in the early 1900s typify what's known as the Adoroundyx style. Now let's look at those slides. 39. What is the speaker mainly discussing? (C) 40. According to the speaker, what gives modern log homes their warm atmosphere? (C) 41. Why did the early settlers use log for building homes? (B) 42. According to the speaker, why were log cabins especially popular with settlers who moved west? (A) 43 to 46 TALK IN A GEOGRAPITY CLASS The Old Canada Road is a long lost trail between the Canadian province of Quebec and Maine in the northeast corner of the United States. Yes it really was lost and nding it again was

27.3 PART C

169

a complex process that involved state of our technology: how the location of the roads was pinpointed was very interesting. And I'll return to it as soon as I gave you a little background information. The road was begun in 1817, a few years before Maine even became a state. At the time Quebec was a major market for livestock, crops and sh. So a road to Quebec was seen by o cials in Maine as necessary for trade. For about 20 years the movement of people and goods was mostly from Maine to Quebec, and then the trend reversed as thousands of Canadians immigrated to Maine to escape poor crops, the lack of jobs and the threat of disease. I think it was a color epidemic. Besides its negative reasons major building projects in Maine also made the state very attractive for the Canadians who needed work. I should stress though that immigration during that period went in both directions. In fact the ow of people and goods went completely unhindered. There wasn't even a border post until around 1850. The people of the time saw Maine and Quebec as single region mainly because of the strong French in uence which is still evident in Maine today. Eventually the road fell into disuse as a major railway was completed. Finally people simply forgot about it and that's how it came to be lost. This brings me back to the original topic. 43. What does the speaker say about the road between Main and Quebec? (B) 44. What is one reason Canadians began to immigrate to Maine during the 1800s? (B) 45. What can be inferred about the region including Maine and Quebec during the early 1800s? (A) 46. What subject is the speaker most likely to discuss next? (D) 47 to 50 LECTURE IN A COLLEGE CLASS OK, in the last class we talked about the classi cation of trees and we ended up with a basic description of angiosperm. You remember that those are plants with true owers and seeds that develop into fruits. The common broad leaf trees we have on campus fall into this category. But our pines don't. Now I hope you all followed my advice and wore comfortable shoes because as I said today we are going to do a little eld study. To get started let me describe a couple of broadleaf trees we have in front of us. I'm sure you've all noticed that this big tree next to Brett Hall. It's a black walnut that must be 80 feet tall. As a matter of fact there is a plaque identifying. It is the tallest black walnut in the state. And from here we can see the beautiful archway of trees at the commons. They are American elms. The ones along the commons were planted when the college was founded 120 years ago. They have distinctive dark green leaves that lock lopsided because the two sides of the leaf are unequal. I want you to notice the elm right outside the Jackson Hall. Some of the leaves have withered and turned yellow, maybe due to Dutch elm disease. Only a few branches seem a ective so far but if this tree is sick it'll have to be cut down. Well, let's move on and I'll describe what we see as we go. 47. What are the students going to do during this class period? (D)

170

\]geo 99 d 1 nj^i`kp

48. In what class is this lecture probably being given? (C) 49. What is remarkable about the black walnut tree outside Brett Hall? (C) 50. What is the problem with the elm tree near Jackson Hall? (B)

You might also like